Sie sind auf Seite 1von 194

Anatomy 2014

Anatomy
Marking: +1 Marks for correct answer and no negative Marking for incorrect answer.

Test Duration (mins) :120

Time Left (mins) : 120

Instructions

Once the test has started, do not press the refresh button (or F5 on your keyboard)
It is advisable to save the test regularly to avoid losing your information, save test refers to
storing the attempted part of the test.
In case of accidental failure of internet connectivity the system will save the attempted portion of
the test automatically.
Once the test time is over, you will be awarded a grace duration to wrap up the test and "Submit"
it.
Submit test refers to the final completion of test; once you submit the test you will not be able to
edit / preview your answers.
Please ensure that you are connected to the internet, while submitting the test.
ANATOMY
Part
(Q.1)

Structures attached to the tip of coracoid process is/are


(a)

Long head of biceps and coracobrachialis

(b)

Short head of biceps and coracobrachialis

(c)

Coracoclavicular ligament

(d)

Pectoralis minor

Your Response :

Correct Answer :
Exp:

b
Short head of biceps and coracobrachialis
Reference: Grays Anatomy, 39th edition
Pg. 11, B D Chaurasia (4th Ed.); Vol-1
The third muscle attached to it is pectoralis minor, which is on the superior
surface.
Coraco-clavicular ligament is also attached to the superior surface of the
coracoid process.

(Q.2)

A. patient came with history of fall and on examination there was tenderness between
the tendons of extensor pollicis longus and brevis. The likely lesion is
(a)

Scaphoid #

(b)

1st metacarpal #

(c)

Lower end of radius #

(d)

Trapezoid #

Your Response :
Correct Answer :
Exp:

a
Scaphoid #* Pg. 130, B D Chaurasia (4th Ed.); Vol-1
EPL & EPB form the boundaries of the anatomical snuff box. Scaphoid
bone lies at the floor of this box and its fracture leads to tenderness in the
box.
Scaphoid fracture is a common injury encountered in family medicine. To
avoid missing this diagnosis, a high index of suspicion and a thorough
history and physical examination are necessary, because early imaging
often is unrevealing. Anatomic snuffbox tenderness is a highly sensitive
test for scaphoid fracture, whereas scaphoid compression pain and
tenderness of the scaphoid tubercle tend to be more specific. Initial
radiographs in patients suspected of having a scaphoid fracture should
include anteroposterior, lateral, oblique, and scaphoid wrist views.
Magnetic resonance imaging or bone scintigraphy may be useful if the
diagnosis remains unclear after an initial period of immobilization.
Nondisplaced distal fractures generally heal well with a well-molded short
arm cast. Although inclusion of the thumb is the standard of care, it may
not be necessary. Nondisplaced proximal, medial, and displaced fractures
warrant referral to an orthopedic subspecialist. The scaphoid bone is the
most commonly fractured carpal bone; this injury occurs most often in

young men. Scaphoid fractures are rare in young children and the elderly
because of the relative weakness of the distal radius compared with the
scaphoid in these age groups.1Scaphoid fractures are significant because a
delay in diagnosis can lead to a variety of adverse outcomes that include
nonunion, delayed union, decreased grip strength, decreased range of
motion, and osteoarthritis of the radiocarpal joint. Timely diagnosis,
appropriate immobilization, and referral when indicated can decrease the
likelihood of adverse outcomes.
(Q.3)

Fascia around nerve bundle of brachial plexus is derived from


(a)

Prevertebral fascia

(b)

Deep cervical fascia

(c)

Clavipectoral fascia

(d)

Pectoral fascia

Your Response :
Correct Answer :
Exp:

a Exp. Prevertebral fascia


Reference: Grays Anatomy, 39th edition
Pg. 68; B D Chaurasia (4th Ed.); Vol-3
*Fascia around the brachial plexus is called as axillary sheath and is a
derivative of pre-vertebral fascia (a).
*Prevertebral fascia (PVF) covers the anterior vertebral muscles and lies
on the anterior aspect of scalenus anterior and medius, thus forming the
floor of posterior triangle of neck.
*Brachial plexus emerge between scalenus and medius in the neck and
pass behind the clavicle along with subclavian artery to reach the axilla.
-In the process they carry an extension of PVF over them as a cover (the
axillary sheath) towards the axilla.
-Subclavian/Axillary veins lie outside the axillary sheath and therefore can
distend freely.
*Applied anatomy:
*Neck infections behind the PVF are usually due to tuberculosis of
cervical vertebra and may form chronic retropharyngeal abscess a
bulging in the posterior wall of pharynx.
-The pus may track into the axilla via the axillary sheath and point in the

posterior/lateral wall of axilla.


-The pus may also extend into the superior mediastinum but doesnt reach
the posterior mediastinum, since the PVF fuses to the fourth thoracic
vertebra.
*PVF is separated from the pharynx/buccopharyngeal fascia by the retropharyngeal space.
-Neck infections in front of PVF in the retropharyngeal space forms acute
retropharyngeal abscess which bulges forward in the paramedian position.
-This is due to attachment of PVF with Buccopharyngeal fascia in the
median plane.
-This infection may spread to the posterior mediastinum via the superior
mediastinum.
*Most of the nerves in the neck are behind the PVF but spinal accessory
nerve lies superficial to it and may get damaged quite often.
-Spinal accessory nerve is the most common nerve damaged iatrogenically.
This happens mostly during I&D (Incision & Drainage) procedures in the
neck.
(Q.4)

'Mallet finger' is a common traumatic lesion resulting in flexion deformity of distal


interphalangeal joint. The basic pathology of this condition is
(a)

Avulsion fracture of middle phalanx

(b)

Rupture of collateral slips of extensor expansion

(c)

Rupture of central slip of extensor expansion

(d)

Dislocation of distal interphalangeal joint

Your Response :
Correct Answer :
Exp:

b
Rupture of collateral slips of extensor expansion*
(7th Ed.)

Pg. 548; Snells

'Mallet finger' describes the characteristic drooping of the finger observed


with injury to the extensor digitorum at its insertion at the base of the distal
phalanx (the collateral slips).
A mallet finger may result from direct trauma to the finger tip or more
commonly, by forcibly bending the finger in extension, for example, when
catching a ball - hence, the synonym, "baseball finger."

In older patients, the associated trauma may be minor for example,


catching the tip of the finger whilst changing the sheets on the bed.
The extensor mechanism is disrupted. The finger is held flexed at the distal
interphalangeal joint. There is no active movement but passive movement
is unimpaired. In time, the proximal phalanx may become hyperextended.
An x-ray should be obtained to look for any associated chip fracture.
Most injuries are treated conservatively. A mallet finger splint is applied to
hold the distal interphalangeal extended whilst permitting movement at the
proximal interphalangeal joint.
If conservative treatment fails then part of the tendon is excised over the
terminal phalanx and the joint held extended with a Kirschner wire.
Extensor expansion (dorsal expansion, dorsal hood) refers to the flattened
tendons (aponeurosis) of extensor muscles that run into the back of the
hand.
At the distal end of the metacarpal, the extensor tendon will expand to
form a hood, which covers the back and sides of the head of the metacarpal
and the proximal phalanx.
The expansion soon divides into three bands:
Two lateral bands pass on either side of the proximal phalanx and stretch
all the way to the distal phalanx. The lumbricals of the hand, the palmar
& dorsal interossei of the hand, also insert on these bands.
A single median band passes down the middle of the finger along the back
of the proximal phalanx, ending at the base of the middle phalanx
(Q.5)

Infection draining the ring finger goes to


(a)

Thenar space

(b)

Mid palmar space

(c)

Ulnar bursa

(d)

Radial bursa

Your Response :
Correct Answer :
Exp:

b
Mid palmar space Pg. 127; B D Chaurasia (4th Ed.); Vol-1
Spaces:

A middle septum attaches the triangular palmar aponeurosis to the third


metacarpal thus dividing the palmar space into a medially placed midpalmar space & laterally placed thenar space.
Infection from thumb and index finger passes towards the thenar space
along the first lumbrical canal.
Middle, ring finger and little finger drain towards mid palmar space along
the 2, 3 & 4th lumbrical canals.
Bursa(s):
Flexor tendon of the thumb has a synovial sheath around it and is called as
radial bursa.
Flexor tendons of the other 4 fingers are enclosed in the ulnar bursa,
which continues into the little finger.
Thumb infection may lead to inflammation of the radial bursa, whereas,
little finger infection involves the ulnar bursa.
Fore arm space of Parona lies proximal to the flexor retinaculum and is
continuous with the radial & ulnar bursa.
Flexor retinaculum separates Fore arm space of Parona from the thenar &
mid-palmar space and they are non-continuous.
Note: Bursa is defined as a potential space lined by synovial membrane.
(Q.6)

Nerves directly related to the humerus include all EXCEPT:


(a)

The axillary nerve

(b)

The radial nerve

(c)

The ulnar nerve

(d)

The musculocutaneous nerve

Your Response :
Correct Answer :
Exp:

d
The musculocutaneous nerve Ref. BDC vol I 3rd ed. 18
Three nerves are directly related to the humerus& and therefore, liable to
injury: the axillary ay the surgical neck, the radial at the radial groove, and
the ulnar behind the medial epicondyle.

(Q.7)

Which of the following structures pass between the External and Internal carotid
arteries ?

(a)

Styloglossus

(b)

Stylopharyngeus

(c)

IXth Cranial nerve

(d)

All of above

Your Response :
Correct Answer :
Exp:

d
All of above
ORDER OF ARRANGEMENT AT SPECIFIC SITES
FEMORAL TRIANGLE

Within femoral sheath from


medial to lateral side- VAN (vein
artery - nerve )
Femoral canal- lymph node of
cloquet/ Rosenmuller
Middle compartment- femoral
vein
Lateral compartment- femoral
artery with femoral branch of
genitofemoral nerve
Outside of sheath and lateralmostfemoral nerve

AT APEX OF FEMORAL
TRIANGLE

From anterior to posterior


Femoral artery
Femoral vein
Profunda femoris artery and vein

POPLITEAL FOSSA

Upper part: medial to lateral


artery- vein- tibial nerve
Middle part: superior to deepnerve-vein-artery
Lower part: medial to lateralnerve-vein-artery

ANTERIOR ASPECT OF ANKLE


THROUGH EXTENSOR

Medial to lateral (The Himalayas

RETINACULUM

Are Never Dry Places )


Tibial anterior muscle
Extensor Hallucis longus
Anterior tibial artery
Deep Peroneal nerve
Extensor Digitorum longus
Peroneus tertius

THROUGH FLEXOR
Medial to lateral(Anterior to
RETINACULUM BEHIND MEDIAL posterior) -( The Doctors Are Not
MALLEOUS
Here)
Tibialis posterior
Flexor Digitorum longus
Posterior tibial artery
Tibial nerve
Flexor Hallucis longus
CUBITAL FOSSA

Medial to lateral (MBBR)


Median nerve
Brachial artery
Biceps tendon
Radial nerve

STRUCTURES WITHIN THE


PAROTID GLAND

Medial to lateral
Arteries- Maxillary and
Superficial temporal
Retromandibular vein
Facial nerve
Parotid lymph node

THYROID GLAND

Dense capillary plexus lies deep to


the capsule
Thyroid is removed along with the

true capsule
PROSTATE

Capillary plexus is present


between true and false capsule
Prostate removed and both the
capsules are left behind

At aortic opening of Diaphragm(T12) Left to RightAorta


Thoracic duct
Azygous vein
ROOT OF LUNG

From before backwards (SVAB)


(same on both sides)
Superior Pulmonary Vein
Pulmonary Artery
Bronchus
From Above downwards
RIGHT SIDE
Eparterial Bronchus
Pulmonary Artery
Hyparterial branch
Inferior pulmonary vein

LEFT SIDE (ABI)


Pulmonary artery
Bronchus
Inferior pulmonary vein
IN BRONCHOPULMONARY
SEGMENT

Pulmonary artery lies dorsolateral


to bronchus

AT HILUM

Pulmonary veins are ventromedial


to bronchus

NECK OF 1ST RIB

Related anteriorly from medial to


lateral side (SPIT)
Sympathetic chain
1st posterior intercostal vein
Intercostal artery
1st thoracic nerve

CAVERNOUS SINUS

Lateral wall above downwards


3rd nerve
4th nerve
5th nerve
Trigeminal ganglion
Within centre of sinus
ICA with venous and symphatetic
plexus
Abducent nerve

STRUCTERS PASSING BETWEEN Styloid process


EXTERNAL AND INTERNAL
Styloglossus
CAROTID ARTERY
Stylopharyngeus
IXth nerve
Pharyngeal branch of vagus nerve
Part of parotid gland
STRUCTERS DEEP TO POSTERIOR From above downwards (NiSHA)
BORDER OF HYOGLOSSUS
IX th nerve
Stylohyoid ligament
Lingual artery
COSTAL GROOVE OF RIBS

From above downwards(VAN)


Posterior intercostal vein
Posterior intercostal artery

Intercostal nerve
AT COMMON CAROTID ARTERY Internal carotid artery is
BIFURCATION
posterolateral to external carotid
artery
IN FRONT OF ALA OF SACRUM

(SLIO)
Symphatetic trunk
Lumbosacral trunk
Iliolumbar artery
Obturator nerve

HILUM OF KIDNEY

Anterior to posterior(VAP)
Vein
Artery
Pelvis

STRUCTURES PIERCING
CLAVIPECTORAL FASCIA

(TALC)
Thoracoacromial vein
Lateral pectoral nerve
Cephalic vein

AT PORTA HEPATIS

Anterior to posterior (DAV)


CBD
Hepatic artery
Portal vein

IN RADIAL/SPIRAL GROOVE OF
HUMERUS

Radial nerve
Profunda brachial artery

INBETWEEN SUPERIOR
CONSTRICTOR AND BASE OF
SKULL

Sinus of Morgagni
Auditory tube
Levator palatine MUSCLEnerve

INBETWEEN SUPERIOR AND


MIDDLE CONSTRICTOR

Stylopharyngeus muscle
Glossopharyngeal nerve

Ascending palatine artery


INBETWEEN MIDDLE AND
INFERIOR CONSTRICTOR

Internal laryngeal nerve


Superior laryngeal vessels

INBETWEEN INFERIOR
Recurrent laryngeal nerve
CONSTRICTOR AND ESOPHAGUS
Inferior laryngeal vessels
Greater sciatic foramen (structures Exp. Pyriformis
passing through)
Exp. Superior and inferior gluteal
nerve and vessels
Exp. Sciatic nerve

Exp. Pudendal nerve

Exp. Internal pudendal vessels

Exp. Posterior cutaneous nerve of


thigh
Exp. Nerve to quadratus
femoris
Exp. Nerve to obturator
internus.
Lesser sciatic foramen (structures
passing through)

Exp. Tendon of obturator internus


Exp. Nerve to obturator internus
Exp. Pudendal nerve
Exp. Internal pudendal vessels
Exp. Origin of gemelli

(Q.8)

The derivative of the second pharyngeal arch is:


(a)

Eustachian tube

(b)

External auditory meatus

(c)

Tensor tympani

(d)

Stylohyoid muscle

Your Response :
Correct Answer :

Exp: Stylohyoid muscle Reference: Langmans Medical Embryology (10th edition)


The stylohyoid is derived from the second pharyngeal arch, which also gives rise to the
muscles of facial expression, the stapedius, the posterior belly of the digastric muscle,
Reichert's cartilage, and the facial nerve.
The Eustachian tubes are derived from the first pharyngeal pouch, which also gives rise
to the middle ear cavity and the inner epithelial lining of the tympanic membrane.
The external auditory meatus (External auditory meatus is derived from the first
pharyngeal cleft, which also gives rise to the outer epithelial lining of the tympanic
membrane.
The palatine tonsil is derived from the epithelial lining of the second pharyngeal pouch.
The tensor tympani is derived from the first pharyngeal arch, which also gives rise to the
muscles of mastication, the anterior belly of the digastric muscle, the mylohyoideus, the
tensor veli palantini, the maxillary and mandibular bones, and the maxillary and
mandibular divisions of the trigeminal nerve.
Pharyngeal Arch Artery
Arch

Cranial Nerve

Skeletal elements

Maxillary and
mandibular
division of
trigemenial (V)

Derived from arch


cartilages
(originating from
neural crest):

Terminal
Branch of
maxillary
artery

Muscles

Muscles of
mastication
(temporalis,
masseter, and
pterygoids),
From
mylohyoid,
maxillary cartilages: anterior belly of
Alispenoid, digastric, tensor
tympani, tensor
incus
veli palatini
(originate from
From
cranial
mandibular:
somitomere 4)
Mackels
cartilage, malleus
Upper portion of
external ear
(auricle) is derived
from dorsal aspect
of 1st pharyngeal

arch.
Derived by direct
ossification from
arch dermal
mesenchyme:
Maxilla,
zygomatic,
squamous portion of
temporal bone,
mandible
2

Stapedius
Facial nerve (VII) Stapes, styloid
artery
process, stylohyoid
(embryologic)
ligament, lesser
and
horns and upper
cortiotympanic
rim of hyoid
artery (adult)
(derived from the
second arch
cartilage; originate
from neural crest).

Muscles of facial
expression
(orbicularis
oculi,
orbicularis oris,
auricularis,
platysma,
frontoooccipitalis,
buccinator),
Lower portion of
posterior belly
external ear
of digastric,
(auricle) is derived stylohyoid,
from 2nd pharyngeal stapedius
arch.
(originate from
cranial
somitomere 6)

Common
Glossopharyngeal Lower rim and
carotid artery, (IX)
greater horn of
most of
hyoid (derived from
internal
the third arch
carotid
cartilage; originate
from neural crest
cells)

Left: Arch of
aorta;
Right: Right
subclavian
artery;
Original
sprouts of

Sytlopharyngeus
(originate from
cranial
somitomere 7)

Superior
Laryngeal cartilages Constrictors of
laryngeal branch (Derived from the pharynx,
of vagus (X)
4th arch cartilage, cricothyroid,
originate from
levator veli
lateral plate
palatine
mesoderm)
(originate from
occipital somites
2-4)

pulmonary
arteries
6

Ductus
arteriosus;
roots of
definitive
pulmonary
arteries

Recurrent
Laryngeal cartilages Intrinsic
laryngeal branch (derived from the muscles of
of vagus (X)
6th-arch cartilage; larynx
originate from
(originate from
lateral plate
occipital somites
mesoderm)
1 and 2)

Derivates of pharyngeal pouches:


First pouch

Auditory tube, which comes in contact


with epithelial line of first pharyngeal
cleft, where future external acoustic
meatus will form.
Distal portion will form tympanic
cavity (lining will become eardrum)
Proximal portion will become auditory
tube

Second pouch

Forms buds that penetrate surrounding


mesenchyme, which together form the
palatine tonsils

Third pouch

Forms thymus and inferior parathyroid


Glands

Fourth pouch

Forms superior parathyroid glands

Fifth pouch

Forms utlimobranchial body

Derivatives of pharyngeal clefts/grooves:


Initially, four clefts exist; however, only one gives rise to a definite structure in
adults.
1st pharyngeal cleft

Penetrates underlying mesenchyme and


forms EAM. The bottom of EAM forms
lateral aspect of tympanic cavity.

2nd pharyngeal cleft

Undergoes active proliferation and


overlaps remaining clefts. It merges with

ectoderm of lower neck such that the


remaining clefts lose contact with
outside. Temporarily, the clefts form an
ectodermally lined cavity, the cervical
sinus, but this disappears during
development.
(Q.9)

The diagram represents the path traversed in the neck by the developing thyroid
gland embryologically. Which point along this path is the commonest site of a Thyroglossal cyst

(a)

Point I

(b)

Point II.

(c)

Point III

(d)

Point IV

Your Response :
Correct Answer :
Exp:

c
Point III.
REF : CLINICAL ANATOMY BY REGIONS, RICHARD SNELL, 8th
Ed, Chap 11
Embryology of the Thyroid Gland
The thyroid gland begins to develop during the third week as an
entodermal thickening in the midline of the floor of the pharynx between
the tuberculum impar and the copula. Later, this thickening becomes a
diverticulum that grows inferiorly into the underlying mesenchyme and is
called the thyroglossal duct. As development continues, the duct elongates,
and its distal end becomes bilobed. Soon, the duct becomes a solid cord of

cells, and as a result of epithelial proliferation, the bilobed terminal


swellings expand to form the thyroid gland.
The thyroid gland now migrates inferiorly in the neck and passes either
anterior to, posterior to, or through the developing body of the hyoid bone.
By the seventh week, it reaches its final position in relation to the larynx
and trachea. Meanwhile, the solid cord connecting the thyroid gland to the
tongue fragments and disappears. The site of origin of the thyroglossal duct
on the tongue remains as a pit called the foramen cecum. The thyroid gland
may now be divided into a small median isthmus and two large lateral
lobes.
In the earliest stages, the thyroid gland consists of a solid mass of cells.
Later, as a result of invasion by surrounding vascular mesenchymal tissue,
the mass becomes broken up into plates and cords and finally into small
clusters of cells. By the third month, colloid starts to accumulate in the
center of each cluster so that follicles are formed. The fibrous capsule and
connective tissue develop from the surrounding mesenchyme.
The ultimobranchial bodies (from the fifth pharyngeal pouch) and neural
crest cells are believed to be incorporated into the thyroid gland, where
they form the parafollicular cells, which produce calcitonin.
Agenesis of the Thyroid
Failure of development of the thyroid gland may occur and is the
commonest cause of cretinism.
Incomplete Descent of the Thyroid
The descent of the thyroid may be arrested at any point between the base of
the tongue and the trachea. Lingual thyroid is the most common form of
incomplete descent. The mass of tissue found just beneath the foramen
cecum may be sufficiently large to obstruct swallowing in the infant.
Ectopic Thyroid Tissue
Ectopic thyroid tissue is occasionally found in the thorax in relation to the
trachea or bronchi or even the esophagus. It is assumed that this thyroid
tissue arises from entodermal cells displaced during the formation of the
laryngotracheal tube or from entodermal cells of the developing esophagus
Persistent Thyroglossal Duct
Conditions related to a persistence of the thyroglossal duct usually appear
in childhood, in adolescence, or in young adults.
Thyroglossal Cyst
Cysts may occur at any point along the thyroglossal tract They occur most
commonly in the region below the hyoid bone. Such a cyst occupies the

midline and develops as a result of persistence of a small amount of


epithelium that continues to secrete mucus. As the cyst enlarges, it is prone
to infection and so it should be removed surgically. Since remnants of the
duct often traverse the body of the hyoid bone, this may have to be excised
also to prevent recurrence.
Thyroglossal Sinus (Fistula)
Occasionally, a thyroglossal cyst ruptures spontaneously, producing a
sinus. Usually, this is a result of an infection of a cyst. All remnants of the
thyroglossal duct should be removed surgically.
(Q.10)

Untrue regarding the radiusfracture :


(a)

The radius is commonly fractured 2 cm above its lower end

(b)

The distal fragment is displaced upwards and backwards.

(c)

The radial styloid process comes to lie distal to the ulnar

(d)

Smiths fracture is reverse of Colles fracture

Your Response :
Correct Answer :
Exp:

c
The radial styloid process comes to lie distal to the ulnar
Ref. BDC vol I 3rd ed. 21
The radius is commonly fractured 2 cm above its lower end.
The distal fragment is displaced upwards and backwards.
The radial styloid process comes to lie proximal to the ulnar
Smiths fracture is reverse of Colles fracture, the distal segment being
palmer flexed rather than dorsi flexed.

(Q.11)

The shaft of radius ossifies from a primary centre which appears during the
__________ week of development.
(a)

8th

(b)

14th

(c)

24th

(d)

36th

Your Response :
Correct Answer :

a
8th Reference: Grays Anatomy, 39th edition

Exp:

Ref. BDC vol I 3rd ed. 21


Ossification of radius:
The shaft ossifies from a primary centre which appears during the 8th
week of development. The lower end ossifies from a secondary centre
which appears during the first year and fuses at 20 years; it is the growing
end of the bone. The upper end (head) ossifies from a secondary centre
which appears during the 4th year and fuses at 18 years.
Extended elbow, the tip of the horizontal line with the two humerus; and
in the flexed bony points from an equilateral locations are disturbed in
dislocation the ulna.
(Q.12)

Which of the followingis not attached to ulna?


(a)

Pronator teres

(b)

Flexor digitorum profundus

(c)

Biceps

(d)

Anconeus

Your Response :
Correct Answer :
Exp:

Biceps
Reference: Grays Anatomy, 39th edition
Ref. BDC vol I 3rd ed. 24
The ulnar head of the pronator teres arises from the medial margin of the coronoid
process.
The flexor digitorum profundus arises from: (a) the upper three-fourths of the
anterior and medial surfaces of the shaft; (b) the medial surfaces of the coronoid and
olecranon processes; and (c) the posterior border of the shaft through an aponeurosis
which also gives origin to the flexor carpi ulnaris and the extensor carpi ulnaris.
The pronator quadratus takes origin from the oblique ridge on the lower part of the
anterior surface.

The flexor carpi ulnaris (ulnar head) arises from the medial side of the olecranon
process and from the posterior border.
The extensor carpi ulnaris arises from the posterior border.
The anconeus is inserted into the lateral aspect of the olecranon process and the
upper one fourth of the posterior surface.
The lateral part of the posterior surface gives origin from above downwards to the
abductor pollicis longus, the extensor pollicis longus, and the extensor indices.
Muscles of the Arm
Muscle

Origin

Insertion

Nerve Supply

Nerve Action
Rootsa

Anterior Compartment Biceps brachii


Long head

Supraglenoid Tuberosity Musculocutaneous C5, 6


tubercle of of radius
nerve
scapula
and
bicipital
aponeurosis
into deep
fascia of
forearm

Short head

Coracoid process

Supinator
of forearm
and flexor
of elbow
joint; weak
flexor of
shoulder
joint

of scapula
Coracobrachialis Coracoid
process of
scapula

Brachialis

Medial
aspect of
shaft of
humerus

Musculocutaneous C5, 6, Flexes arm


nerve
7
and
alsoprocess
of weak
adductor

Front of
Coronoid Musculocutaneous C5, 6
lower half of process of nerve
humerus
ulna

Flexor of
elbow joint

Posterior Compartment Triceps


Long head

Infraglenoid tubercle of scapula

Lateral head

Upper half Olecranon Radial nerve


of posterior process of
surface of
ulna
shaft of

C6, 7, Extensor
of elbow
8
joint

humerus
Medial head

(Q.13)

Lower half of posterior surface of shaft of humerus

The adjoining diagram demonstrates the anatomical structure of skin. Which of the
following is not considered as an appendage of skin ?

(a)

1 = Sebaceous gland

(b)

2 = Hair follicle

(c)

3 = Erector pili muscle

(d)

4= Eccrine sweat gland

Your Response :
Correct Answer :
Exp:

c
3 = Erector pili muscle
REF : Clinical Anatomy by Regions, Richard Snell 8th Ed Chapter 1
Pg 6
The skin over joints always folds in the same place, the skin creases. At
these sites, the skin is thinner than elsewhere and is firmly tethered to
underlying structures by strong bands of fibrous tissue. The appendages
of the skin are the nails, hair follicles, sebaceous glands, and sweat
glands. The nails are keratinized plates on the dorsal surfaces of the tips
of the fingers and toes. The proximal edge of the plate is the root of the
nail . With the exception of the distal edge of the plate, the nail is
surrounded and overlapped by folds of skin known as nail folds. The
surface of skin covered by the nail is the nail bed . Hairs grow out of

follicles, which are invaginations of the epidermis into the dermis. The
follicles lie obliquely to the skin surface, and their expanded extremities,
called hair bulbs, penetrate to the deeper part of the dermis. Each hair
bulb is concave at its end, and the concavity is occupied by vascular
connective tissue called hair papilla. A band of smooth muscle, the
arrector pili, connects the undersurface of the follicle to the superficial
part of the dermis. The muscle is innervated by sympathetic nerve fibers,
and its contraction causes the hair to move into a more vertical position; it
also compresses the sebaceous gland and causes it to extrude some of its
secretion. The pull of the muscle also causes dimpling of the skin surface,
so-called gooseflesh. Hairs are distributed in various numbers over the
whole surface of the body, except on the lips, the palms of the hands, the
sides of the fingers, the glans penis and clitoris, the labia minora and the
internal surface of the labia majora, and the soles and sides of the feet and
the sides of the toes. Sebaceous glands pour their secretion, the sebum,
onto the shafts of the hairs as they pass up through the necks of the
follicles. They are situated on the sloping undersurface of the follicles and
lie within the dermis. Sebum is an oily material that helps preserve the
flexibility of the emerging hair. It also oils the surface epidermis around
the mouth of the follicle. Sweat glands are long, spiral, tubular glands
distributed over the surface of the body, except on the red margins of the
lips, the nail beds, and the glans penis and clitoris. These glands extend
through the full thickness of the dermis, and their extremities may lie in
the superficial fascia. The sweat glands are therefore the most deeply
penetrating structures of all the epidermal appendage.
(Q.14)

Largest commisure of the brain is ?


(a)

Anterior commisure

(b)

Posterior commisure

(c)

Corpus callosum

(d)

None of the above

Your Response :
Correct Answer :
Exp:

c
Corpus callosum
The -est list in Anatomy :
Largest and deepest cranial
fossa

Posterior

Thickest/strongest orbital wall Lateral

Thinnest/weakest

Medial

Farthest and deepest quadrant Antero-inferior


of tympanic membrane
Smallest and most numerous
lingual papillae

Filiform

Largest diploic vein

Occipital diploic vein

Largest terminal branch of


ophthalmic nerve

Frontal nerve

Smallest terminal branch of


ophthalmic nerve

Lacrimal nerve

Largest of the sympathetic


cardiac branches

Middle cervical cardiac branch

Largest scalenous muscle

S.medius

Smallest scalenous muscle

S.posterior

Thickest intervertebral disc

At lumbar region

Thinnest intervertebral disc

At superior thoracic region

Largest papillary muscle of rt.


Ventricle

Anterior papillary muscle

Rib with the largest obliquity

9th rib

Shortest, broadest and most


curved rib

1st rib

Largest rib

7th rib

Thickest and toughest of the


meninges

Duramater

Largest branch of vertebral


artery

PICA

Largest branch of ICA

MCA

Largest horn of lateral ventricle Inferior horn


Largest commissure of brain

Corpus callosum

Thickest part of corpus

Splenium

callosum
Thickest cutaneous nerve

Greater occipital nerve

Largest organnele in eukaryotes Endoplasmic reticulum


Male urethra : Shortest part

Membranous

Male urethra : Longest part

Penile

Male urethra : widest and most Prostatic


dilatable part
Male urethra : Narrowest and
least dilatable part

Urethral orifice

Strongest ligament of the body Ilio-femoral/Bigelow ligament


Largest peripheral
parasympathetic ganglion

Sphenopalatine ganglion

Largest bursa of the body

Subacromial bursa

Largest joint

Knee

Longest muscle

Sartorius

Largest branch of coeliac trunk Splenic artery

(Q.15)

Smallest bone and muscle

Stapes and stapedius

Largest sesamoid bone

Patella

Longest vein

GSV

True about Platysma?


(a)

The platysma is a thin, broad sheet of muscle deep to subcutaneous


plane.

(b)

It is inserted on the body of mandible and angle of mouth

(c)

The platysma is supplied by the greater auricular nerve.

(d)

The platysma may protect the internal jugular vein from external
pressure.

Your Response :
Correct Answer :

Exp:

It is inserted on the body of mandible and angle of mouth


Ref. BDC vol I 3rd ed. 39
Platysma
The platysma is a thin, broad sheet of subcutaneous muscle.
The fibres of the muscle arise from the deep fascia covering the pectoralis
major; run upwards and medially crossing the clavicle and the side of the
neck; and are inserted into the base of the mandible, and into skin over the
posterior and lower part of the face.
The platysma is supplied by the facial nerve.
When the angle of the mouth is pulled down, the muscle contracts and
wrinkles the skin of the neck.
The platysma may protect the external jugular vein (which underlies the
muscle) from external pressure.
Clinical Identification of the Platysma :
The Platysma can be seen as a thin sheet of muscle just beneath the skin
by having the patient clench his or her jaws firmly. The muscle extends
from the body of the mandible downward over the clavicle onto the
anterior chest wall.
Platysma Tone and Neck Incisions
In lacerations or surgical incisions in the neck it is very important that the
subcutaneous layer with the Platysma be carefully sutured, since the tone
of the Platysma can pull on the scar tissue, resulting in broad, unsightly
scars.
Platysma Innervation, Mouth Distortion, and Neck Incisions
The Platysma muscle is innervated by the cervical branch of the facial
nerve. This nerve emerges from the lower end of the parotid gland and
travels forward to the Platysma; it then sometimes crosses the lower
border of the mandible to supply the depressor anguli oris muscle. Skin
lacerations over the mandible or upper part of the neck may distort the
shape of the mouth.
Muscles of the Neck
Muscle

Origin

Insertion

Nerve
Supply

Action

Platysma

Deep fascia
over

Body of
mandible

Facial
nerve

Depresses
mandible

pectoralis
major and
deltoid
Sternocleidomastoid Manubrium
sterni and
medial third
of clavicle

and angle of cervical


mouth
branch

and angle
of mouth

Mastoid
process of
temporal
bone and
occipital
bone

Two
muscles
acting
together
extend
head and
flex neck;
one
muscle
rotates
head to
opposite
side

Spinal part
of
accessory
nerve and
C2 and 3

Digastric
Posterior belly

Mastoid
Intermediate Facial
process of
tendon is
nerve
temporal bone held to hyoid
by fascial
sling

Depresses
mandible
or elevates
hyoid
bone

Anterior belly

Body of
mandible

Stylohyoid

Styloid
process

Body of
hyoid bone

Facial
nerve

Elevates
hyoid
bone

Mylohyoid

Mylohyoid
line of body
of mandible

Body of
hyoid bone
and fibrous
raphe

Inferior
alveolar
nerve

Elevates
floor of
mouth and
hyoid
bone or
depresses
mandible

Geniohyoid

Inferior
Body of
mental spine hyoid bone
of mandible

First
cervical
nerve

Elevates
hyoid
bone or
depresses
mandible

Nerve to
mylohyoid

Sternohyoid

Manubrium
sterni and
clavicle

Body of
hyoid bone

Ansa
Depresses
cervicalis; hyoid
C1, 2, and bone
3

Sternothyroid

Manubrium
sterni

Oblique line
on lamina of
thyroid
cartilage

Ansa
Depresses
cervicalis; larynx
C1, 2, and
3

Thyrohyoid

Oblique line
on lamina of
thyroid
cartilage

Lower
border of
body of
hyoid bone

First
cervical
nerve

Inferior belly

Upper margin
of scapula
and
suprascapular
ligament

Intermediate
tendon is
held to
clavicle and
first rib by
fascial sling

Ansa
Depresses
cervicalis; hyoid
C1, 2, and bone
3

Superior belly

Lower border
of body of
hyoid bone

Scalenus anterior

Transverse
First rib
processes of
third, fourth,
fifth, and
sixth cervical
vertebrae

C4, 5, and Elevates


6
first rib;
laterally
flexes and
rotates
cervical
part of
vertebral
column

Scalenus medius

Transverse
processes of
upper six
cervical
vertebrae

Anterior
rami of
cervical
nerves

Depresses
hyoid
bone or
elevates
larynx

Omohyoid

First rib

Elevates
first rib;
laterally
flexes and
rotates
cervical
part of
vertebral

column
Scalenus posterior

(Q.16)

Transverse
Second rib
processes of
lower cervical
vertebrae

Anterior
rami of
cervical
nerves

Elevates
second rib;
laterally
flexes and
rotates
cervical
part of
vertebral
column

Damage to the parasagittal region and falx cerebri will most likely result in which of
the following neurologic deficits?
(a)
(b)
(c)
(d)

Altered taste
Leg paralysis
Loss of facial sensation
Ptosis

Your Response :
Correct Answer :
Exp:

b
Leg paralysis
A meningioma of the parasagittal region and the falx cerebri would be
located at the top of the brain, near the midline. In this position, it could
compress the sensory or motor cortex supplying the lower extremities.
The falx cerebri is a fold of dura mater that projects between the cerebral
hemispheres in the longitudinal tissues. Its interior portions attach
anteriorly to the crista galli and posteriorly to the internal occipital crest.
Taste is supplied by cranial nerves VII, IX, and X. These nerves arise
from the brainstem.
Facial sensation is supplied by cranial nerve V, the nuclei of which are in
the brainstem. Furthermore, the area of the sensory cortex that subserves
the face is on the lateral aspect of the cortex and would not be affected by
a tumor in the parasagittal region.
Ptosis can be caused by a deficit in cranial nerve III, which arises from the
brainstem.
Unilateral deafness suggests damage to cranial nerve VIII, which arises

from the brainstem.


(Q.17)

Content of deltopectoral groove are all EXCEPT:


(a)

The cephalic vein

(b)

The axillary artery

(c)

The deltoid branch of the thoraco-acromial artery

(d)

Deltopectoral lymph nodes

Your Response :
Correct Answer :
Exp:

b
The axillary artery
Reference: Grays Anatomy, 39th edition
Ref. BDC vol I 3rd ed. 44
DEEP PECTORAL FASCIA
The deep fascia covering the pectoralis is called the pectoral fascia.
It is thin attached to the muscle by numerous spate between the fasciculi
of the muscle.
It is superiorly to the clavicle, and anteriorly to the sternum.
Superolatera1ly, it passes over the infraclavicular fossa and deltopectoral
groove to become continuous with, the fascia covering the deltoid.
Inferolaterally, the fascia curves round the inferolateral border of the
pectoralis major to become continuous with the axillary fascia.
Inferiorly continuous with the fascia over the thorax and rectus sheath.
The pectoral fascia is connected with clavipectoral fascia by a septum
passing deep along the deltopectoral groove.
The cephalic vein, the deltoid branch of the thoraco-acromial artery, and
the deltopectoral lymph nodes lie in deltopectoral groove, under the deep
fascia on the medial side of the septum.

(Q.18)

The narrowest part of ureter is at the:


(a)

Ureteropelvic junction

(b)

Iliac vessel crossing

(c)

Pelvic ureter

(d)

Ureterovesical junction

Your Response :
Correct Answer :
Exp:

d
Ureterovesical junction Reference: Keith L. Moore, Clinically oriented
Anatomy, 6th edition
Ureter runs retroperitonially after starting as continuation of renal pelvis
at PUJ ,down the anterior aspect of posas muscle, separated from it by
transversalis fascia. As it enters the pelvis it crosses over the anterior
aspect of common iliac artery bifurcation immediately in front of the SI
joint. It descends along lateral pelvic, just medial to the obturator in ternis
to the level of ischial spine ,from where it runs anteromedially until it
enters the superolateral angle of bladder base at UV junction .Vas crosses
over ureter separating it from bladder just before ureter enters the bladder
wall. The ureters run obliquely through bladder wall for around 2cm.

(Q.19)

Which of the following forms from paraxial mesoderm?


(a)

A.drenal cortex

(b)

A.drenal medulla

(c)

Humerus

(d)

Biceps brachii

Your Response :
Correct Answer :
Exp:

d
Biceps brachii
The muscles of the extremities form from the somites that are derived
from paraxial mesoderm; the bones, tendons, and connective tissue of the
extremities are derived from somatopleuric mesoderm. The intermediate
mesoderm is the origin of the urogenital systems and the adrenal cortex.
The adrenal medulla forms from the neural crest. The humerus (answer c)
forms from somatopleuric mesoderm, but the muscles attached to it are of
somite origin. The masseter is a muscle of mastication formed from the
first branchial arch and innervated by branchial visceral efferent (special
visceral efferent) fibers from the nucleus ambiguus compared with the
general somatic efferent innervation of the biceps and other muscles, not
of branchial arch origin.

(Q.20)

The cerebral cortex forms from which of the following?


(a)

Telencephalon

(b)

Myelencephalon

(c)

Metencephalon

(d)

Mesencephalon

Your Response :
Correct Answer :
Exp:

(Q.21)

a
Telencephalon Reference: Langmans Medical Embryology (10th
edition)
Primary Brain Vesicle Secondary Brain
Vesicle

Adult Brain
Derivative

Prosencephalon
(Forebrain)

Telencephalon

Cerebral cortex, corpus


striatum

Diencephalon

Hypothalamus,
thalamus

Mesencephalon
(Midbrain)

Mesencephalon

Superior & inferior


colliculi

Rhombencephalon
(Hindbrain)

Metencephalon

Pons and cerebellum

Metencephalon

Medulla

A sharp instrument passing through the superior orbital fissure would most likely
sever which of the following structures?
(a)

Abducens nerve

(b)

Facial nerve

(c)

Mandibular nerve

(d)

Middle meningeal artery

Your Response :
Correct Answer :
Exp:

a
Reference: Keith L. Moore, Clinically oriented Anatomy, 6th edition

A good way to remember what passes through the superior oribital fissure
is that everything that innervates the eye, other than the optic nerve,
passes through this fissure. This incudes the oculomotor nerve (CN III),
the trochlear nerve (CN IV), the ophthalmic nerve (V1), and the abducens
nerve (CN VI).
The facial nerve (CN VII; ) passes through the internal auditory meatus.
The mandibular nerve (V3; ) passes through the foramen ovale.
The maxillary nerve (V2; ) passes through the foramen rotundum.
The middle meningeal artery passes through the foramen spinosum.
Openings Into the Orbital Cavity
OPENING

ANATOMICAL DETAILS

Orbital opening

Lies anteriorly About one-sixth of


the eye is exposed; the remainder is
protected by the walls of the orbit.

Supraorbital notch /Foramen

The supraorbital notch is situated on


the superior orbital margin. It
transmits the supraorbital nerve and
blood vessels.

Infraorbital groove and canal

Situated on the floor of the orbit in


the orbital plate of the maxilla they
transmit the infraorbital nerve (a
continuation of the maxillary nerve)
and blood vessels.

Nasolacrimal canal

Located anteriorly on the medial


wall; it communicates with the
inferior meatus of the nose. It
transmits the nasolacrimal duct.

Inferior orbital fissure

Located posteriorly between the


maxilla and the greater wing of the
sphenoid; it communicates with the
pterygopalatine fossa. It transmits the
maxillary nerve and its zygomatic
branch, the inferior ophthalmic vein,
and sympathetic nerves.

Superior orbital fissure

Located posteriorly between the


greater and lesser wings of the
sphenoid it communicates with the
middle cranial fossa. It transmits the

lacrimal nerve, the frontal nerve, the


trochlear nerve, the oculomotor nerve
(upper and lower divisions), the
abducent nerve, the nasociliary
nerve, and the superior ophthalmic
vein.
Optic canal

(Q.22)

Located posteriorly in the lesser wing


of the sphenoid it communicates with
the middle cranial fossa. It transmits
the optic nerve and the ophthalmic
artery.

The structure responsible for the linkage of the intermediate filament network of
cells to the basal lamina is which of the following?
(a)

Macula adherens

(b)

Zonula adherens

(c)

Hemidesmosomes

(d)

Zonula occludens

Your Response :
Correct Answer :
Exp:

c
Hemidesmosomes Reference: Junqueira's Basic Histology: Text and
Atlas, 11th Edition
The hemidesmosome interacts with the extracellular matrix molecules
within the basal lamina through intermediate filament proteins. The
hemidesmosomes combined with the desmosomes act to distribute tensile
forces through the epithelial sheet and the supporting connective tissues.
Tight junctions characteristically surround the apical margins of the cells
in epithelia such as the intestinal mucosa, the walls of the renal tubules,
and the choroid plexus. They are also important to endothelial barrier
function. They are made up of ridgeshalf from one cell and half from
the otherwhich adhere so strongly at cell junctions that they almost
obliterate the space between the cells. There are three main families of
transmembrane proteins that contribute to tight junctions: occludin,
junctional adhesion molecules (JAMs), and claudins; and several more
proteins that interact from the cytosolic side. Tight junctions permit the
passage of some ions and solute in between adjacent cells (paracellular
pathway) and the degree of this "leakiness" varies, depending in part on
the protein makeup of the tight junction. Extracellular fluxes of ions and

solute across epithelia at these junctions are a significant part of overall


ion and solute flux. In addition, tight junctions prevent the movement of
proteins in the plane of the membrane, helping to maintain the different
distribution of transporters and channels in the apical and basolateral cell
membranes that make transport across epithelia possible.
In epithelial cells, each zonula adherens is usually a continuous structure
on the basal side of the zonula occludens, and it is a major site of
attachment for intracellular microfilaments. It contains cadherins.
Desmosomes are patches characterized by apposed thickenings of the
membranes of two adjacent cells. Attached to the thickened area in each
cell are intermediate filaments, some running parallel to the membrane
and others radiating away from it. Between the two membrane
thickenings the intercellular space contains filamentous material that
includes cadherins and the extracellular portions of several other
transmembrane proteins.
Hemidesmosomes look like half-desmosomes that attach cells to the
underlying basal lamina and are connected intracellularly to intermediate
filaments. However, they contain integrins rather than cadherins. Focal
adhesions also attach cells to their basal laminas. As noted previously,
they are labile structures associated with actin filaments inside the cell,
and they play an important role in cell movement.
(Q.23)

The extracellular matrix and the cytoskeleton communicate across the cell
membrane through which of the following?
(a)

Proteoglycans

(b)

Integrins

(c)

Cadherins

(d)

Intermediate filaments

Your Response :
Correct Answer :
Exp:

b
Integrins Reference: Junqueira's Basic Histology: Text and Atlas, 11th
Edition
The integrins are transmembrane heterodimers (integral membrane
proteins) that act as membrane receptors for extracellular matrix
components. The best examples are the fibronectin receptor and the
laminin receptor. The receptor structure includes an intra cytosolic portion
that binds to the actin cytoskeleton through the attachment proteins talin
or -actinin. The extracellular portion has specificity for extracellular

matrix molecules. Proteoglycans are located on the extracellular surface


of the plasma membrane and throughout the extracellular matrix. The
cadherins function as transmembrane glycoproteins involved in the
formation of parts of the intercellular junctional complexes. Cadherins are
components of the desmosome and zonula adherens. Intermediate
filaments and micro- tubules are found intracellularly and constitute the
cytoskeleton.
(Q.24)

Cribriform fascia is pierced by the following structures EXCEPT


(a)

Long saphenous vein

(b)

Superior epigastric vein

(c)

Superficial Epigastric artery

(d)

Superficial external pudendal artery

Your Response :
Correct Answer :
Exp:

b
Superior epigastric vein
Reference: Grays Anatomy, 39th edition
(Ref. BDC vol.2 4th ed. Pg. 50)
The Saphenous opening is an oval opening in the superomedial part of the
fascia lata closed by the cribriform fascia, which covers the opening.
The Saphenous opening is 4 cm below and lateral to pubic tubercle.
It is about Exp.5 cm long and 2 cm broad with its long axis directed
downwards and laterally.
Above, lateral and below sharp border called falciform margin.
Covered by a thin perforated part of the superficial fascia called the
cribriform fascia (pierced by the great saphenous vein, the 3 superficial
branches of the femoral A., and lymphatics).
It transmits the great saphenous vein and other smaller vessels (like
superficial epigastric artery and superficial external pudendal artery), as
well as the femoral branch of the genitofemoral nerve.
The cribriform fascia, which is pierced by the structures passing through
the opening, closes the aperture and must be removed to expose it.

(Q.25)

Appendix of testis is a remnant of?

(a)

Mesonephric duct

(b)

Paramesonephric duct

(c)

Metanephros

(d)

Urogenital Sinus

Your Response :
Correct Answer :
Exp:

b
Paramesonephric duct
Reference: Grays Anatomy, 39th edition
(Ref. Netter's Atlas of human embryology pg. 173)
The APPENDIX TESTIS (OR HYDATID OF MORGAGNI) is a remnant
of the cranial part of the paramesonephric or Mllerian duct, present on
the upper pole of the testis and attached to the tunica vaginalis. It is
present about 90% of the time.
Clinical significance
Although it has no physiological function, it can be medically significant
in that it can, rarely, undergo torsion (i.e. become twisted), causing acute
one-sided testicular pain and may require surgical excision to achieve
relief. 1/3 of patients present with a palpable "blue dot" discoloration on
the scrotum.
This is nearly diagnostic of this condition. Although if clinical suspicion
is high for testicular torsion, a surgical exploration of the scrotum is
warranted.
Occasionally a torsion of the hydatid of Morgagni can produce symptoms
mimicking those created by a testicular torsion; a torsion of the hydatid,
however, does not lead to any impairment of testicular function.
GARTNER'S DUCT is a remnant of the mesonephric duct in the female.
The paired mesonephric ducts in the male, in contrast, go on to form the
paired epididymis, ductus deferens, ejaculatory duct and seminal vesicle.

(Q.26)

Danger area of face is so called because:


(a)

Leads to cavernous sinus thrombosis following infection

(b)

This area is liable to formations of tumors

(c)

This area is connected directly to ear

(d)

This region is having poor arterial supply.

Your Response :
Correct Answer :
Exp:

a
Leads to cavernous sinus thrombosis following infection
vol.3 4th ed. Pg. 59)

(Ref. BDC

The facial vein communicates with the cavernous sinus through the deep
connections of the facial vein, which include:
A. Communications between the supraorbital and superior ophthalmic
veins
B. Connection with pterygoid plexus through the deep facial vein
Infections from the face can spread in retrograde direction and cause
thrombosis of cavernous sinus. This is specially likely to occur in the
presence of infection in the upper lip and the lower part of nose.
This area is therefore called the dangerous area of face.
(Q.27)

The superior ophthalmic vein directly communicates with which of the following
dural venous sinuses?
(a)

Cavernous sinus

(b)

Occipital sinus

(c)

Sigmoid sinus

(d)

Superior petrosal sinus

Your Response :
Correct Answer :
Exp:

a
Cavernous sinus Reference: Keith L. Moore, Clinically oriented
Anatomy, 6th edition
The anterior continuation of the cavernous sinus, the superior ophthalmic
vein, passes through the superior orbital fissure to enter the orbit. Veins of
the face communicate with the superior ophthalmic vein. Because of the
absence of valves in emissary veins, venous flow may occur in either
direction. Cutaneous infections may be carried into the cavernous sinus
and result in a cavernous sinus infection, which may lead to an infected
cavernous sinus thrombosis. The cavernous sinus is lateral to the pituitary

gland and contains portions of cranial nerves III, IV, V1, V2, and VI, and
the internal carotid artery.
The occipital sinus is at the base of the falx cerebelli in the posterior
cranial fossa. It drains into the confluence of sinuses.
The sigmoid sinus is the anterior continuation of the transverse sinus in
the middle cranial fossa. The sigmoid sinus passes through the jugular
foramen and drains into the internal jugular vein.
The superior petrosal sinus is at the apex of the petrous portion of the
temporal bone and is a posterior continuation of the cavernous sinus. The
superior petrosal sinus connects the cavernous sinus with the sigmoid
sinus.
The straight sinus is at the intersection of the falx cerebri and the falx
cerebelli in the posterior cranial fossa. The straight sinus connects the
inferior sagittal sinus with the confluence of sinuses.
(Q.28)

Lymphatic drainage of glans penis?


(a)

Deep inguinal lymph nodes

(b)

Superficial Inguinal lymph nodes

(c)

Internal Iliac lymph nodes

(d)

External Iliac lymph nodes

Your Response :
Correct Answer :
Exp:

a
Deep inguinal lymph node (Ref. BD Chaurasia, Anatomy, 2nd vol.,
3rd ed., 44)
Femoral sheath is funnel shaped asymmetrical sleeve of fascia enclosing
34 cm of femoral vessels.
Its contents are:
Lateral (arterial) compartmentfemoral artery and femoral branch of
genitofemoral nerve
Intermediate (venous) compartmentfemoral vein, and
Medial compartment (femoral canal)which contain lymph node of
Cloquet or Rosenmller, lymphatics and areolar tissue. This node drains
glans in male and clitoris in female.

- Relation from lateral to medial - NAVEL.


(Q.29)

Eversion and inversion of foot occurs mainly at:


(a)

Talocalcanean joint & Talonavicular joint

(b)

Talocalcaneonavicular joint & Talocalcanean joint

(c)

Calcaneocuboid joint & Talonavicular joint

(d)

Talonavicular joint & Calcaneocuboid joint

Your Response :
Correct Answer :
Exp:

b
Talocalcaneonavicular joint & Talocalcanean joint
(Ref. BD Chaurasia, Anatomy, Vol 2, 2nd ed., 155)
INVERSION AND EVERSION OF FOOT
Inversion is much more free than eversion.
Joints taking part:
Main:

1 Subtalar (Talocalcanean)
2 Talocalcaneonavicular

Accessory:

1 Calcaneocuboid

2 Talonavicular
Movement
Inversion

Principle muscles
Tibialis anterior

Tibialis posterior

Flexor hallucis longus

Flexor digitorum longus


Eversion

Accessory muscles

Peroneus longus

Peroneus brevis
peroneus tertius
Inversion is limited by:
Tension of peronei
Tension of cervical ligament

Eversion is limited by:


Tension of tibialis anterior
Tension of tibialis posterior
Tension of deltoid ligament
(Q.30)

Which structure does not pass through greater sciatic foramen?


(a)

Superior gluteal nerve and vessels

(b)

Pudendal nerve

(c)

Accessory obturator vessels

(d)

Inferior gluteal nerve

Your Response :
Correct Answer :
Exp:

c
Accessory obturator vessels
Reference: Grays Anatomy, 39th edition
(Ref. BD Chaurasia, Anatomy, Vol 2, 2nd ed., 48)
The Main Structures Passing Through Greater Sciatic Notch:
1 Superior gluteal nerve
2 Inferior gluteal nerve, and
3 Pudendal nerve (S2, S3, S4).

Greater sciatic foramen (structures passing through)


foramen (structures passing through)
1 Pyriformis

1 Tendon of obturator internus

2 Superior and inferior gluteal nerve and vessels


obturator internus
3 Sciatic nerve
4 Pudendal nerve
5 Internal pudendal vessels

Lesser sciatic

2 Nerve to

3 Pudendal nerve
4 Internal pudendal vessels
5 Origin of gemelli

6 Posterior cutaneous nerve of thigh


7 Nerve to quadratus femoris
8 Nerve to obturator internus.
(Q.31)

Loop of Galen is for:


(a)

Laryngeal nerve supply

(b)

Lymphatic drainage of larynx

(c)

Blood supply of larynx

(d)

Tendinous insertion of laryngeal muscles

Your Response :
Correct Answer :
Exp:

a
Laryngeal nerve supply
(Ref. The anatomy and physiology of the mammalian larynx By
Donald Frederick pg. 151)
The internal branch of the superior laryngeal nerve is joined to the
recurrent laryngeal nerve by a communication called the ramus
communicans of Loop of Galen.
The Galen anastomosis (also called the ramus anastomoticus or Ansa of
Galen) is a connection between the RLN and the internal branch of the
SLN (see the image above). Generally, the posterior branch of the RLN
contributes to the anastomosis; however, the anterior branch can also
contribute to the anastomosis. Traditionally, the Galen anastomosis has
been described to provide purely sensory and autonomic innervation.
More recent studies have shown that the anastomosis may also contain
motor fibers.Despite being initially described as a single nerve, it may
exist as a single trunk, several branches, or a plexus.
The "human communicating nerve" is an anastomosis between the
external branch of the SLN and the distal RLN. Approximately 70% of
human larynges have this anastomosis. The human communicating nerve
may contain both sensory innervation to the larynx and motor innervation
to the thyroarytenoid muscle.

(Q.32)

Space of Gillete is?


(a)

Space Posterior to pharynx

(b)

Space Posterior to larynx

(c)

Space anterior to larynx

(d)

Space anterior to pharynx

Your Response :
Correct Answer :
Exp:

a
Space Posterior to pharynx
(Ref. Cunningham's Textbook of Anatomy 4th Ed. pg. 134)
Anatomically, the retropharyngeal space of Gillette is bound by the
pharynx anteriorly and the prevertebral cervical fascia posteriorly.
Retropharyngeal space: It lies behind the pharynx between the
buccopharyngeal fascia covering pharyngeal
constrictor muscles and the pre vertebral fascia. It extends from the base
of skull to the bifurcation of trachea. The
space is divided into two lateral compartments (spaces of Gillette) by a
fibrous raphe. Each lateral space contains retropharyngeal nodes which
usually disappear at 3-4 years of age. Parapharyngeal space communicates
with the retropharyngeal space. Infection of retropharyngeal space can
pass down behind the oesophagus into the
mediastinum .

(Q.33)

Oro- Pharyngeal isthmus is?


(a)

Path of nasopharynx communication with the oropharynx

(b)

Path of Laryngopharynx communication with the oropharynx

(c)

Path of nasopharynx communication with the Laryngopharynx

(d)

Remanant of thyroid isthmus

Your Response :
Correct Answer :
Exp:

a
Path of nasopharynx communication with the oropharynx
(Ref. Basic Human Anatomy - O'Rahilly, Mller, Carpenter &
Swenson chapter 53)
The nasopharynx, at least in its anterior part, may be regarded as the

posterior portion of the nasal cavity, with which it has a common function
as part of the respiratory system. The nasopharynx communicates with the
oropharynx through the pharyngeal isthmus, which is bounded by the soft
palate, the palatopharyngeal arches, and the posterior wall of the pharynx.
The isthmus is closed by muscular action during swallowing. The choanae
are the junction between nasopharynx and the nasal cavity proper.
(Q.34)

Which of the following is not lined by non-keratinized stratified squamous


epithelium?
(a)

Hypopharynx and laryngopharynx

(b)

Oesophagus

(c)

Cornea

(d)

Tympanic membrane

Your Response :
Correct Answer :
Exp:

d
Tympanic membrane
277, 291)

(Ref. IB Singh Histology 2nd ed. Pg. 66, 70,

EPITHELIAL LINING AT VARIOUS SITES IN THE BODY :


SITE

EPITHELIUM TYPE

Pleura,Pericardium, Peritoneum, Simple squamous epithelium


Endocardium
Throglossal cyst

Pseudostratified columnar and


Squamous epithelium

Alveoli in premature/preterms

Low cuboidal epithelium

Ethmoidal nasal polyp

Early stage : Ciliated columnar


epithelium
Late stage : Transitional and
squamous epithelium(Metaplastic)

Paranasal sinuses

Ciliated columnar epithelium

Nasolabial cysts

Pseudostratified columnar
epithelium

Bartholins duct

Multilayered columnar epithelium

Bartholins gland acini

Low columnar / Cuboidal single


layer

Endocervix / cervical canal

Ciliated High columnar epithelium

Ectocervix

Squamous epithelium

Endometrium

Columnar epithelium

Adult vagina

Stratified squamous epithelium

Newborn vagina

Transitional epithelium

Endosalpinx

Tall ciliated columnar epithelium

Chocolate cyst of ovary in


endometriosis

Columnar cells with tendency to


form papillae

Vocal cord and upper part of


vestibule of larynx

Nonkeratinized Stratified
squamous epithelium

Rest of the larynx

Cilitaed columnar epithelium

Bowmans capsule

Simple squamous epithelium

PCT

Simple cuboidal epithelium with


brush border

DCT

Simple cuboidal epithelium


without brush border

Thin Loop of Henle

Low cuboidal epithelium or


squamous epithelium

Thick LH

Cuboidal epithelium

Collecting duct

Simple cuboidal or columnar


epithelium

Proximal urethra

Transitional epithelium

Mid urethra

Pseudostratifid columnar
epithelium

Most distal part of urethra

Stratified squamous epithelium

Mouth , oral cavity and


esophagus

Nonkeratinized Stratified
squamous epithelium

Stomach

Simple columnar epithelium

Small intestine

Columnar epithelium with brush


border Microvilli

Large intestine

Simple columnar epithelium

Transition zone of anal canal

Stratified columnar epithelium

Anal canal /anal verge

Stratified squamous epithelium

Thyroid follicles

Cuboidal cells

Ovarian germinal epithelium

Cuboidal cells

Gall bladder

Columnar cells with brush border

Secondary ovarian follicles

Stratified cuboidal epithelium

Ducts of salivary glands

Stratified columnar epithelium

Sensory epithelium of olfactory


area

Pseudostratified columnar
epithelium

Trachea and large bronchi

Pseudostratified columnar
epithelium

Part of Auditory tube


Ductus deferens
(Q.35)

Which of the following is not supplied by obturator nerve?


(a)

Obturator externus

(b)

Obturator internus

(c)

Adductor magnus

(d)

Gracilis

Your Response :
Correct Answer :
Exp:

b
obturator internus
Reference: Grays Anatomy, 39th edition
(Ref. BDC vol. II, 3rd ed. 39, 52, 56, 279)
The lumbosacral plexus is formed by the anterior rami of spinal nerves
T12 through SExp. The innervation of the lower limb arises from
segments L2 through 5Exp.

The major nerves of the lower limb are the:


Femoral nerve-posterior divisions of L2 through L4
Obturator nerve-anterior divisions of L2 through L4
Tibial nerve-anterior divisions of L4 through S3
Common peroneal nerve-posterior divisions of L4 through SExp.
Obturator Nerve
The Obturator nerve arises from the lumbar plexus (L2, 3, and 4) and
emerges on the medial border of the psoas muscle within the abdomen. It
runs forward on the lateral wall of the pelvis to reach the upper part of the
Obturator foramen , where it divides into anterior and posterior divisions.
Branches
The anterior division passes downward in front of the Obturator externus
and the adductor brevis and behind the pectineus and adductor longus. It
gives muscular branches to the gracilis, adductor brevis, and adductor
longus, and occasionally to the pectineus. It gives articular branches to the
hip joint and terminates as a small nerve that supplies the femoral artery.
It contributes a variable branch to the subsartorial plexus and supplies the
skin on the medial side of the thigh.
The posterior division pierces the Obturator externus and passes
downward behind the adductor brevis and in front of the adductor
magnus. It terminates by descending through the opening in the adductor
magnus to supply the knee joint. It gives muscular branches to the
Obturator externus, to the adductor part of the adductor magnus, and
occasionally to the adductor brevis.
Muscles of the Medial Fascial Compartment of the Thigh
Muscle

Origin

Insertion

Gracilis

Inferior
ramus of
pubis,
ramus of
ischium

Upper part Obturator


of shaft of
nerve
tibia on
medial
surface

Adductor Body of
longus
pubis,
medial to
pubic

Nerve Supply Nerve Action


Rootsa
L2, 3

Adducts
thigh at hip
joint; flexes
leg at knee
joint

Posterior Obturator nerve L2, 3,


surface of
4
shaft of
femur
(linea

Adducts
thigh at hip
joint and
assists in
lateral

tubercle

aspera)

rotation

Adductor Inferior
brevis
ramus of
pubis

Posterior Obturator nerve L2, 3,


surface of
4
shaft of
femur
(linea
aspera)

Adducts
thigh at hip
joint and
assists in
lateral
rotation

Adductor Inferior
magnus ramus of
pubis,
ramus of
ischium,
ischial
tuberosity

Posterior
surface of
shaft of
femur,
adductor
tubercle of
femur

Adducts
thigh at hip
joint and
assists in
lateral
rotation;
hamstring
portion
extends
thigh at hip
joint

Obturator Outer
externus surface of
obturator
membrane
and pubic
and ischial
rami

Medial
Obturator nerve L3, 4
surface of
greater
trochanter

Adductor
L2, 3,
portion:
4
obturator
nerveHamstring
portion: sciatic
nerve

Laterally
rotates thigh
at hip joint

Pudendal Nerve and the Nerve to the Obturator Internus


Branches of the sacral plexus, the pudendal nerve, and nerve to the
Obturator internus leave the pelvis through the lower part of the greater
sciatic foramen, below the piriformis. They cross the ischial spine with
the internal pudendal artery and immediately re-enter the pelvis through
the lesser sciatic foramen; they then lie in the ischiorectal fossa. The
pudendal nerve supplies structures in the perineum. The nerve to the
Obturator internus supplies the Obturator internus muscle on its pelvic
surface.
(Q.36)

Which of the following is true about the course of azygos vein?


(a)

A.nterior to superior vena cava

(b)

Passes through the esophageal hiatus in diaphragm

(c)

Passess through middle mediastinum

(d)

Posterior to oesophagus

Your Response :
Correct Answer :
Exp:

d
Posterior to oesophagus(Ref. BDC 4th ed. Vol. I 213 and Vol. II
274)
THE AZYGOS SYSTEM OF VEINS
The Azygos Vein
The azygos vein is the main channel of all venous return from below the
diaphragm to the right atrium except for hepatic venous return which will
go directly into the atrium.
It connects the superior and inferior venae cavae, either directly by joining
the IVC or indirectly by the hemiazygos and accessory hemiazygos veins.
The azygos vein drains blood from the posterior walls of the thorax and
abdomen.
The azygos vein is normally formed in the right posterior mediastinum
from the right intercostal veins and it ascends to arch over the structures
of the right hilum and join the right posterior aspect of the superior vena
cava.
The azygos vein may arise from the posterior surface of IVC near the
renal veins, or from the right renal vein, or may be formed by the union of
right ascending lumbar vein and the right subcostal vein.
It enters the thorax either by passing through the aortic opening of the
diaphragm, or by piercing the right crus.
It ascends in the posterior mediastinum, passing close to the right sides of
the bodies of the inferior eight thoracic vertebrae (T4-T12).
It is covered anteriorly by the oesophagus as it passes posterior to the root
of the right lung.
It then arches over the superior aspect of this root to join the SVC.
In addition to the posterior intercostal veins, the azygos vein
communicates with the vertebral venous plexuses.
This vein also receives the mediastinal, oesophageal, and bronchial veins.

(Q.37)

In preparation for a procedure to remove the fingernail on an index finger, the


physician would most likely anesthetize a branch of the

(a)

Anterior interosseus nerve

(b)

Median nerve

(c)

Musculocutaneous nerve

(d)

Radial nerve

Your Response :
Correct Answer :
Exp:

b
Median nerve Reference: Keith L. Moore, Clinically oriented Anatomy,
6th edition
The median nerve supplies the surface of the lateral palm, the palmar
surface of the first three digits, and the distal dorsal surface of the index
and middle fingers (including the nail beds). Therefore, prior to
performing surgery in this area, it is essential to anesthetize a branch of
this nerve (possibly a proper digital branch) to eliminate pain sensation
around the nail bed of the index finger. The median nerve of the branchial
plexus distributes to the flexor muscles on the forearm (flexor carpi
radialis and palmaris longus), the pronators (p. quadratus and p. teres),
digital flexors, and skin over the lateral surface of the hand.
Neither the anterior interosseus nor the musculocutaneous nerves supplies
the hand. The anterior interosseous nerve supplies the flexor pollicis
longus, the lateral half of flexor digitorum profundus, and pronator
quadratus. The musculocutaneous nerve supplies the coracobrachialis,
biceps, and most of the brachialis muscle, then becomes the lateral
cutaneous nerve of the forearm.
The radial nerve supplies skin on the radial side of the dorsal surface of
the hand, but not the fingertips.
The ulnar nerve supplies the palmar and dorsal surfaces of the medial
hand, including the palmar and dorsal surfaces of the fourth and fifth
digits.

(Q.38)

Muscles of mastication are supplied by?


(a)

Facial nerve

(b)

Mandibular nerve

(c)

Maxillary nerve

(d)

Glossopharyngeal nerve

Your Response :
Correct Answer :
Exp:

b
Mandibular nerve
(Ref. Human Embryology by Inderbirsingh, 7th ed., p 112)
Branchial arches
First arch derivatives
1). Meckels
cartilage

Mandible, Malleus,
Incus, Sphenomandibular
ligament, Anterior ligament of malleus.

2). Muscles

Four muscles of mastication (temporalis, masseter,


lateral and medial pterygoids).
Tensor tympani, Tensor veli palatini,
Mylohyoid, Anterior belly of digastric.
(Hint4M, 2 T, M and A)

3). Nerve of first Mandibular nerve (post-trematic) and chorda tympani


arch
(pretrematic).
4). Artery of first Maxillary artery.
arch
- The muscles of mastication move the mandible during mastication and
speech.
- They are the masseter, the temporalis, the lateral pterygoid and the
medial pterygoid.
- They develop from the mesoderm of the first branchial arch, and the
supplied by the mandibular nerve which is the nerve of that arch.
For a detailed account of Pharyngeal arch derivatives please refer to
explanation of Q. No 8 above.
(Q.39)

False regarding Sympathetic nervous system ?


(a)

Adrenal medulla is essentially a sympathetic ganglion.

(b)

Noradrenaline is major transmitter

(c)

Arises from Dorso-lumbar region of cord

(d)

Sympathetic ganglia are located on the target organ

Your Response :
Correct Answer :
Exp:

d
Sympathetic ganglia are located on the target organ
ANS (Autonomic Nervous system) Parasympathetic and
sympathetic
Chemical transmission at autonomic junction: -(between pre and postganglionic neuron; and between the postganglion neurons and the
autonomic effectors) The principal transmitter agents involved are
Acetyl choline and Non-epinephrine, although Dopamine is also secreted
by interneurons in the sympathetic ganglia and GnRH is secreted by some
of the preganglionic neurons
On the basis of the chemical mediator, ANS is divided into: ICholinergic division (Ach transmitter) And II- Nor-adrenergic division
(Nor Ad transmitter)
Examples of cholinergic neurons:- (Ach transmitter)
All preganglionic neuron Q
Parasympathetic postganglionic neurons Q
Sympathetic post ganglion neurons which innervate sweat gland Q
Sympathetic neurons which ends in the blood vessels (= produce
vasodilation) Q
Remaining post ganglionic sympathetic neurons are Noradrenergic Q
The highest seat regulating ANS is in hypothalmus; posterior and lateral
nuclei and primarily sympathetic while anterior and medial nuclei are
primarily parasympathetic
ICA cells (intrinsic cardiac Adrenergic cells) Epinephrine (Adenergic)
arid norepinephrine.
The adrenal medulla is essentially a sympathetic ganglion in which the
post ganglion cells have lost their axons and secrete norepinephrine,
epinephrine, and some Dopamine directly into the blood stream.
Difference between sympathetic and paraysmpathetic division of ANS: Trait

Sympathetic

Parasympathetic

1 Origin Q

Dorso-lumbar (T1 to
L2 or L3)

Craniosacral (III, VII,


IX, X, S2-S4)

2 Ganglia Q

Away from the organ On or close to organ

3 Distribution Q

Wide

Limited to head, neck and


trunk

4 Post ganglionic fibers Long

Short

5 Pre: Postganglionic
fibers ratio Q

1:20 to 1:100

11 to 1:2 (except in
enteric plexus)

6 Transmitter Q

Nor adrenaline (major) Acetyl choline


Acetyl choline (Minor)

7 Function Q

(Q.40)

Tacking stress and


emergency

Assimilation of food
conservation of energy

Thyroid develops from:


(a)

Endoderm of thyroglossal duct

(b)

First arch

(c)

Second arch

(d)

Third arch

Your Response :
Correct Answer :
Exp:

a
Endoderm of thyroglossal duct

(Ref. Bailey and Love 25th ed. 255)

Embryology Of Thyroid Gland


The thyroid develops from a median endodermal thyroid diverticulum
which grows down infront of the neck fromt he floor of the primitive
pharynx, just caudal to the tuberculum impar. The lower end of the
diverticulum enlarges to form the gland.
The rest of the diverticulum remains narrow and is known as the
thyroglossal duct. Most of the duct soon disappears.
The position of the upper end is marked by the foramen caecum of the
tongue, and the lower end often persists as the pyramidal lobe.
The thyroid gland descends early in foetal life from the base of the tongue
towards its position in the lower neck with the isthmus lying over the
second and third tracheal ring.

At the time of its descent the hyoid bone has not been formed and the
track of the descent of the thyroid gland is variable passing in front,
through or behind the eventual position of the hyoid body.
Thyroglossal duct cysts represent a persistence of this track and may
therefore be found anywhere in or adjacent to the midline from the tongue
base to the thyroid isthmus.
Rarely, a thyroglossal cyst may be the only functioning thyroid tissue in
the body.
HEAD AND NECK : VISCERA/ORGAN DEVELOPMENT - A
COMPREHENSIVE ACCOUNT :
Tongue

Appears at 4th week.


Musculature derived from mesoderm of occipital
somites. Precursor muscles cells migrate to region of
tongue and are innervated by general sensory efferent
fibers of CN XII.
Mucosa derived from anterior endoderm lining
arches 1-4; accordingly, innervation depends on arch
derivation:
Mucosa of anterior 2/3 of tongue comes
from the first arch CN V
Mucosa of posterior 1/3 of tongue comes
from third and forth arch CN IX, X
Special taste of anterior 2/3 of tongue comes from
CN VII.
Special taste of posterior 1/3 of tongue comes from
CN X.
Tongue freed from floor of mouth by extensive
degeneration of underlying tissue. Midline frenulum
continues to anchor tongue to floor of mouth.

Thyroid Gland Develops as ingrowth of mucosal epithelium located


in the midline of the tongue (at foramen cecum). It
descends along front of pharyngeal gut, but remains
connected to tongue by thyrooglossal duct, which is
obliterated later in development. Thyroid gland
descends to a point just caudal to laryngeal cartilages.
Facial
a) medial nasal prominence forms midline of nose,
structures (general) philtrum and primary palate

b) lateral nasal prominence forms alae of nose


c) maxillary prominence forms cheek region and
lateral lip
d) clefts can form at inter-prominence fusion lines
Nose

At the time of anterior neural tube closure,


mesenchyme around forebrain, frontonasal
prominence (FNP), has smooth rounded extended
contour. Nasal placodes (thickening of surface
ectoderm to become peripheral neural tissue) develop
on frontolateral aspects of FNP. Mesenchyme swells
around nasal placode producing a medial and lateral
nasal prominence (nasomedial and nasolateral
processes). These nasal prominences form the nose.

Mouth

Stomadeum (primitive oral cavity) forms between


frontonasal prominence and first pharyngeal
arch. The first pharyngeal arch forms the dorsal
maxillary prominence and ventral mandibular
prominence. The maxillary prominence will merge
with medial nasal prominences, pushing them closer
to cause fusion. Fused medial nasal prominences
will form midline of nose and midline of upper lip
(philtrum) and primary palate (first 4 teeth).

Nasolacrimal
structures

Maxillary and lateral nasal prominences are separated


by deep furrow, the nasolacrimal groove. Ectoderm
in floor of groove forms epithelial cord, which
detaches from overlying ectoderm. The epithelial
cord canalizes to form the nasolacrimal duct. The
upper end of the duct widens to form the lacrimal
sac. After detachment of the cord, the maxillary and
lateral nasal prominences merge with each other,
resulting in the formation of a nasolacrimal duct that
runs from the medial corner of the eye to the inferior
meatus of the nasal cavity.
The maxillary prominences enlarge to form the
cheeks and maxillae.
The lateral nasal prominences form the alae of the
nose.

Secondary (hard)
palate

Main part of definitive palate formed by two palatine


shelves derived from intraoral bilateral extensions of
the maxillary prominences. These appear at the 6th
week. They are directed obliquely downward on

each side of the tongue; they move down when


mandible gets bigger.
At the seventh week, they ascend to attain a
horizontal position, then fuse to form the secondary
palate. At the time the palatine shelves fuse, the
nasal septum (an outgrowth of median tissue of the
frontonasal prominence) grows down and joins the
cephalic aspect of the newly formed palate
Anteriorly, shelves fuse with triangular primary
palate. The incisive foramen marks the midline
between the primary and secondary palate.
External Ear

The auricle is derived from 6 auricular hillocks


(mesenchymal proliferations) along the dorsal aspect
of arches 1 (top of ear) and 2 (bottom of ear). These
fuse to form the definitive auricle. At the mandible
grows, the ear is pushed upward and backward from
its initial horizontal position on the neck.
The EAM is derived from the 1st pharyngeal arch.
The eardrum (tympanic membrane) is composed of 3
layers of cells: 1) ectodermal epithelial lining of
bottom of EAM; 2) endodermal epithelium lining of
tympanic cavity; 3) intermediate layer of connective
tissue.
The eardrum is composed of multiple cell layers
because it represents the first pharyngeal membrane,
and thus lies at the junction of the first pharyngeal
pouch and cleft.

Middle Ear

The middle ear consists of an auditory tube (from the


1st pharyngeal pouch, along with tympanic cavity)
and the ossicles (from pharyngeal arches 1 and 2
cartilage).
The first arch cartilage forms the malleus and
incus. The tensor tympani (muscle of the malleus) is
derived from the fourth somitomere (associated with
the first arch) and is therefore innervated by CN V.
The second arch cartilage forms the stapes. The
stapedius (muscles of the stapes) is derived from the
sixth somitomere (associated with the second arch)
and is therefore innervated by CN VII.
The ossicles are initially embedded in mesenchyme,

but in the 8th month, the mesenchyme degenerates


and an endodermal epithelial lining of the tympanic
cavity envelops the ossicles and connects them to the
wall of the cavity in a mesentery-like fashion.
Inner Ear

The inner ear is derived thickening of surface


ectoderm on both sides of the hindbrain (otic
placodes). The placodes invaginate to form otic
vesicles (otocytes). The vesicles then divide into
ventral and dorsal components.
The ventral component forms the saccule and
cochlear duct.
The dorsal component forms the utricle and
semicircular canals and endolymphatic duct.

Cochlear Duct

Derived from an outgrowth of the saccule during the


6th week. The outgrowth penetrates the surrounding
mesenchyme in a spiral fashion. The surrounding
mesenchyme forms the cartilage and undergoes
vacuolization.
The scala vestibule and scale tympani form and
surround the cochlear duct. They are filled with
periplymp to receive mechanical vibrations of
ossicles. The mechanical stimuli activates sensory
(ciliary) cells in the cochlear duct.

Semicircular canals The utricle is initially three flattened outpocketings,


which lose the central core. From this three
semicircular canals are forms, each at 90 degree
angles from one another. Sensory cells arise in the
ampulla at one end of each canal, in the utricle and
saccule.
Eye

At week 4, two depressions are evident on each of


the forebrain hemispheres. As the anterior neural
fold closes, the optic pits elongate to form the optic
vesicles. The optic vesicles remain connected to the
forebrain by optic stalks.
The invagination of the optic vesicles forms a
bilayered optic cup. The bilayered cup becomes the
dual layered retina (neural and pigmented layer)
Surface ectoderm forms the lens placode, which
invaginates with the optic cup.

The optic stalk is deficient ventrally to contain


choroids fissure to allow blood vessels into the eye
(hyaloid artery). The artery feeds the growing lens,
but will its distal portion will eventually degenerate
such that the adult lens receives no hyaloid
vasculature.
At the 7th week, the choroids fissure closes and walls
fuse as the retinal nerve get bigger.
The anterior rim of the optic vesicles forms the retina
and iris. The iris is an outgrowth of the distal edge of
the retina.
Optic vesicles induces/maintains the development of
the lens vesicle, which forms the definitive
lens. Following separation of the lens vesicle from
the surface ectoderm, the cornea develops in the
anterior 1/5th of the eye.
The lens and retina are surrounded by mesenchyme
which forms a tough connective tissue, the sclera,
that is continuous with the dura mater around the
optic nerve.
Iridopupillary membrane forms to separate the
anterior and posterior chambers of the eye. The
membrane breaks down to allow for the pupil
Mesenchyme surrounding the forming eye forms
musculature (ciliary muscles and pupillary muscles
from somitomeres 1 and 2; innervated by CN III),
supportive connective tissue elements and
vasculature.
Eyelids

(Q.41)

Formed by an outgrowth of ectoderm that is fused at


its midline in the 2nd trimester, but later reopen.

Which is true about vaginal skin?


(a)

It has no glands

(b)

Lined by columnar epithelium

(c)

Is almost always dry

(d)

None of the above

Your Response :

Correct Answer :
Exp:

a
It has no glands

(Ref. BDC anatomy 4th ed. Vol. II. Pg. 177)

VAGINA
The vagina extends from the vestibule of the external genitalia to the
cervix.
It contains no glands.
The mucus lubricating it originates from the glands of the cervix and the
vestibular glands.
It is lined by a stratified squamous epithelium that is rich in glycogen.
Vaginal Changes Caused by Changes in Estrogen Levels
Estrogenic phase: During the estrogenic phase vaginal fluid has a lower
pH than during the rest of the cycle, resulting from the formation of lactic
acid by bacteria metabolizing glycogen.
Postestrogenic phase: The drop in estrogen levels induces a decrease in
glycogen levels, which in turn causes an increase in vaginal pH and, thus,
an increase in the likelihood of infection.
(Q.42)

The following muscles of eyelid are supplied by sympathetic innervation?


(a)

Orbicularis oculi

(b)

Mullers muscle

(c)

Levator palpebrae superioris

(d)

Fibrous layer

Your Response :
Correct Answer :
Exp:

b
Mullers muscle Reference: Grays Anatomy, 39th edition
The muscles of the eyelids are:
Levator palpebr superioris.
Orbicularis oculi.
Corrugator.
The Orbicularis oculi and Corrugator are supplied by the facial nerve.

Corrugator draws the eyebrow downward and medial ward, producing the
vertical wrinkles of the forehead. It is the frowning muscle, and may be
regarded as the principal muscle in the expression of suffering.
Muller's muscleThe superior palpebral muscle is a smooth muscle that acts an eyelid
elevator.
Origin- arises from the inferior or bulbar aspect of the levator palpebrae
behind the fornix.
Insertion-upper edge of the tarsal plate, between the levator and
conjunctiva.
Action- eyelid elevator
Size- 15-20 mm at its origin, 10 mm in vertical length, slightly sider at its
insertion
Relations-lies between the tendon of the levator and the conjunctiva in the
eyelid
Innervation- sympathetic fibers
(Q.43)

True regarding llio-inguinal nerve is


(a)

Arises from lumbar plexus

(b)

Branch of Dorsal root of L1

(c)

Pierces anterior wall of inguinal canal

(d)

Passes between external oblique & transverses abdominis muscles

Your Response :
Correct Answer :
Exp:

a
Arises from lumbar plexus

(Ref. BDC anatomy Vol. II 3rd ed. 278)

The ilioinguinal nerve (L1) has the same course as the iliohypogastric
nerve, but on a slightly lower level.
LUMBAR PLEXUS
The lumbar plexus lies in the posterior part of the substance of the
PSOAS MAJOR muscle.
It is formed by the ventral rami of the upper 4 lumbar nerves.

It branches include:
Lateral cutaneous nerve of the thigh (L2, L3 dorsal divisions)
Lumbosacral trunk (L4, 5; ventral trunk)
Obturator nerve (L2, 3, 4; dorsal divisions)
Femoral nerve (L2, 3, 4; dorsal divisions)
Ilioinguinal nerve (L1)
Iliohypogastric nerve (L1)
Genitofemoral nerve (L1, L2 ventral divisions
(Mnemonic = FIIG LLO)
Theiliohypogastric nerveis the superior branch of the anterior ramus
ofspinal nerveL1 (one of thelumbar nerves) after this nerve receives fibers
from T12 (subcostal nerve). The inferior branch is theilioinguinal nerve.
It emerges from the upper part of the lateral border of thepsoas major, and
crosses obliquely in front of thequadratus lumborumto the iliac crest,
running posterior to the kidneys.
It then perforates the posterior part of thetransversus abdominis, near the
crest of the ilium, and divides between that muscle and theobliquus
internusabdominis into a lateral and an anterior cutaneous branch.
Branches include:
Lateral cutaneous branch of iliohypogastric nerve
Anterior cutaneous branch of iliohypogastric nerve
Communication
The iliohypogastric nerve communicates with thesubcostal
nerveandilioinguinal nerves.
(Q.44)

Depressor of Temporo-mandibular joint is


(a)

Temporalis muscle

(b)

Masseter muscle

(c)

Lateral pterygoid muscle

(d)

Medial pterygoid muscle

Your Response :
Correct Answer :
Exp:

c
Lateral pterygoid muscle
third ed. pg. 115)

(Ref. Anatomy by BD Chaurasia, Vol. III,

Depression is brought about mainly by lateral pterygoid.


The digastric, geniohyoid and mylohyoid muscles help when the mouth is
opened wide or against resistance.
MUSCLES OF MASTICATION
There are four major muscles of mastication:
Masseter,
Temporalis,
Lateral pterygoid, and
Medial pterygoid.
Q

The masseter, temporalis, and medial pterygoid muscles elevate the


mandible.
Q

The lateral pterygoid depresses and protrudes the mandible.

The medial and lateral pterygoids protrude the mandible when they
contract together and deviate the mandible from side to side in a grinding
motion.
The anterior belly of the digastrics and the mylohyoid are suprahyoid
muscles, which also act as muscles of mastication by depressing the
mandible.
Muscles in the infratemporal fossa and, in general, muscles that move the
mandible are innervated by the mandibular nerve of the trigeminal (CN
V3).
Skin over the mandible plus mucosa of the anterior two thirds of the
tongue and adjacent oral cavity is innervated by sensory fibers of CN
VExp.
(Q.45)

Space of Disse is seen in


(a)

Liver

(b)

Spleen

(c)

Kidney

(d)

Thymus

Your Response :
Correct Answer :
Exp:

a
Liver Reference: Junqueira's Basic Histology: Text and Atlas, 11th
Edition
Hepatocytes
Hepatocytes are 20- to 30-mm polyhedral cells.
Liver regeneration can occur rapidly under some circumstances. As much
as 90% can be replaced in about 2 weeks.
Their six or more surfaces may either contact another cell to form gap
junctions and bile canaliculi or form a free surface with microvilli
exposed to the perisinusoidal space of Disse.'"
Abundant glycogen in these cells takes the form of electron-dense
granules that are clustered near the SER.
There are several hundred mitochondria per liver cell.
The hepatocyte produces proteins for export (e.g., albumin, prothrombin,
fibrinogen), secretes bile, stores lipids and carbohydrates, converts lipids
and amino acidsJnto glucose via the enzymatic process of
gluconeogenesis, and detoxifies and inactivates drugs by oxidation,
methylation, and conjugation.

(Q.46)

Hassalls Corpuscles are present in


(a)

Thymus

(b)

Lymph node

(c)

Spleen

(d)

Liver

Your Response :
Correct Answer :
Exp:

a
Thymus Reference: Junqueira's Basic Histology: Text and Atlas, 11th
Edition
(Ref. Textbook of histology by Bloom, 11th ed.427)

The cortical lymphocytes of the thymus arise from stem cells of bone
marrow origin.
Most (95%) of the lymphocytes (T-lymphocytes) produced are
autoallergic (act against the host or self antigens), short-lived (3-5 days)
and never move out of the organ
They are destroyed within the thymus by phagocytes
Their remnants are seen in Hassals corpuscles.
Q

The thymus contains epithelial reticular cells and Hassall corpuscles in


the medulla and lacks germinal centers.
THYMUS
Flattened bilobed structure (sail shaped in children) lying between
sternum and pericardium.
Site of T-cell maturation and is Encapsulated.
Arises from epithelium of 3rd branchial pouches and lymphocytes
are of mesenchymal origin.
Is largest relative to body size during foetal life and at birth is twothird of its mature weight.
Mature thymus is at the age of 1 year and mass peaks just before
puberty.
Cortex is dense with immature T cells, medulla is pale with mature.
T cell and epithelial reticular cells and contains Hassalls
Corpuscles.
In Ataxia-Telangiectasia (Louis-Bar Syndrome)there is usually a
rudimentary thymus at best, with a paucity of Hassall's corpuscles,
associated with a generalized decrease in overall lymphoid tissue mass.
(Q.47)

Blood supply of external nose derives from


(a)

Ophthalmic artery

(b)

Maxillary artery

(c)

Palatine artery

(d)

Both Ophthalmic and maxillary arteries

Your Response :

Correct Answer :
Exp:

d
Both Ophthalmic and external maxillary arteries
(Ref. Head and Neck Surgery - Otolaryngology by Byron 3rd edition
437)
The external carotid artery provides blood to the nose primarily through
the maxillary artery and secondarily via the facial artery.
The facial artery gives rise to the superior labial artery, which in turn
gives nasal arterial branches medially to the septum and laterally to the
ala.
The maxillary artery is the terminal branch of the external carotid artery.
Within the infratemporal fossa, the maxillary artery passes either between
or lateral to the superior and inferior heads of the lateral pterygoid muscle.
The artery then enters the pterygopalatine fossa inferolaterally via the
ptergomaxillary fissure.
The maxillary artery and its branches are generally more anteroinferior
within the pterygopalatine fossa than the maxillary and vidian nerves, an
anatomic point that facilitates maxillary artery ligation for epistaxis.
The maxillary artery terminal branches of relevance to the subject of
epistaxis within the pterygopalatine fossa are the descending or greater
palatine artery, the pharyngeal artery, the posterior nasal artery, and the
sphenopalatine artery .
The maxillary artery bifurcates into the sphenopalatine and posterior nasal
arteries at or distal to the sphenopalatine foramen but may bifurcate as
early as the pterygomaxillary fissure.
The sphenopalatine artery supplies the septal mucosa and in the region of
the anteroinferior septum anastomoses with the greater palatine artery, the
anterior ethmoid artery, and the nasal branches of the facial artery, thus
forming Kiesselbach's plexus or Little's area.
The posterior nasal artery supplies the lateral nasal wall and the
turbinates. Superiorly it anastomoses with the ethmoid arteries. Inferiorly
it anastomoses with the pharyngeal arterial branches of the maxillary
artery, thus forming the Woodruff naso-nasopharyngeal plexus.

(Q.48)

Vasa vasorum provide a function analogous to that of which of the following?


(a)

Valves

(b)

Basal lamina

(c)

Coronary arteries

(d)

Endothelial diaphragms

Your Response :
Correct Answer :
Exp:

c
Coronary arteries
Vasa vasorum (VV) are vessels within a vessel and are found primarily in
the adventitia of large arteries and veins. They provide nutrition and
oxygenated blood to the thick media and adventitia of these vessels,
which are unable to obtain nutrition by diffusion from the lumen.
Coronary arteries fulfill a similar function for the myocardium.

(Q.49)

The weight of the upper limb is transmitted to the axial skeleton by:
(a)

Coracoclavicular ligament

(b)

Coracoacromial ligament

(c)

Costoclavicular ligament

(d)

Coracohumeral ligament

Your Response :
Correct Answer :
Exp:

a
Coracoclavicular ligament
(Ref. Keith Moore clinical anatomy 4th/pg. 787; BDC vol-Exp.
3rd/pg. 132)
The weight of the upper limb is transmitted to the axial skeleton by
Coracoclavicular ligament.
Coracoclavicular If this ligament tore, the shoulder would fall away
from the clavicle.
Acromioclavicular The AC joint is weak and easily injured by a direct
blow.
Coracoacromial

The coracoacromial arch is so strong that the humeral


body or clavicle would fracture first.

Costoclavicular

This is located between the first rib and the clavicle,


not near the acromion.

(Q.50)

In flexion and abduction of shoulder all of the following structures are compressed
except:
(a)

. Subacromial bursa

(b)

Long head of biceps

(c)

Suprascapular nerve

(d)

Supraspinatus tendon

Your Response :
Correct Answer :
Exp:

c
Suprascapular nerve
(Ref. Ebnezar textbook of Orthopedics 2nd/pg. 192)
Painful arc syndrome or Impingement syndrome
Impingement of following structures beneath coracoacromial arch,
between undersurface of acromion and head and greater tuberosity of
humerus.
Supraspinatous tendon
Long head of biceps
Subacromial bursa
Etiology:
Partial tear, inflammation or calcific deposits involving Rotator cuff
muscles, bicipital tendon, subacromial subdeltoid bursitis or fracture of
greater tuberosity.

(Q.51)

Which of the following is the smallest active functional unit of the lung?
(a)

An alveolus

(b)

A. respiratory bronchiolar unit

(c)

A. bronchopulmonary segment

(d)

Segmental bronchi

Your Response :
Correct Answer :

Exp:

A respiratory bronchiolar unit


The smallest functional unit of the lung is the respiratory bronchiolar unit,
which contains a respiratory bronchiole and the alveoli associated with it.
This unit allows for air conduction and gas exchange. The alveolus
(answer a) is only associated with gas exchange, and the bronchi form
part of the conduction system. The bronchopulmonary segment is a
functional unit of lung structure, but it is not the smallest unit.
Bronchopulmonary segments are particularly important in surgical
resections of the lung because they represent functional units with
connective tissue boundaries and individualized vasculature, including
pulmonary and bronchial arteries, pulmonary lymphatics, and pulmonary
nerves, all of which follow the air-conducting system of the bronchial tree
and its branches. Segmental bronchi and intrapulmonary bronchi are part
of the conduction system.

(Q.52)

The lung cells known as congestive heart failure cells are which of the following?
(a)

Type I pneumocytes

(b)

Type II pneumocytes

(c)

Macrophages

(d)

Fibroblasts

Your Response :
Correct Answer :
Exp:

c
Macrophages
The alveolar macrophage (containing hemosiderin) has been called the
congestive heart failure cell. The presence of large numbers of these
cells, containing hemosiderin granules, is an indicator of edematous lung
changes. During congestive heart failure, edema results in leakage of
erythrocytes into the alveoli. Transferrin and hemoglobin are also present
in the edematous fluid released from the capillaries. These two products
are phagocytosed by alveolar macrophages, which convert those products
to hemosiderin.

(Q.53)

All of the following are branches of the External Carotid Artery (ECA.) except:
(a)

Superior thyroid artery

(b)

Anterior Ethmoidal artery

(c)

Occipital artery

(d)

Posterior auricular artery

Your Response :
Correct Answer :
Exp:

b
Anterior Ethmoidal artery (Reference: Grays Anatomy, 39th edition)
Branches of external Carotid artery are eight:
1 Superior thyroid artery
3 Facial artery

2 Ascending pharyngeal artery

4 Inferior alveolar artery

5 Occipital artery

6 Posterior auricular artery

7 Middle meningeal artery


(Q.54)

8 Superficial temporal artery

The carpal tunnel contains all of the following important structures except:
(a)

Median Nerve

(b)

Flexor pollicis longus

(c)

Flexor carpi radials

(d)

Flexor digitorum superficial.

Your Response :
Correct Answer :
Exp:

c
(Flexor carpi radials) Reference: Grays Anatomy, 39th edition
Contents
The carpal tunnel contains a total of9 tendons, surrounded by synovial
sheaths, and themediannerve.
Tendons
The tendon offlexor pollicis longus
4 tendons offlexor digitorum profundus
4 tendons offlexor digitorum superficialis
The 8 tendons of the flexor digitorum profundus and flexor digitorum
superficialis are surrounded by a single synovial sheath. The tendon of
flexor pollicis longus is surrounded by its own synovial sheath. These
sheaths allow free movement of the tendons. Sometimes you may hear
that the carpal tunnel contains another tendon, the flexor carpi radialis

tendon, but this is located within the flexor retinaculum and not within the
carpal tunnel itself!
Median Nerve
Once it passes through the carpal tunnel, the median nerve divides into 2
branches: therecurrent branchandpalmar digitalnerves. The palmar digital
nerves give sensory innervation to thepalmarskinanddorsalnail bedsof
thelateralthree and a half digits.They also provide motor innervation to
thelateral two lumbricals. The recurrent branch supplies thethenarmuscle
group.
Clinical Relevance: Carpal Tunnel Syndrome
Compression of the median nerve within the carpal tunnel can causecarpal
tunnel syndrome (CTS). It is the most common mononeuropathy and can
be caused by thickened ligaments and tendon sheaths. Its aetiology is,
however, most often idiopathic. If left untreated, CTS can cause weakness
and atrophy of thethenarmuscles. The patients history will comment on
numbness, tingling and pain in thedistribution of the median nerve.The
pain will usually radiate to the forearm. Symptoms are often associated
with waking the patient from their sleep and being worse in the mornings.
Tests for CTS can be performed during physical examination:
Tapping the nerve in the carpal tunnel to elicit pain in median nerve
distribution (Tinels Sign)
Holding the wrist in flexion for 60 seconds to elicit numbness/pain in
median nerve distribution (Phalensmanoeuvre)
Treatment involves the use of asplint, holding the wrist in dorsiflexion
overnight to relieve symptoms. If this is
unsuccessful,corticosterioidinjections into the carpal tunnel can be used.
In severe case, surgical decompression of the carpal tunnel may be
required.
(Q.55)

The following statements concerning chorda tympani nerve are true except that it:
(a)

Carries secretomotor fibers to submandibular gland

(b)

Joins lingual nerve in infratemporal fossa

(c)

Is a branch of facial nerve

(d)

Contains postganglionic parasympathetic fibers.

Your Response :

Correct Answer :
Exp:

d
Contains postganglionic parasympathetic fibers
Chorda tympani nerve:
Arises from facial nerve 6mm above stylomastoid foramen.
Runs in bony facial canal, enters middle ear, lies in close relation to
tympanic membrane, leaves middle ear throngh petrotympanic fissure,
passes medial to spine of sphenoid & enters intratemporal fossa to join
lingnal nerve, through which it is disturbed.
It carries secretomotor fibers from submandibular ganglion & Supplies to
SMG & SLG & taste fibers to anterior2/3rd of tongue.

(Q.56)

Femoral ring is bounded by following structures except:


(a)

Femoral vein

(b)

Inguinal ligament

(c)

Femoral artery

(d)

Lacunar ligament

Your Response :
Correct Answer :
Exp:

c
Femoral artery (Ref. Human Anatomy by Chaurasia-3rd Edn-45)
Femoral ring is proximal end (base) of femoral canal.
It is bounded:
Anteriorly by Inguinal ligament
Posteriorly by Pectineus
Medially by Lacunar ligament &
Laterally by Femoral vein
From medial to lateral the relation is:
Lacunal ligament Femoral ring Canal Femoral Vein Femoral
artery Femoral nerve.

(Q.57)

The following group of lymph nodes receives


lymphatics from the uterus except:

(a)

External iliac

(b)

Internal iliac

(c)

Superficial Inguinal

(d)

Deep Inguinal

Your Response :
Correct Answer :
Exp:

d
Deep Inguinal

(Ref. Human Anatomy by Chaurasia-3rd Edn-319)

Lymphatic drainage of Uterus:


Fundus and prox body of uterus Aortic, superficial Inguinal nodes.
From cervix External iliac, Internal iliac, Sacral.
Distal body External iliac nodes.
(Q.58)

In an adult male, on per rectal examination, the following structures can be felt
anteriorly except:
(a)

Internal iliac lymph nodes

(b)

Bulb of Penis

(c)

Prostate

(d)

Seminal vesicles when enlarged

Your Response :
Correct Answer :
Exp:

a
(Internal iliac lymph nodes) (Ref. Human Anatomy by Chaurasia-3rd
Edn-331, 334)
In a normal person, the following structures can be palpated by a finger
passed per rectum:
Males

Females

Posterior surface of prostate

Perineal body

Seminal vesicles

Cervix

Vasa deferentia

Occasionally ovaries

Anorectal ring

Anorectal ring

Coccyx and sacrum

Coccyx and sacrum

Ischiorectal fossac and ischial


spines

Ischiorectal fossac ischial spine

Internal iliac lymph nodes are situated higher up, while bulb of Penis,
which is a part of root of penis attached to perineal membrane may be
palpable on PR examination anteriorly.
(Q.59)

Benign prostatic hypertrophy results in obstruction of urinary tract. The specific


condition is associated with enlargement of the:
(a)

Entire prostate gland

(b)

Lateral lobes

(c)

Median lobe

(d)

Posterior lobes

Your Response :
Correct Answer :
Exp:

c
(Median lobe) Reference: Grays Anatomy, 39th edition
There are 3 zones of prostate:
i) Central Zone (10% of prostatic cancer occur here; zone of BHP)
ii) Transitional Zone (20% of prostatic cancer occur here)
iii) Peripheral (70% of prostatic cancer occur here)
Anatomical division of lobes of prostate is of less clinical significance as
compared to zonal anatomy.
The various lobes are:
1Median lobe (Lobe of BHP i.e. Benign hypertrophy of prostate)
2Anterior lobe

(Q.60)

3 Posterior lobe and

4Two lateral lobes

The prostatic urethra is characterized by all the following features, except that it:
(a)

Is the widest and most dilatable part

(b)

It is circular in cross section.

(c)

Lies closer to anterior surface of prostate

(d)

Receives prostatic ductules along its posterior wall

Your Response :
Correct Answer :
Exp:

b
It is circular in cross section.
Prostatic urethra is widest and most dilatable part of male urethra.
Measures 3cm in length.
Semilunar on transverse section on with its convexity directed
forwards/anteriorly and concavity posteriorly.
Posterior wall is characterized by urethras crest/verumontanum, colliculus
seminalis and prostatic sinuses of openings of 20-30 prostatic glands
Membranous urethra is narrowest and least dilatable part of male urethra
containing bulb.
(Ref. BDC anatomy 4th ed. vol.2- pg.349; Oxford Textbook of Surgery
/2nd pg. 1430)
The lymphatics from the spongy part of urethra pass mostly to deep
inguinal lymph nodes, but some may end in the superficial inguinal and
the external iliac nodes.
The male urethra
In male sthe urethra is longer, but in the female urethra is about 3 cm long
and lies in the anterior vaginal wall.
It is surrounded at its midpoint by the somatic external sphincter (supplied
by the pudendal nerve S2, 3, 4), which, in combination with the internal
sphincter at the level of the bladder neck, maintains continence.
The male urethra is divided into four segments.
The prostatic urethra

extending from the bladder neck to the apex


of the prostate.
It is the widest part of the urethra
It is notable for a small protuberance on its
posterior surface near its distal end, known as
the verumontanum. Q
More than 95 percent are type 1 Posterior
Urethral valves, running from the

verumontanum to the distal urethra. Q


Verumontanum serves as an important
endoscopic landmark indicating the most
proximal aspect of the somatic external
sphincter.
The ejaculatory ducts and the prostatic glands
drain into the floor of the urethra at the level
of the verumontanum.
It is semilunar in shape on cross-section. Q
The membranous
urethra

running between the apex of the prostate and


the external perineal membran
It is 2 cm long and is surrounded by the
somatic external sphincter (innervated by the
pudendal nerve, S234).
The prostatomembranous junction is the most
common site of urethral disruption associated
with pelvic trauma. Q

The bulbar urethra

extends from the perineal membrane to the


beginning of the pendulous urethra.
It is surrounded by the bulbospongiosus
muscle.
It is the area most often injured when a person
falls astride an object.Q

The penile/ anterior/


pendulous urethra

(Q.61)

is contained within the penis.


Just proximal to the meatus (the narrowest
part of the urethra) there is a slight dilatation:
this area is known as the navicular fossa.

All of the following are components of white pulp of spleen, EXCEPT:


(a)

Periarteriolar lymphoid sheath

(b)

B cells

(c)

Antigen presenting cells

(d)

Vascular sinus

Your Response :

Correct Answer :
Exp:

d
(Vascular sinus) Reference: Junqueira's Basic Histology: Text and
Atlas, 11th Edition
The spleen is anatomically and functionally divided into two
compartments: the red pulp, where particles are effectively removed
from the blood, and the white pulp, where specific immune responses are
generated. The cellular components of both red and white pulp are
different.
A striking abundance of memory T cells is found in the white and red
pulp with an overall ratio of T and B cells in the white pulp being similar
to that in lymph nodes. Both NK and gamma delta T cells can be found in
white pulp and lymph nodes, but granulocytes are absent. Dendritic cell
subsets (antigen presenting cells) are present in spleen. The distribution of
dendritic cell subsets (antigen presenting cells) in spleen is significantly
different from that in lymph nodes as a different migration and retention
mechanism exists in the white pulp.
The reticular framework of the Periarteriolar lymphoid
sheath (PALS), lymph follicle (LF), and marginal zone (MZ) is
specialized is composed of heterogeneous components in the human
spleen. The heterogeneity of the framework may induce the segregation of
T and B lymphocytes.
The reticular framework of the white pulp (WP) and marginal zone (MZ)
consists of reticulum cells and reticulin fibers.

(Q.62)

In adults, the spinal cord normally ends at:


(a)

Lower border of L1

(b)

Lower border of L3

(c)

Lower border of S1

(d)

Lower border of S5

Your Response :
Correct Answer :
Exp:

a
Lower border of L1 (Ref. Anatomy by B.D. Chaurasia 3rd ed. Vol. III
103)
In adults, the spinal cord normally ends at the lower border of L1 and in
children it ends at LExp.

(Q.63)

Middle meningeal artery is direct branch of:


(a)

External carotid artery

(b)

Internal maxillary artery

(c)

Superficial temporal artery

(d)

Middle cerebral artery

Your Response :
Correct Answer :
Exp:

b
Internal maxillary artery (Reference: Grays Anatomy, 39th edition)
Following are the 8 branches of External Carotid Artery:
1 Superior thyroid artery

2 Ascending pharyngeal artery

3 Facial artery

4 Inferior alveolar artery

5 Occipital artery

6 Posterior auricular artery

7 Internal maxillary artery

8 Superficial temporal artery

Branches of Internal Maxillary Artery are:


Branches of 1st part of internal maxillary artery are middle meningeal
artery, tympanic artery, inferior alveolar artery, etc.
Branches of 2nd part are muscular branches
Branches of 3rd part are sphenopalatine artery, etc.
(Q.64)

All the following features are seen in neurons from dorsal root ganglia, except:
(a)

They have centrally located nuclei

(b)

They are derived from neural crest cells

(c)

They are multipolar

(d)

They contain lipofuscin granules

Your Response :
Correct Answer :
Exp:

c
They are multipolar
(Ref. IB Singh embryology 7th/pg. 312; IB singh histology 4th/pg.
164)

DRG or spinal ganglia are unipolar or pseudounipolar.


Sympathetic or autonomic ganglia are multipolar.
DRG (sensory or spinal ganglion)

Autonomic sympathetic
ganglion

1 Consist of pseudounipolar neurons

Consist of multipolar neurons

2 Has cell bodies of afferent neurons

Has cell bodies of efferent


neurons.

3 Neurons arranged in groups separated


by nerve fibers in groups

Neurons and nerve fibers are


scattered.

4 Large rounded cell body with central


nucleus and prominent nucleolus

Cell body is smaller and


irregular with eccentric
nucleus and prominent
nucleolus.

Neurons of sensory spinal nerve root ganglion, sympathetic (autonomic)


ganglia and sensory ganglia of 5th to 10th cranial nerves except 6th are
derived from neural crest.
(Q.65)

Which of the following is an intraarticular tendon?


(a)

Sartorius

(b)

Semitendinosus

(c)

Anconeus

(d)

Popliteus

Your Response :
Correct Answer :
Exp:

d
Popliteus (Ref. Anatomy by B.D. Chaurasia 3rd ed. Vol. II - 114)
Popliteus has intraarticular (knee joint) tendon.
Origin and insertion
It originates from lateral surface of lateral condyle of femur. Origin is
intracapsular .outer margin of lateral meniscus of knee joint. Insertionposterior surface of shaft of tibia above soleal line.
Innervation

Nerve supply is via thetibial nervefrom spinal roots L5 and SExp.


Action
The Popliteus assists in flexing the leg upon thethigh; when the leg is
flexed, it will rotate the tibia inward.
It is especially called into action at the beginning of the act of bending the
knee, in as much as it produces the slightinward rotationof the tibia, which
is essential in the early stage of this movement.
When the knee is in full extension; the femur slightly medially rotates on
the tibia to lock the knee joint in place. Popliteus is often referred to as the
"Key" to unlocking the knee since it begins knee flexion by laterally
rotating the femur on the tibia.
Popliteus is also attached to the lateral meniscus in the knee; and draws it
posteriorly during knee flexion to prevent crushing the meniscus between
the tibia/femur as the knee flexes.
(Q.66)

Identify the wrong match regarding the tendon anatomy terminology and its
pictorial representation?

(a)

Fusiform tendon

(b)

Triangular tendon

(c)

Strap with tendinous intersections

(d)

Multipennate tendon
Your Response :
Correct Answer :
Exp:

d
Multipennate tendon. REF : Clinical Anatomy by Regions, Richard
Snell 8th Ed Chapter 1 Pg 10
Option D depicts a Bipennate tendon anatomy.
The muscle fibers are bound together with delicate areolar tissue, which is
condensed on the surface to form a fibrous envelope, the epimysium. The
individual fibers of a muscle are arranged either parallel or oblique to the
long axis of the muscle. Because a muscle shortens by one third to one
half its resting length when it contracts, it follows that muscles whose
fibers run parallel to the line of pull will bring about a greater degree of
movement compared with those whose fibers run obliquely. Examples of
muscles with parallel fiber arrangements are the sternocleidomastoid, the
rectus abdominis, and the sartorius. Muscles whose fibers run obliquely to
the line of pull are referred to as pennate muscles (they resemble a
feather). A unipennate muscle is one in which the tendon lies along one
side of the muscle and the muscle fibers pass obliquely to it (e.g., extensor
digitorum longus). A bipennate muscle is one in which the tendon lies in
the center of the muscle and the muscle fibers pass to it from two sides
(e.g., rectus femoris). A multipennate muscle may be arranged as a series
of bipennate muscles lying alongside one another (e.g., acromial fibers of
the deltoid) or may have the tendon lying within its center and the muscle
fibers passing to it from all sides, converging as they go (e.g., tibialis
anterior). For a given volume of muscle substance, pennate muscles have

many more fibers compared to muscles with parallel fiber arrangements


and are therefore more powerful; in other words, range of movement has
been sacrificed for strength.
(Q.67)

Ligamentum teres is remnant of:


(a)

Left umbilical vein

(b)

Right umbilical vein

(c)

Ductus arteriosus

(d)

Ductus venosus

Your Response :
Correct Answer :
Exp:

a
Left umbilical vein) Reference: Grays Anatomy, 39th edition
The two Umbilical Veins fuse early to form a single trunk in the bodystalk, but remain separate within the embryo and pass forward to the sinus
venosus in the sidewalls of the body. Like the vitelline veins, their direct
connection with the sinus venosus becomes interrupted by the developing
liver, and thus at this stage the whole of the blood from the yolk-sac and
placenta passes through the substance of the liver before it reaches the
heart.
The right umbilical and right vitelline veins shrivel and disappear; the left
umbilical, on the other hand, becomes enlarged and opens into the upper
venous ring of the vitelline veins; with the atrophy of the yolk-sac the left
vitelline vein also undergoes atrophy and disappears.
Finally a direct branch is established between this ring and the right
hepatic vein; this branch is named the ductus venosus, and, enlarging
rapidly, it forms a wide channel through which most of the blood,
returned from the placenta, is carried direct to the heart without passing
through the liver.
A small proportion of the blood from the placenta is, however, conveyed
from the left umbilical vein to the liver through the left vena advehens.
The left umbilical vein and the ductus venosus undergo atrophy and
obliteration after birth, and form respectively the ligamentum teres and
ligamentum venosum of the liver.

(Q.68)

Study the structure of Lower Esophageal sphincter displayed alongside. Which of the
following is not a component of LES?

(a)

Lower esophageal smooth muscle (Intrinsic sphincter)

(b)

Skeletal muscle fibres of crura of diaphragm

(c)

Sling fibres of stomach

(d)

Phrenicoesophageal ligament.

Your Response :
Correct Answer :
Exp:

d
Phrenicoesophageal ligament.
Lower Esophageal Sphincter
Unlike the rest of the esophagus, the musculature of the gastroesophageal
junction (lower esophageal sphincter; LES) is tonically active but
relaxes on swallowing. The tonic activity of the LES between meals
prevents reflux of gastric contents into the esophagus. The LES is made
up of three components . The esophageal smooth muscle is more
prominent at the junction with the stomach (intrinsic sphincter). Fibers
of the crural portion of the diaphragm, a skeletal muscle, surround the
esophagus at this point (extrinsic sphincter) and exert a pinchcock-like
action on the esophagus. In addition, the oblique or sling fibers of the
stomach wall create a flap valve that helps close off the esophagogastric
junction and prevent regurgitation when intragastric pressure rises. The
tone of the LES is under neural control. Release of acetylcholine from
vagal endings causes the intrinsic sphincter to contract, and release of NO
and VIP from interneurons innervated by other vagal fibers causes it to
relax. Contraction of the crural portion of the diaphragm, which is
innervated by the phrenic nerves, is coordinated with respiration and
contractions of chest and abdominal muscles. Thus, the intrinsic and
extrinsic sphincters operate together to permit orderly flow of food into
the stomach and to prevent reflux of gastric contents into the esophagus.
Note that the lower esophageal sphincter (intrinsic sphincter) is
supplemented by the crural portion of the diaphragm (extrinsic sphincter),
and that the two are anchored to each other by the phrenoesophageal

ligament.
(Q.69)

Direct Branches of Axillary artery are all EXCEPT:


(a)

Anterior circumflex humeral artery

(b)

Posterior circumflex humeral artery

(c)

Thoracodorsal artery

(d)

Subscapular artery

Your Response :
Correct Answer :
Exp:

c
Thoracodorsal artery
The axillary artery has several smaller branches. The origin of these
branches is highly variable (e.g. the posterior and anterior circumflex
arteries often have a common trunk). An arterial branch is named for its
course, not its origin.
First part (1 branch)
Superior thoracic artery(Supreme thoracic artery)
Second part (2 branches)
Thoraco-acromial artery
Lateral thoracic artery
Third part (3 branches)
Subscapular artery
Anterior humeral circumflex artery
Posterior humeral circumflex artery
Continues as thebrachial arterypast the inferior border of theteres major.
Thoraco-dorsal artery is a branch of Subscapular artery and thus not a
direct branch from Axillary artery.

(Q.70)

False statement regarding small intestine is ?


(a)

The caliber of small intestine is smaller

(b)

Longitudinal muscle around the small intestine forms continuous coat

(c)

Villi are present in small intestine

(d)

Small intestine has fatty tags attached to its wall.

Your Response :
Correct Answer :
Exp:

d
Small intestine has fatty tags attached to its wall.
REF : Clinical Anatomy by Regions : Richard Snell : 8th Ed
Differences Between the Small and Large Intestine
External Differences
The small intestine (with the exception of the duodenum) is mobile,
whereas the ascending and descending parts of the colon are fixed.
The caliber of the full small intestine is smaller than that of the filled large
intestine.
The small intestine (with the exception of the duodenum) has a mesentery
that passes downward across the midline into the right iliac fossa.
The longitudinal muscle of the small intestine forms a continuous layer
around the gut. In the large intestine (with the exception of the appendix)
the longitudinal muscle is collected into three bands, the teniae coli.
The small intestine has no fatty tags attached to its wall. The large
intestine has fatty tags, called the appendices epiploicae.
The wall of the small intestine is smooth, whereas that of the large
intestine is sacculated.
Internal Differences
The mucous membrane of the small intestine has permanent folds, called
plicae circulares, which are absent in the large intestine.
The mucous membrane of the small intestine has villi, which are absent in
the large intestine.
Aggregations of lymphoid tissue called Peyer's patches are found in the
mucous membrane of the small intestine; these are absent in the large
intestine.

(Q.71)

Which of the following is a Traction epiphysis?


(a)

Coracoid process

(b)

Lesser trochanter

(c)

Anterior superior iliac spine

(d)

Xiphisternum

Your Response :
Correct Answer :
Exp:

b
Lesser trochanter
Epiphysis:
The ends and the tips of a long bone that ossify from secondary centers
are known as epiphyses. Epiphyses are of 4 types and they can be better
understood by explaining their types;
Pressure epiphyses:These epiphyses are articular that is they take part in
joint formation. These epiphyses are the weight transmitting epiphyses.
Examples are head of the humerus, lower end of the radius etc.
Traction epiphyses:These are non-articular and do not take part in the
formation of joints. They also do not take part in transmission of weight.
The actual job of traction epiphyses is to provide attachment to tendons of
muscles. The traction epiphyses ossify later than the pressure epiphyses.
Examples of these epiphyses are tubercles of humerus (greater tubercle
and lesser tubercle) and trochanters of femur (greater trochanter and lesser
trochanter)
Atavistic epiphyses:These epiphyses are phylogenetically independent
but they become fused in man. Examples are coracoid process of scapula
and os trigonum.
Aberrant epiphyses:As the name indicates (aberrant = not usual) these
epiphyses are not always present. Examples are epiphyses at the head of
the first metacarpal bone and at the base of other metacarpal bones.

(Q.72)

All are functions of Sertoli cells EXCEPT:


(a)

Maintains blood testis barrier

(b)

Formation of androgen binding protein

(c)

Provide nutrition to sperm

(d)

Secretes testosterone.

Your Response :

Correct Answer :
Exp:

d
(Secretes testosterone) Reference: Junqueira's Basic Histology: Text
and Atlas, 11th Edition
SERTOLI CELLS
Elongated supporting cells with base at basal lamina and apex near lumen
of seminiferous tubule
Lateral processes surround spermatogonia and spermatocytes
Nucleus euchromatic with distinct nucleolus
Form blood testis barrier
Connected by gap junctions
Functions of Sertoli Cells
Support and nourish cells of seminiferous tubule
No vasculature inside tubule so Sertoli cells transport nutrients to luminal
region
Phagocytose cast-off cytoplasmic droplets from spermatids
Secrete fluid into lumen that helps sperm flow in tubules
Produce androgen binding protein
Produce inhibin which decreases FSH release by pituitary, slowing
spermatogenesis
INTERSTITIAL TISSUE
Between seminiferous tubules are vessels, nerves, fibroblasts and other
connective tissue cells (Interstitial cells of Leydig).
After puberty Leydig cells differentiate (testosterone producing cells)
Leydig cells are eosinophilic and have lots of SER and mitochondria with
tubular cristae.

(Q.73)

Which of the following passes through the posterior part of Jugular foramen?
(a)

Inferior petrosal sinus

(b)

Vagus nerve

(c)

Superior petrosal sinus

(d)

Internal jugular vein

Your Response :
Correct Answer :
Exp:

a
Internal jugular vein (Ref. BDC Anatomy Vol III, 4th ed., 1619)
Structures Passing Through Jugular Foramen
Through anterior part
Through mid part

Inferior petrosal sinus

9th, 10th and 11th cranial nerves

Through posterior part

Internal jugular vein

Summary of the More Important Openings in the Base of the Skull and
the Structures That Pass Through Them
Opening in
Skull

Bone of Skull

Structures Transmitted

Anterior Cranial Fossa


Perforations in Ethmoid
cribriform plate

Olfactory nerves

Middle Cranial Fossa


Optic canal

Lesser wing of
sphenoid

Optic nerve, ophthalmic artery

Superior orbital Between lesser and Lacrimal, frontal, trochlear,


fissure
greater wings of
oculomotor, nasociliary, and
sphenoid
abducent nerves; superior
ophthalmic vein
Foramen
rotundum

Greater wing of
sphenoid

Maxillary division of the


trigeminal nerve

Foramen ovale Greater wing of


sphenoid

Mandibular division of the


trigeminal nerve, lesser petrosal
nerve

Foramen
spinosum

Greater wing of
sphenoid

Middle meningeal artery

Foramen
lacerum

Between petrous
Internal carotid artery
part of temporal and

sphenoid
Posterior Cranial Fossa

(Q.74)

Foramen
magnum

Occipital

Medulla oblongata, spinal part of


accessory nerve, and right and left
vertebral arteries

Hypoglossal
canal

Occipital

Hypoglossal nerve

Jugular
foramen

Between petrous
Glossopharyngeal, vagus, and
part of temporal and accessory nerves; sigmoid sinus
condylar part of
becomes internal jugular vein
occipital

Internal
acoustic
meatus

Petrous part of
temporal

Vestibulocochlear and facial


nerves

Which of the following develops from the third pharyngeal pouch?


(a)

Superior parathyroids

(b)

Inferior parathyroids

(c)

Tonsils

(d)

Thyroid gland

Your Response :
Correct Answer :
Exp:

b
Inferior parathyroids
BRANCHIAL POUCH DERIVATIVES
Medical Embryology (10th edition)
1st pouch
2nd pouch

Middle ear cavity, Eustachian tube, mastoid air cells.


Epithelial lining of palatine tonsil.

3rd pouch (dorsal wings)


3rd pouch (ventral wings)
4th pouch
(Q.75)

Reference: Langmans

Inferior parathyroids.
Thymus.

Superior parathyroids.

Secretomotor nerve fibers to the parotid gland are supplied by which of the

following?
(a)

Facial

(b)

Vagus

(c)

Glossopharyngeal

(d)

Trigeminal

Your Response :
Correct Answer :
Exp:

c
(Glossopharyngeal) Reference: Keith L. Moore, Clinically oriented
Anatomy, 6th edition
Nerve Supply Of Parotid Gland
The parotid gland receives its PNS fibers from cranial nerve IX.
The fibers travel with the lesser superficial petrosal nerve, synapse at the
otic ganglion, and travel to the
Gland by way of the auriculotemporal nerve.
The PNS fibers to the submandibular and sublingual glands travel with
the chorda tympani branch of
Cranial nerve VII, which joins the lingual nerve before synapsing at the
submandibular ganglion just adjacent to the glands.
Sympathetic nerve fibers arise in the superior cervical ganglion and travel
with the glands arterial supply:
External carotid artery to the parotid, lingual artery to the submandibular
gland, and facial artery to the sublingual gland.
The auriculotemporal nerve, a branch of the mandibular (third) division of
cranial nerve V, carries
postganglionic parasympathetic fibers from the otic ganglion to the
parotid gland.
Auriculotemporal nerve injury during a parotidectomy can result in
gustatory sweating (Frey syndrome).
There is no role surgery in established Freys syndrome.

(Q.76)

In a post polio case, ilio-tibial tract contracture is likely to result in:


(a)

Extension at the hip and knee

(b)

Extension at the hip

(c)

Flexion at the hip and the knee

(d)

Extension at the knee

Your Response :
Correct Answer :
Exp:

c
Flexion at the hip and the knee
Flexion at the hip and the knee
Explanation:
-Contracture of iliotibial tract in polio leads to flexion at the hip & the
knee joint.
-It also leads to abduction and lateral rotation at the hip joint and the knee
joint.

(Q.77)

Gartners ducts are remnants of:


(a)

Mesonephric duct

(b)

Paramesonephric duct

(c)

Metanephric blastema

(d)

Urogenital sinus

Your Response :
Correct Answer :
Exp:

a
Mesonephric duct (Ref. BDC Vol II 4th /., 366)
MESONEPHRIC (WOLFFIAN) DUCT
Gartners ducts are remnants of the mesonephric (Wolffian) duct in the
female and consist of a series of tiny vestigial cysts extending along the
lateral aspect of the uterus to the vaginal vestibule.
At 28 days gestation, a bud arises from the caudal end of each
mesonephric (wolffian) duct and grows dorsad until its blind end reaches
a mass of metanephric mesoderm that forms the metanephric cap. The

stalk of this bud forms the ureter, pelvis, calices, and collecting tubules.
The metanephric cap forms the nephrons.
The mesonephric ducts, in a male, migrate caudad, become incorporated
into the wall of the urogenital sinus, and form the vas deferens and
ejaculatory ducts. In a female, they regress and remain only as the ducts
of Gartner in the vaginal walls.
(Q.78)

False statement regarding renal anomalies is ?


(a)
(b)
(c)
(d)

Horse shoe kidney is low lying than normal


Rosette kidney is a result of the early fusion of the two ureteric buds
in the pelvis
Post caval ureter is when ureter courses anterior to the IVC
Bifid ureters are more liable to become infected or to be the seat of
calculus formation than a normal ureter.

Your Response :
Correct Answer :
Exp:

c
Post caval ureter is when ureter courses anterior to the IVC
Development of the Kidneys and Ureters
Three sets of structures in the urinary system appear, called the
pronephros, mesonephros, and metanephros. In the human, the
metanephros is responsible for the permanent kidney. The metanephros
develops from two sources: the ureteric bud from the mesonephric duct
and the metanephrogenic cap from the intermediate cell mass of
mesenchyme of the lower lumbar and sacral regions.
Ureteric Bud
The ureteric bud arises as an outgrowth of the mesonephric duct. It forms
the ureter, which dilates at its upper end to form the pelvis of the ureter.
The pelvis later gives off branches that form the major calyces, and these
in turn divide and branch to form the minor calyces and the collecting
tubules.
Metanephrogenic Cap
The metanephrogenic cap condenses around the ureteric bud and forms
the glomerular capsules, the proximal and distal convoluted tubules, and
the loops of Henle. The glomerular capsule becomes invaginated by a
cluster of capillaries that form the glomerulus. Each distal convoluted
tubule formed from the metanephrogenic cap tissue becomes joined to a

collecting tubule derived from the ureteric bud. The surface of the kidney
is lobulated at first, but after birth, this lobulation usually soon disappears.
The developing kidney is initially a pelvic organ and receives its blood
supply from the pelvic continuation of the aorta, the middle sacral artery.
Later, the kidneys ascend up the posterior abdominal wall. This so-called
ascent is caused mainly by the growth of the body in the lumbar and
sacral regions and by the straightening of its curvature. The ureter
elongates as the ascent continues.
The kidney is vascularized at successively higher levels by successively
higher lateral splanchnic arteries, branches of the aorta. The kidneys reach
their final position opposite the second lumbar vertebra. Because of the
large size of the right lobe of the liver, the right kidney lies at a slightly
lower level than the left kidney.
RENAL ANOMALIES : A COMPREHENSIVE ACCOUNT :
Polycystic Kidney A hereditary disease, polycystic kidneys can be
transmitted by either parent. It may be associated
with congenital cysts of the liver, pancreas, and lung.
Both kidneys are enormously enlarged and riddled
with cysts. Polycystic kidney is thought to be caused
by a failure of union between the developing
convoluted tubules and collecting tubules. The
accumulation of urine in the proximal tubules results
in the formation of retention cysts.
Pelvic Kidney

In pelvic kidney, the kidney is arrested in some part


of its normal ascent; it usually is found at the brim of
the pelvis. Such a kidney may present with no signs
or symptoms and may function normally. However,
should an ectopic kidney become inflamed, it may
because of its unusual position give rise to a
mistaken diagnosis.

Horseshoe Kidney When the caudal ends of both kidneys fuse as they
develop, the result is horseshoe kidney. Both kidneys
commence to ascend from the pelvis, but the
interconnecting bridge becomes trapped behind the
inferior mesenteric artery so that the kidneys come
to rest in the low lumbar region. Both ureters are
kinked as they pass inferiorly over the bridge of
renal tissue, producing urinary stasis, which may
result in infection and stone formation. Surgical
division of the bridge corrects the condition.
Unilateral Double The kidney on one side may be double, with separate

Kidney

ureters and blood vessels. In unilateral double


kidney, the ureteric bud on one side crosses the
midline as it ascends, and its upper pole fuses with
the lower pole of the normally placed kidney. Here
again, angulation of the ureter may result in stasis of
the urine and may require surgical treatment.

Rosette Kidney

Both kidneys may fuse together at their hila, and


they usually remain in the pelvis. The two kidneys
together form a rosette. This is the result of the early
fusion of the two ureteric buds in the pelvis.

Supernumerary
Renal Arteries

Supernumerary renal arteries are relatively common.


They represent persistent fetal renal arteries, which
grow in sequence from the aorta to supply the kidney
as it ascends from the pelvis. Their occurrence is
clinically important because a supernumerary artery
may cross the pelviureteral junction and obstruct the
outflow of urine, producing dilatation of the calyces
and pelvis, a condition known as hydronephrosis

Double Pelvis

Double pelvis of the ureter is usually unilateral. The


upper pelvis is small and drains the upper group of
calyces; the larger lower pelvis drains the middle and
lower groups of calyces. The cause is a premature
division of the ureteric bud near its termination.

Bifid Ureter

In bifid ureter, the ureters may join in the lower third


of their course, may open through a common orifice
into the bladder, or may open independently into the
bladder. In the latter case, one ureter crosses its
fellow and may produce urinary obstruction. The
cause of bifid ureter is a premature division of the
ureteric bud.
Cases of double pelvis and double ureters may be
found by chance on radiologic investigation of the
urinary tract. They are more liable to become
infected or to be the seat of calculus formation than a
normal ureter.

Megaloureter

Megaloureter may be unilateral or bilateral and


shows complete absence of motility. The cause is
unknown. Because of the urinary stasis, the ureter is
prone to infection. Plastic surgery is required to
improve the rate of drainage.

Postcaval Ureter

(Q.79)

The right ureter may ascend posterior to the inferior


vena cava and may be obstructed by it. Surgical
rerouting of the ureter with reimplantation of the
distal end into the bladder is the treatment of choice.

Invertor of foot:
(a)

Gastrocnemius

(b)

Tibialis posterior

(c)

Peroneus tertius

(d)

Peroneus longus

Your Response :
Correct Answer :
Exp:

b
Tibialis posterior (Ref. BDC Vol. II 4th ed. ed., 156)
Tibialis anterior and Tibialis posterior are principal invertors of foot.
Peroneus longus and Peroneus brevis are principal evertors of foot.

(Q.80)

White line of Toldt is:


(a)

Reflected lateral attachment of colonic peritonium.

(b)

Avascular plane at anal verge.

(c)

Line between upper and lower anal canal.

(d)

Supporting tissue of anal cushions.

Your Response :
Correct Answer :
Exp:

a
Reflected lateral attachment of colonic peritonium
Lateral reflection of posterior parietal pleura of abdomen over the
mesentery of the ascending and descending colon. It is the junction of
parietal peritoneum with Denonvilliers fascia.

(Q.81)

Which of the following lymph nodes drain the tongue base?


(a)

Submental and Submandibular nodes.

(b)

Superior deep jugular nodes.

(c)

Inferior deep jugular nodes.

(d)

Supraclavicular group.

Your Response :
Correct Answer :
Exp:

b
Superior deep jugular nodes (Ref. BDC Vol III 4th ed. 252)
The Lymphatics Of The Neck
The lymphatics of the neck are arranged into deep and superficial chains.
The deep jugular chain extends from the base of the skull to the clavicle
and is formed into superior, middle and inferior groups of lymph nodes.
The superior deep jugular nodes receive primary drainage from the soft
palate, tonsils, palatoglossal and palatopharyngeal arches, posterior
tongue, base of the tongue, pyriform sinus and the larynx above the vocal
folds.
The middle deep jugular nodes receive primary drainage from the larynx
above the vocal folds, lower pyriform sinus and posterior cricoid. They
receive secondary drainage from the deep jugular nodes above them and
the lower retropharyngeal nodes.
The inferior deep jugular nodes receive primary drainage from the
thyroid, trachea and cervical esophagus. They receive secondary drainage
from the deep jugular nodes above them and the paratracheal nodes. The
retropharyngeal and paratracheal nodes lie posteriorly around the midline
viscera. They receive drainage from these viscera and from the deep
structures in the midline of the head, i.e. the nasopharynx, posterior nasal
cavity, paranasal sinuses, posterior oropharynx. They drain towards the
deep jugular chain.
The superficial nodes tend to drain secondarily as mentioned to the deep
nodes. The superficial nodes are the submental, superficial cervical,
submandibular, spinal accessory and anterior scalene.
The submental nodes drain the chin, the middle of the lower lip, tip of the
tongue and anterior mouth. These nodes in turn drain to the
submandibular nodes.
The submandibular nodes drain the upper lip, lateral lower lip, lower
nasal cavity, anterior mouth and the skin of the cheek. The submandibular
nodes in turn drain to the superior deep jugular nodes.
The superficial cervical nodes located along the external jugular vein

receive drainage from the cutaneous lymphatics of the face, especially


from around the parotid gland, behind the ear, parotid and occipital nodes.
The superficial cervical then drain into the superior deep jugular nodes.
The nodes in the posterior triangle lie along the spinal accessory nerve.
These nodes drain the parietal and occipital regions of the scalp. The
upper nodes drain to the superior deep jugular nodes while the lower
nodes drain down to the supraclavicular nodes.
The anterior scalene (Virchows) nodes receive drainage from the thoracic
duct and are located at the junction of the thoracic duct and left subclavian
vein. They are usually the site of metastases from lower down in the body
(e.g. stomach). The supraclavicular nodes receive drainage from the spinal
accessory nodes and from infraclavicular sources.
All of these lymphatics eventually drain into the venous system either
through or together with the thoracic duct on the left, or the right
lymphatic duct.
(Q.82)

Testes are at the deep inguinal ring at the intrauterine age of _______?
(a)

3 months

(b)

6 months

(c)

9 months

(d)

At the birth

Your Response :
Correct Answer :
Exp:

b
6 months (Ref. BDC Vol II 4th ed., 218 and Bailey and Love 24th ed.,
1402)
TESTICULAR DESCENT
LOCATION of TESTES
At Deep Inguinal ring
Inguinal canal

4-6 months
7 months

Superficial Inguinal ring


Scrotum
Base of scrotum

AGE

early part of 8th month

late part of 8th month


9 months

Between the 3rd month of pregnancy and its end the testes become

transferred from the lumbar area (ventro-medial to the mesonephros) into


the future scrotum. This transfer is due to a combination of growth
processes and hormonal influences. The gubernaculum testis also plays a
decisive role in this phenomenon.
The gubernaculum testis arises in the course of the 7th week from the
lower gubernaculum, after the mesonephros has atrophied. Cranially it has
its origin at the testis and inserts in the region of the genital swelling
(future scrotum).
At the same time, at the inguinal canal along the lower gubernaculum, an
evagination of the peritoneum arises, the vaginal process, on which the
testes will slide through the inguinal canal.
In that the vaginal process lengthens downwardly, it takes the muscle
fibers of the oblique internal muscle and the transverse muscle with it.
The muscle fascia of the transverse muscle is the innermost layer and in
the scrotal region, it forms the internal spermatic fascia of the spermatic
cord and the scrotum. The muscle layer of the musculus cremaster is
formed from fibers of the oblique internal and transverse muscles.
Externally, the external spermatic fascia is formed from the superficial
aponeurosis of the oblique external abdominal muscle.
The region, where the testes pass through the abdominal wall, is called the
inguinal canal.
Between the 7th and the 12th week the gubernaculum shortens and pulls
the testes, the deferent duct and its vessels downwards.
Between the 3rd and 7th month the testes stay in the area of the inguinal
canal so they can enter into it. They reach the scrotum at roughly the time
of birth under the influence of the androgen hormone.
While in the first year of life the upper part of the vaginal process
becomes obliterated, there remains only the peritoneo-vaginal ligament.
The lower portion persists as the tunica vaginalis testis, which consists of
a parietal and a visceral layer.
(Q.83)

Navicular tuberosity gives attachment to:


(a)

Tibialis anterior

(b)

Tibialis posterior

(c)

Adductor hallucis longus

(d)

Flexor hallucis brevis

Your Response :
Correct Answer :
Exp:

b
Tibialis posterior (Ref. BDC Vol II 4th ed. 100, 106, 115)
Navicular Bone
The navicular is a flattened, oval, boat-shaped bone.
Located between the head of the talus and the three cuneiform bones, it
has facets for articulation with each of them.
The navicular also has an occasional facet for articulation with the cuboid
bone.
Medially and inferiorly, there is a rough navicular tuberosity to which the
tendon of the tibialis posterior muscle attaches.

(Q.84)

GA.LT constitutes ________ % of body lymphoid tissue.


(a)

40

(b)

50

(c)

60

(d)

70

Your Response :
Correct Answer :
Exp:

d
70 (Ref. Winterobes hematology 11th ed. 78)
GUT-ASSOCIATED LYMPHOID TISSUE (GALT)
The digestive tracts immune system is often referred to as gut-associated
lymphoid tissue (GALT) and works to protect the body from invasion.
GALT is an example of mucosa-associated lymphoid tissue.
About 70% of the bodys immune system is found in the digestive tract.
The GALT is made up of several types of lymphoid tissue that produce
and store immune cells that carry out attacks and defend against
pathogens.
Most Common site of extra-nodal of Lymphoma: GI tract (stomach)
Lymphoid tissue in the gut is comprised of the following :

Tonsils (Waldeyers ring)


Adenoids (Pharyngeal tonsils)
Peyers patches
Lymphoid aggregates in the appendix and large intestine
Lymphoid tissue accumulating with age in the stomach
Small lymphoid aggregates in the oesophagus
Diffusely distributed lymphoid cells and plasma cells in the lamina
propria of the gut
(Q.85)

The following paranasal sinus drains in middle meatus except ?


(a)

Maxillary sinus

(b)

Frontal sinus

(c)

Anterior ethmoidal sinus

(d)

Posterior ethmoidal sinus

Your Response :
Correct Answer :
Exp:

d
Paranasal Sinuses and Their Site of Drainage Into the Nosea
Sinus

Site of Drainage

Maxillary sinus

Middle meatus through hiatus semilunaris

Frontal sinuses

Middle meatus via infundibulum

Sphenoidal sinuses

Sphenoethmoidal recess

Ethmoidal sinuses
Anterior group

Infundibulum and into middle meatus

Middle group

Middle meatus on or above bulla ethmoidalis

Posterior group

Superior meatus

Note that maxillary and sphenoidal sinuses are present in rudimentary


form at birth, enlarge appreciably after the eighth year, and are fully

formed in adolescence.

(Q.86)

DangerousArea of Scalp:
(a)

Loose areola tissue

(b)

Dense subcutaneous tissue

(c)

Galiea aponeurotica

(d)

Pericranium

Your Response :
Correct Answer :
Exp:

a
Loose areola tissue (Ref. BDC Vol III, 3rd ed., pg 38)
LAYERS OF SCALP
S Skin

C - Subcutaneous tissue

A Aponeurosis

L - Loose areolar tissue

P - Pericranium

Venous plexus in loose areolar tissue has communication with the


intracranial venous sinus; hence the infection of scalp involving loose
areolar tissue may spread to the venous sinuses and hence can be
dangerous.
(Q.87)

Lymphatic flow is maintained by?


(a)

Movement & gravity

(b)

Contraction of Muscles

(c)

Valve in lymphatics

(d)

Valve in vessels

Your Response :
Correct Answer :
Exp:

b
Contraction of Muscles

(Ref. Ganong Physiology 22nd ed. ch. 30)

LYMPHATIC CIRCULATION
Lymphatic vessels can be divided into two types: initial lymphatics and

collecting lymphatics.
The former lack valves and smooth muscle in their walls, and they are
found in regions such as the intestine or skeletal muscle.
Tissue fluid appears to enter them through loose junctions between the
endothelial cells that form their walls.
The fluid in them apparently is massaged by muscle contractions of the
organs and contraction of arterioles and venules, with which they are
often associated.
They drain into the collecting lymphatics, which have valves and smooth
muscle in their walls and contract in a peristaltic fashion, propelling the
lymph along the vessels.
Flow in the collecting lymphatics is further aided by movements of
skeletal muscle, the negative intrathoracic pressure during inspiration, and
the suction effect of high-velocity flow of blood in the veins in which then
lymphatics terminate.
However, the contractions are the principal factor propelling the lymph.
(Q.88)

Lockwoods ligament is located at ?


(a)

Foot

(b)

Wrist

(c)

CV Junction

(d)

Eyeball

Your Response :
Correct Answer :
Exp:

d
Eyeball
SOME SPECIFIC NAMED LIGAMENTS IN THE BODY :
Weitbrechts ligament

The oblique cord connecting the


ulna and the radius

Pouparts ligament

Inguinal ligament

Petits ligament

Thickened portion of Pelvic fascia


between the cervix and vagina.
Passes posteriorly in the rectoUterine fold to attach to the Ant.

surface of sacrum

(Q.89)

Gimbernats ligament

Lacunar ligament which forms the


medial portion of the femoral ring

Mackenrodts ligament

Transverse cervical ligamentimportant support of the uterus

Lisfrancs ligament

Ligament joining the cuneiform


bone of ankle to the 2nd metatarsal.

Henles ligament

The conjoined tendon of the


transverses abdominis muscle

Lockwoods ligament

Suspensory ligament of the eyeball

Which of the following vein is present in posterior interventricular sulcus?


(a)

Coronary sinus

(b)

Small cardiac vein

(c)

Middle cardiac vein

(d)

Great cardiac vein

Your Response :
Correct Answer :
Exp:

c
Middle cardiac vein (Ref. Grays Anatomy of the Human Body
section V Angiology)
THE VEINS OF THE HEART
Coronary Sinus (Sinus Coronarius).
Most of the veins of the heart open into the coronary sinus.
This is a wide venous channel about Exp.25 cm. in length situated in the
posterior part of the coronary sulcus, and covered by muscular fibers from
the left atrium.
It ends in the right atrium between the opening of the inferior vena cava
and the atrioventricular aperture, its orifice being guarded by a semilunar
valve, the valve of the coronary sinus (valve of Thebesius).
Tributaries.Its tributaries are the great, small, and middle cardiac veins,
the posterior vein of the left ventricle, and the oblique vein of the left

atrium, all of which, except the last, are provided with valves at their
orifices.
The Great Cardiac Vein (v. cordis magna; left coronary vein) begins at the
apex of the heart and ascends along the anterior longitudinal sulcus to the
base of the ventricles. It then curves to the left in the coronary sulcus, and
reaching the back of the heart, opens into the left extremity of the
coronary sinus. It receives tributaries from the left atrium and from both
ventricles: one, the left marginal vein, is of considerable size, and ascends
along the left margin of the heart.
The Small Cardiac Vein (v. cordis parva; right coronary vein) runs in the
coronary sulcus between the right atrium and ventricle, and opens into the
right extremity of the coronary sinus. It receives blood from the back of
the right atrium and ventricle; the right marginal vein ascends along the
right margin of the heart and joins it in the coronary sulcus, or opens
directly into the right atrium.
The Middle Cardiac Vein (v. cordis media) commences at the apex of the
heart, ascends in the posterior longitudinal sulcus, and ends in the
coronary sinus near its right extremity.
The Posterior Vein of the Left Ventricle (v. posterior ventriculi sinistri)
runs on the diaphragmatic surface of the left ventricle to the coronary
sinus, but may end in the great cardiac vein.
The Oblique Vein of the Left Atrium (v. obliqua atrii sinistri [Marshalli];
oblique vein of Marshall) is a small vessel which descends obliquely on
the back of the left atrium and ends in the coronary sinus near its left
extremity; it is continuous above with the ligament of the left vena cava
(lig. ven cav sinistr vestigial fold of Marshall), and the two structures
form the remnant of the left Cuvierian duct.
The following cardiac veins do not end in the coronary sinus:
The anterior cardiac veins, comprising three or four small vessels which
collect blood from the front of the right ventricle and open into the right
atrium; the right marginal vein frequently opens into the right atrium, and
is therefore sometimes regarded as belonging to this group;
The smallest cardiac veins (veins of Thebesius), consisting of a number of
minute veins which arise in the muscular wall of the heart; the majority
open into the atria, but a few end in the ventricles.
(Q.90)

Centroacinar cells are characteristically seen in?


(a)

Salivary gland

(b)

Von Ebner gland

(c)

Pyloric glands

(d)

Exocrine pancreas

Your Response :
Correct Answer :
Exp:

d
Exocrine pancreas (Ref. Grays Anatomy of the Human Body; Fig.
1105)
Centroacinar cellsare spindle-shaped cells in the exocrinepancreas.
Centroacinar cells are an extension of theintercalated ductcells into each
pancreaticacinus.The intercalated ducts take the bicarbonate tointralobular
ductswhich become lobular ducts. These lobular ducts finally converge to
form the main pancreatic duct
These cells are commonly known asduct cells, and secrete an
aqueousbicarbonatesolution under stimulation by the hormonesecretin.
They also secretemucin.
In structure, the pancreas resembles the salivary glands.
It differs from them, however, in certain particulars, and is looser and
softer in its texture.
It is not enclosed in a distinct capsule, but is surrounded by areolar tissue,
which dips into its interior, and connects together the various lobules of
which it is composed.
Each lobule, like the lobules of the salivary glands, consists of one of the
ultimate ramifications of the main duct, ending in a number of cecal
pouches or alveoli, which are tubular and somewhat convoluted.
The minute ducts connected with the alveoli are narrow and lined with
flattened cells.
The alveoli are almost completely filled with secreting cells, so that
scarcely any lumen is visible.
In some animals spindle-shaped cells occupy the center of the alveolus
and are known as the centroacinar cells of Langerh These are
prolongations of the terminal ducts.
The true secreting cells which line the wall of the alveolus are very
characteristic. They are columnar in shape and present two zones: an outer
one, clear and finely striated next the basement membrane, and an inner
granular one next the lumen.
The connective tissue between the alveoli presents in certain parts
collections of cells, which are termed interalveolar cell islets (islands of

Langerhans).
The cells of these stain lightly with hematoxylin or carmine, and are more
or less polyhedral in shape, forming a net-work in which ramify many
capillaries.
There are two main types of cell in the islets, distinguished as A-cells and
B-cells according to the special staining reactions of the granules they
contain.
The cell islets have been supposed to produce the internal secretion of the
pancreas which is necessary for carbohydrate metabolism, but numerous
researches have so far failed to elucidate their real function.
The walls of the pancreatic duct are thin, consisting of two coats, an
external fibrous and an internal mucous; the latter is smooth, and
furnished near its termination with a few scattered follicles.
(Q.91)

If a patient has a drooping right eyelid and a dilated right pupil, which of the
following neural structures is most likely affected?
(a)

Cervical sympathetic chain

(b)
(c)

Facial nerve
Oculomotor nerve

(d)

Superior cervical ganglion

Your Response :
Correct Answer :
Exp:

c
Reference: Keith L. Moore, Clinically oriented Anatomy, 6th edition
The oculomotor nerve innervates the levator palpebrae superioris, which
elevates the eyelid. This nerve also innervates the inferior oblique
muscles, as well as the superior, inferior, and medial rectus muscles. The
oculomotor nerve also contains preganglionic parasympathetic fibers that
synapse, in the ciliary ganglion, on postganglionic parasympathetic nerve
fibers that innervate the sphincter pupillae muscle, which constricts the
pupil. A lesion of the oculomotor nerve may therefore result in both
drooping of the eyelid (ptosis) and dilation of the pupil (mydriasis).
The cervical sympathetic chain contains preganglionic sympathetic nerve
fibers, arising from the upper thoracic spinal cord, which ascend to the
cervical sympathetic ganglia. A lesion of these nerves may result in
Horner's syndrome, which includes a ptosis and miosis (pupillary
constriction) and, often, anhidrosis (lack of sweating).

The facial nerve innervates the muscles of facial expression, including the
orbicularis oculi muscle. A lesion of this nerve may therefore result in the
inability to close the eye.
The superior cervical ganglion contains the cell bodies of postganglionic
sympathetic nerves that innervate structures in the head. A lesion of this
structure will cause Horner's syndrome.
The trigeminal nerve provides sensory innervation to much of the head. A
lesion of this nerve may interfere with the corneal blink reflex.
(Q.92)

Which of the following cranial nerves supplies superior oblique muscle?


(a)

Abducens

(b)

Oculomotor

(c)

Trochlear

(d)

Trigeminal

Your Response :
Correct Answer :
Exp:

c
Trochler nerve
Trochlear nerve is the only cranial nerve, which emerges on the dorsal
aspect of Brain stem from superior medullar velum near frenulum veli just
below the inferior colliculus.
It supplies the superior oblique muscle of eyeball & when it is damaged
diplopia occurs on locking downward
LR6- SO4 (Lateral rectus is supplied by 6th cranial nerve & superior
oblique by 4th cranial nerve).

(Q.93)

Cervix develops from?


(a)

Wolffian duct

(b)

Mullerian duct

(c)

Sinovaginal bulb

(d)

Metanephros

Your Response :
Correct Answer :

Exp:

Mullerian duct

(Ref. Langmans embryology 11th ed. 345)

DEVELOPMENT OF UTERUS
In the absence of MIS production by Sertoli cells, the paramesonephric
(mullerian) ducts are stimulated by estrogens to form the uterine tubes,
uterus, cervix, and upper vagina. The fornices and the upper portion of the
vagina are formed by vacuolization of the paramesonephric tissue, and the
lower protion of the vagina is formed by vacuolization of the sinovaginal
bulbs.
(Q.94)

The least common type of female pelvis is?


(a)

Gynecoid

(b)

Android

(c)

Anthropoid

(d)

Platypelloid

Your Response :
Correct Answer :
Exp:

d
Platypelloid Reference: Grays Anatomy, 39th edition
The Female Pelvis
Deformities of the pelvis may be responsible for dystocia (difficult labor).
A contracted pelvis may obstruct the normal passage of the fetus. It may
be indirectly responsible for dystocia by causing conditions such as
malpresentation or malposition of the fetus, premature rupture of the fetal
membranes, and uterine inertia.
The cause of pelvic deformities may be congenital (rare) or acquired from
disease, poor posture, or fractures caused by injury. Pelvic deformities are
more common in women who have grown up in a poor environment and
are undernourished. It is probable that these women suffered in their
youth from minor degrees of rickets.
In 1933, Caldwell and Moloy classified pelves into four groups: gynecoid,
android, anthropoid, and platypelloid (Fig. 6-5C). The gynecoid type,
present in about 41% of women, is the typical female pelvis, which was
previously described.
The android type, present in about 33% of white females and 16% of
black females, is the male or funnel-shaped pelvis with a contracted
outlet.

The anthropoid type, present in about 24% of white females and 41% of
black females, is long, narrow, and oval shaped.
The platypelloid type, present in only about 2% of women, is a wide
pelvis flattened at the brim, with the promontory of the sacrum pushed
forward
(Q.95)

Respiratory compromise is typically seen in ?


(a)

Incomplete unilateral recurrent laryngeal nerve palsy

(b)

Complete unilateral recurrent laryngeal nerve palsy

(c)

Bilateral recurrent laryngeal nerve palsy

(d)

Superior and Recurrent laryngeal nerve palsy

Your Response :
Correct Answer :

Exp: Bilateral recurrent laryngeal nerve palsy


PARALYSED EXTENT OF
POSITION VOICE RESPIRATION SWALLOWING
LARYNGEAL INVOLVEMENT OF
NERVE
VOCAL
CORD

(Q.96)

Incomplete
Unilateral
Recurrent L
N

Median

Normal Normal

Normal

Complete
Unilateral
Recurrent L
N

Paramedian Hoarse Normal


or
Normal

Normal

Bilateral
Recurrent L
N

Median or Normal Dyspnoea


Paramedian

Normal

Superior with Unilateral or


Recurrent L bilateral
N

Cadaveric, Feeble Normal


Slack,
or
Wavy
Rough

Inhalation into
larynx

All of the following structures are present in lateral wall of Cavernous sinus
EXCEPT:
(a)

6th Cranial Nerve

(b)

3rd Cranial Nerve

(c)

4th Cranial Nerve

(d)

5th Cranial nerve

Your Response :
Correct Answer :
Exp:

a
6th Cranial Nerve
Reference: Grays Anatomy, 39th edition
Content of cavernous sign
The sinus may be joined by severalanastomosesacross the midline. The
cavernous sinus receives blood via the ophthalmic vein through the
superior orbital fissure and from superficial cortical veins, and is
connected to the basilar plexus of veins posteriorly. Thecarotid artery
(carotid siphon), and cranial nerves III, IV, V1, V2and VI all pass through
this blood filled space. Infection from the face may reach the cavernous
sinus through its many anastomotic connections, with severe
consequences. The cavernous sinus drains by two channels, the superior
and inferiorpetrosal sinuses, ultimately into the internal jugular vein.
Each cavernous sinus (one for each hemisphere of the brain) contains the
following:
Vertically, from superior to inferior (within the lateral wall of the sinus)
Oculomotor Nerve(CN III)
Trochlear Nerve(CN IV)
Abducens Nerve(CN VI)
Ophthalmic Nerve, the V1branch of thetrigeminal nerve(CN V)
Maxillary Nerve, the V2branch of CN V
These nerves, with the exception of CN V2, pass through the cavernous
sinus to enter the orbital apex through the superior orbital fissure. The
maxillary nerve, division V2of the trigeminal nerve travels through the
lower portion of the sinus and exits via the foramen rotundum.
Horizontally, from medial to lateral
Internal Carotid Artery(andsympathetic plexus). See alsocavernous part of
internal carotid artery.
The optic nerve lies just above and outside the cavernous sinus, superior

and lateral to the pituitary gland on each side, and enters the orbital apex
via the optic canal.
(Q.97)

All of the following are Secondary cartilaginous joints except ?


(a)

Symphysis pubis

(b)

Manubrio-sternal joint

(c)

Sacro-coccygeal joint

(d)

Middle tibio-fibular joint

Your Response :
Correct Answer :
Exp:

d
Middle tibio-fibular joint
TYPES OF JOINTS IN BODY : HEAD-TO-TOE :
JOINT

TYPE

Temporomandibular joints

Condylar (Bicondylar) synovial


joint

Cricothyroid and cricoareytenoid


joint

Synovial joint

Atlanto-occipital joint, Wrist joint


and Metacarpo-phalangeal joint

Ellipsoid joint

Sternoclavicular joint

Saddle (sellar) joint

Ear ossicles

Saddle joint

Melleo-Incal joint

Ball and socket joint

Incudo-stapedial joint

Syndesmosis

Stapes footplate
Joint between ala of vomer and
Rostrum of sphenoid

Schindylesis (Wedge and groove


suture)

Costo-vertebral joint

Plane synovial joint

Costo-transverse joint

Synovial joint

1st chondrosternal joint,

Primary cartilaginous joint /


Synchondrosis / Hyaline

All Costochondral joints,

cartilaginous joint.

Spheno-occipital joint
Joint between epiphysis and
diaphysis of growing bone
2nd 7th chondrosternal joint

Synovial joint

5th 9th costal cartilage articulation

Synovial joint

10th costal cartilage is united to 9th


costal cartilage by

Fibrous tissue.

Superior and inferior Radio-ulnar


joints

Pivot (Trochoid) joint

Median Atlanto-axial joint


Middle radio-ulnar joint

Syndesmosis

Sterno-clavicular joint

Saddle joint

Elbow and Ankle joint

Hinge synovial joint

Hip joint

Ball and socket synovial joint

Knee joint

Compound synovial joint


2 condylar joints between medial
and lateral condyles of femur and
tibia.
1 saddle joint between femur and
patella.

Superior Tibio-fibular joint

Plane synovial joint

Middle Tibio-fibular joint

Fibrous joint

Inferior Tibio-fibular joint

Syndesmosis`

Talo-calcaneal / Subtalar joint

Plane synovial joint

Calcaneo-cuboid joint

Saddle joint

Talo-calcaneo-navicular joint

Ball and socket synovial joint

Other intertarsal joints

Plane synovial joint

Symphysis pubis

Secondary cartilaginous joint /


Symphyses / Fibrocartilagenous

Sacro-coccygeal joint

joint

Manubriosternal joint
Intervertebral joints
(Q.98)

Germ cell develops from:


(a)

Endoderm

(b)

Mesoderm

(c)

Ectoderm

(d)

Ectodermal cleft

Your Response :
Correct Answer :
Exp:

c
Ectoderm
EndodermInnermost layer of the cells derived from the inner cell mass
of the blastocyst; it gives rise to lungs, other respiratory structures, and
digestive organs, or generally "the gut".
MesodermMiddle layer of a group of cells derived from the inner cell
mass of the blastocyst; it gives rise to bone, muscle, connective tissue,
kidneys, and related structures.
EctodermOutermost germ layer of cells derived from the inner cell
mass of the blastocyst; gives rise to the nervous system, sensory organs,
skin, and related structures.
Embryonic germ cellsPluripotent stem cells that are derived from early
germ cells (those that would become sperm and eggs). Embryonic germ
cells (EG cells) are thought to have properties similar to embryonic stem
cells.
Embryonic stem cellsPrimitive (undifferentiated) cells derived from a
5-day preimplantation embryo that have the potential to become a wide
variety of specialized cell types.

(Q.99)

Inferior pancreaticoduodenal artery is a branch of?


(a)

SMA

(b)

IMA

(c)

Coeliac artery

(d)

Aorta

Your Response :
Correct Answer :
Exp:

a
SMA Reference: Keith L. Moore, Clinically oriented Anatomy, 6th
edition
The inferior pancreaticoduodenal artery is a branch of the superior
mesenteric artery or from its first intestinal branch, opposite the upper
border of the inferior part of the duodenum.
It courses to the right between the head of the pancreas and duodenum,
and then ascends to anastomose with the anterior and posterior superior
pancreaticoduodenal artery.
It distributes branches to the head of the pancreas and to the descending
and inferior parts of the duodenum.

(Q.100)

Glossopharyngeal nerve supplies which muscle?


(a)

Stylopharyngeus

(b)

Posterior belly of digastric

(c)

Anterior belly of digastric

(d)

Stylohyoid

Your Response :
Correct Answer :
Exp:

a
Stylopharyngeus (Ref. ENT by PL Dhingra, 6th/pg. 359)
Third arch derivatives are:
- Cartilage: Greater horn of hyoid &
Muscle: Stylopharyngeus
Nerve of third arch: Glossopharyngeal nerve.
PALATE
All of the muscles of the palate, except for the tensor veli palatini, are
innervated by the vagus nerve.
The tensor veli palatini is innervated by the maxillary division of the
trigeminal nerve.

PHARYNX
All the muscles of the pharynx, except for the stylopharyngeus, are
innervated by the vagus nerve.
The stylopharyngeus muscle is innervated by the glossopharyngeal
nerve.Q
LARYNX
All the muscles of the larynx, except for the cricothyroid muscle, are
innervated by the recurrent laryngeal branch of the vagus nerve.
The cricothyroid muscle is innervated by the external laryngeal branch of
the vagus nerve
(Q.101)

True about Omphalocele is all except ?


(a)

Umbilical cord is attached at apex

(b)

Bowel is usually normal

(c)

Defect is not covered with membrane

(d)

Significant incidence of associated anomalies

Your Response :
Correct Answer :
Exp:

c
Defect is not covered with membrane
GASTROSCHISIS

OMPHALOCELE

Incidence 1 in 6000, Incidence is


increasing.

1 in 4000, Incidence is static

Defect is open

Defect is covered by membrane

Defect size 2 to 5 cm

Defect size 2 to 15cm

Umbilical cord is to left of defect Umbilical cord is at the apex of


at its normal attachment site.
membrane
Bowel inflamed

Bowel normal.

Alimentation is delayed.

Alimentation is normal.

Associated anomalies 10 %.

Associated anomalies 60%.

Important Associations :

Cardiac anomalies,

Intestinal atresia.

(Q.102)

Chromosomal anomalies, Downs


syndrome.

The abductor of vocal cord is?


(a)

Posterior cricoarytenoid

(b)

Lateral cricoarytenoid

(c)

Thyroarytenoid

(d)

Cricoarytenoid

Your Response :
Correct Answer :
Exp:

a
Posterior cricoarytenoid (Ref. BDC vol.Exp. 4th/ pg. 243)
The only abductor of vocal cord is Posterior cricoarytenoid. Q
LARYNX : INTRINSIC MUSCLES

(Q.103)

Laryngeal muscle

Action

Aryepiglottic Muscle

Draws the epiglottis posteriorly and


downward during swallowing

Oblique Arytenoid

Draws arytenoid cartilages together


adducting vocal folds

Transverse Arytenoid

Draws arytenoid cartilages together,


adducting vocal folds

Posterior Cricoarytenoid
Muscles

Opens space between the vocal cords


(abduct) by rotating the arytenoids laterally
(abduct), causing the vocal cords to separate
opening the rima glottidis.

Thyroarytenoideus

Draws arytenoid cartilage forward relaxing


and adducting the vocal folds

Vocalis muscle

Relaxes segments of the vocal ligament


thereby adjusting voice pitch.

False statement pertaining to Pancreatic embryology is ?


(a)

The main pancreatic duct is derived from the entire ventral pancreatic
duct and the distal part of the dorsal pancreatic duct

(b)

The pancreatic islets arise as small buds from the developing ducts

(c)

The inferior part of the head and the uncinate process of the pancreas
are formed from the dorsal pancreatic bud

(d)

Congenital fibrocystic disease in the pancreas is caused by an


abnormality in the secretion of mucus.

Your Response :
Correct Answer :
Exp:

c
The inferior part of the head and the uncinate process of the
pancreas are formed from the dorsal pancreatic bud
Development of the Pancreas
The pancreas develops from a dorsal and ventral bud of entodermal cells
that arise from the foregut. The dorsal bud originates a short distance
above the ventral bud and grows into the dorsal mesentery. The ventral
bud arises in common with the hepatic bud, close to the junction of the
foregut with the midgut. A canalized duct system now develops in each
bud. The rotation of the stomach and duodenum, together with the rapid
growth of the left side of the duodenum, results in the ventral bud's
coming into contact with the dorsal bud, and fusion occurs.
Fusion also occurs between the ducts, so that the main pancreatic duct is
derived from the entire ventral pancreatic duct and the distal part of the
dorsal pancreatic duct. The main pancreatic duct joins the bile duct and
enters the second part of the duodenum. The proximal part of the dorsal
pancreatic duct may persist as an accessory duct, which may or may not
open into the duodenum about Exp.75 in. (2 cm) above the opening of
the main duct.
Continued growth of the entodermal cells of the now-fused ventral and
dorsal pancreatic buds extends into the surrounding mesenchyme as
columns of cells. These columns give off side branches, which later
become canalized to form collecting ducts. Secretory acini appear at the
ends of the ducts.
The pancreatic islets arise as small buds from the developing ducts.
Later, these cells sever their connection with the duct system and form
isolated groups of cells that start to secrete insulin and glucagon at about
the fifth month.
The inferior part of the head and the uncinate process of the pancreas are
formed from the ventral pancreatic bud; the superior part of the head, the
neck, the body, and the tail of the pancreas are formed from the dorsal
pancreatic bud

Entrance of the Bile Duct and Pancreatic Duct into the Duodenum
As seen from development, the bile duct and the main pancreatic duct are
joined to one another. They pass obliquely through the wall of the second
part of the duodenum to open on the summit of the major duodenal
papilla, which is surrounded by the sphincter of Oddi. In some
individuals, they pass separately through the duodenal wall, although in
close contact, and open separately on the summit of the duodenal papilla.
In other individuals, the two ducts join and form a common dilatation,
the hepatopancreatic ampulla (ampulla of Vater). This opens on the
summit of the duodenal papilla.
Anular Pancreas
In annular pancreas, the ventral pancreatic bud becomes fixed so that,
when the stomach and duodenum rotate, the ventral bud is pulled around
the right side of the duodenum to fuse with the dorsal bud of the
pancreas, thus encircling the duodenum. This may cause obstruction of
the duodenum, and vomiting may start a few hours after birth. Early
surgical relief of the obstruction is necessary.
Ectopic Pancreas
Ectopic pancreatic tissue may be found in the submucosa of the stomach,
duodenum, small intestine (including Meckel's diverticulum), and
gallbladder, and in the spleen. It is important in that it may protrude into
the lumen of the gut and be responsible for causing intussusception.
Congenital Fibrocystic Disease
Basically, congenital fibrocystic disease in the pancreas is caused by an
abnormality in the secretion of mucus. The mucus produced is
excessively viscid and obstructs the pancreatic duct, which leads to
pancreatitis with subsequent fibrosis. The condition also involves the
lungs, kidneys, and liver
(Q.104)

All are true regarding the pudendal nerve, except:


(a)

Sensory and motor

(b)

Derived from S2, 3, 4

(c)

Entersperineum through lesser sciatic foramen

(d)

Main nerve supply of pelvic organs

Your Response :
Correct Answer :

Exp:

Ref: Keith L. Moore, Clinically oriented Anatomy, 6th edition


Main nerve supply of pelvic organs
-Pudendal nerve is a somatic nerve and is not the main supply of pelvic
viscera (D).
-It is the nerve of perineum carrying root value S-2, 3 & 4 (B) and is a
mixed nerve with both the sensory and motor (A) components.
-Pelvic viscera are mainly supplied by the autonomic nervous system:
sympathetic (T-10, 11, 12; L-1, 2) and parasympathetic (S-2, 3, 4).
Pudendal nerve passes through both the sciatic notches and the
information given in other choices is perfectly fine, in anatomical
position.

(Q.105)

Supination Pronation does not occur at:


(a)

Radiocarpal joint.

(b)

Superior Radioulnar joint.

(c)

Middle Radioulnar joint.

(d)

Inferior Radioulnar joint.

Your Response :
Correct Answer :
Exp:

a
Radiocarpal joint (Ref. BDC I 3rd ed. 142; Snells anatomy 467)
The wrist joints are grouped transversely into the radiocarpal and
intercarpal joints. The radiocarpal joint is composed of the distal articular
surface of the radius and the proximal carpal row: scaphoid, lunate, and
triquetrum. The intercarpal joint consists of the proximal row and its
articulation with the distal carpal row, composed of the scaphoid
trapezium, trapezoid, capitate, and hamate. Supination Pronation does
not occur at Radiocarpal joint.

(Q.106)

Claw hand is seen in palsy of:


(a)

Radial nerve

(b)

Ulnar nerve

(c)

Median nerve

(d)

Post interosseous nerve

Your Response :
Correct Answer :
Exp:

b
Ulnar nerve
-------------------------------------------------------------------------------------------------------------------------------------------------------S.

Nerve

Muscle Group

Functional Deficit

-------------------------------------------------------------------------------------------------------------------------------------------------------1

Long thoracic

Serratus anterior

2 Suprascapular
initiating arm

Supraspinatus and

Infraspinatus
3 Axillary
abduct arm

Winging scapula
Difficulty in

abduction

Deltoid and Teres minor

4 Radial
extension, loss of

Extensors of forearms,

wrist proximal phalanges

Inability to fully

Loss of arm

foreams extension,

supination,
and thumb

abduction, loss of wrist extension


(wrist drop), loss of proximal
phalangeal extension and
thumb extension.

5 Musculoflexion,

Flexors of arm and

cutaneous
6 Median
pronator, and

forearm

Weak arm and forearm

weak forearm supination.

Wrist and hand flexors

Paralysis of flexor,

thenar muscles, inability to fully


flex the index and middle fingers
(sign of benediction), Pointing
index finger, Labourers nerve.

7 Ulnar
the distal and

Wrist and hand flexors

Inability to extend

middle phalanges (clawhand);


loss of thumb abduction,
Froment sign positive.
-------------------------------------------------------------------------------------------------------------------------------------------------------(Q.107)

True about posterior cruciate ligament:


(a)

Intrasynovial ligament

(b)

Attached to lateral epicondyle of tibia.

(c)

Prevents posterior displacement of tibia

(d)

Buckling of the posterior cruciate ligament is an indirect sign of


meniscal tear.

Your Response :
Correct Answer :
Exp:

c
Prevents post displacement of tibia. (Ref. BDC II 3rd ed. 128)
There are also two intra-articular, extrasynovial ligaments, the anterior
and posterior cruciate ligaments (ACL and PCL). The normal anterior
cruciate ligament is oriented obliquely in the lateral aspect of the
intercondylar notch, at an angle approximately that of the intercondylar
roof. The PCL is seen as a smoothly curved arc-like band extending
from the internal aspect of the medial femoral condyle to the posterior
aspect of the tibia. Buckling of the posterior cruciate ligament is an
indirect sign of ACL tear.

(Q.108)

Structure passing from thorax to abdomen behind the attachment of diaphragm are
allEXCEPT:
(a)

Thoracic duct

(b)

Aorta

(c)

Azygous vein

(d)

Greater Splanchnic nerve

Your Response :
Correct Answer :
Exp:

d
Greater Splanchnic nerve (Reference: Grays Anatomy, 39th edition)
The esophageal opening is within the crura just below their insertion in
the posterior aspect of the central tendon opposite the T10 vertebral
body. Through the esophageal hiatus run the distal esophagus, the
anterior and posterior branches of the vagus nerve, and esophageal
branches of the left gastric artery.
As the esophagus exits into the peritoneal cavity, the aorta, azygous vein,
hemiazygous vein, and thoracic duct all remain retrocrural and are
bordered laterally by mediastinal parietal pleura.

(Q.109)

Not true regarding trigone of bladder:


(a)

Lined by transitional epithelium.

(b)

Mucosa is loosely attached to muscular coat.

(c)

It develops by investing of mesonephric duct & urogenital sinus.

(d)

It mucosa is smooth.

Your Response :
Correct Answer :
Exp:

b
Mucosa is loosely attached to muscular coat.
(Ref. BDC II 3rd ed. 306 and Textbook of surgery by S. Das -1200)
The posterior portion of the bladder floor contains the trigone, the
anteroinferior vertex of which is the internal urethral orifice and whose
lateral corners are the ureteral orifices. Walls of undistended bladder are
thrown into folds because mucosa is not firmly attached to submucosa
except at the Trigone, which is smooth triangular area of mucous
membrane covering base of bladder, bordered by internal urethral orifice
below and the two ureteric orifices above.

(Q.110)

All the following structures crosses mid saggital plane EXCEPT:


(a)

Left renal vein

(b)

Left brachiocephalic vein

(c)

Left Gonadal vein

(d)

Hemiazygos vein

Your Response :
Correct Answer :
Exp:

c
Left Gonadal vein
The left renal vein courses between the aorta and superior mesenteric
artery to join the inferior vena cava. The left gonadal vein will drain into
to left renal vein. The right gonadal vein drains directly into the inferior
venal cava.
Because the inferior vena cava is on the right half of the body, the left
renal vein is generally the longer of the two.
Because the inferior vena cava is not laterally symmetrical, the left renal
vein often receives the following veins:
Left suprarenal vein
Left lumbar vein
Left gonadal vein (left testicular vein in males, left ovarian vein in
females)
This is in contrast to the right side of the body, where these veins drain
directly into the IVC.

(Q.111)

Posterior communicating artery is branch of:


(a)

Internal carotid artery

(b)

Middle cerebral artery

(c)

Basilar artery

(d)

Posterior cerebral artery

Your Response :
Correct Answer :
Exp:

a
Internal carotid artery
Posterior Communicating Artery
Second branch of supraclinoid internal carotid.
Connects anterior circulation (carotid system) with posterior circulation
(vertebrobasilar system)

Supplies thalamus, hypothalamus, optic chiasm, and mamillary bodies.


Common site for aneurysms.
(Q.112)

Cervical vertebra can be easily differentiated from thoracic vertebra by:


(a)

Large vertebral body

(b)

Presence of Foramen transversarium

(c)

Presence of superior articular facet

(d)

Wide Vertebral canal

Your Response :
Correct Answer :
Exp:

b
Presence of Foramen transversorium
Vertebral bodies in the cervical spine (neck)The cervical spine has seven
vertebral bodies (segments). The top two segments are unique:
The first cervical segment (called the atlas) is a ring that does not have a
vertebral body. It is attached to the second vertebral body (the axis),
which acts as a post that the first vertebral ring rotates around. Most of
the rotation in the neck is located in these top two segments.
Like the rest of the spine, the next five vertebral segments have three
joints at each segment, including one disc in the front and paired facet
joints in the back.
Unlike the rest of the spine, the segments in the cervical spine contain
openings in each vertebral body for arteries to carry blood to the brain
(vertebral artery running through the transverse foramen).
Vertebral bodies in the thoracic spine (upper back)The thoracic spine has
twelve vertebral bodies. These structures have very little motion because
they are firmly attached to the ribs and sternum (breastbone). Because
there is little motion, this region of the spine is not usually a source of
back pain, although the junction between the spine and the ribs
(costovertebral junction) can be a source of pain.
Vertebral bodies in the lumbar spine (lower back)The lumbar spine has
five vertebral bodies that extend from the lower thoracic spine (upper
back) to the sacrum (bottom of the spine). The vertebral bodies of the
lower back are the largest of the spine because they bear the majority of
the bodys weight. The paired facet joints on the back of the vertebral
segments are aligned so that they allow flexion/extension but not a lot of

rotation.
(Q.113)

During chewing the major burden is over:


(a)

Maxillozygomatic arch

(b)

Pterygo zygomatic arch

(c)

Maxillopterygoid arch

(d)

Zygomaticotemporal arch

Your Response :
Correct Answer :
Exp:

c
Maxillopterygoid arch
Maxillopterygoid arch bears the maximum load during the process of
chewing.

(Q.114)

Branches of Supraclinoid Segment of Internal CarotidArtery:


(a)

Ophthalmic artery

(b)

Medullary branches

(c)

Branches to pituitary

(d)

Anterior cerebral artery

Your Response :
Correct Answer :
Exp:

a
Ophthalmic artery
ICA begins at the bifurcation of the Common Carotid Artery (level of
C4). Terminal branches: Anterior Cerebral and Middle Cerebral Arteries.
Four segments of ICA are:
1 Cervical,

2 Petrous,

3 Cavernous

4 Supraclinoid

Supraclinoid Segment of Internal Carotid Artery


Begins after penetration of dura, continues until bifurcation into Anterior
and Middle Cerebral Arteries. Its three Branches include:
Ophthalmic Artery,

Posterior Communicating Artery,


Anterior Choroidal Artery
(Q.115)

Classically the Papez circuit of recent memory is related to:


(a)

Sub pulvinar nucleus

(b)

Anterior thalamic nucleus

(c)

Ventromedial nucleus

(d)

Pulvinar nucleus

Your Response :
Correct Answer :
Exp:

b
Anterior thalamic nucleus
Papez circuit
Described by James Papez in 1937, the Papez circuit of the brain is one
of the major pathways of the limbic system and is chiefly involved in the
cortical control of emotion.
It plays a role in storing memory.
The initial pathway was described as follows:
Hippocampus fornix mammillary bodies
Mammillary bodies mammillothalamic tract anterior thalamic
nucleus
Anterior thalamic nucleus genu of the internal capsule cingulate
gyrus
Cingulate gyrus cingulum parahippocampal gyrus
Parahippocampal gyrus entorhinal cortex perforant pathway
hippocampus
Since then, new findings in neuroanatomy and brain function have
elucidated a larger circuit that also includes the prefrontal cortex (PFC),
amygdala, and septum among other areas. The PFC and amygdala are
key components in this larger loop.

(Q.116)

Meralgia paresthetica involves:

(a)

Lateral cutaneous nerve of thigh.

(b)

Medial cutaneous nerve of thigh.

(c)

Genitofemoral nerve

(d)

Common femoral nerve

Your Response :
Correct Answer :
Exp:

a
Lateral cutaneous nerve of thigh.
Meralgia paresthetica
It is a painful mononeuropathy of the lateral femoral cutaneous nerve
(LFCN) i.e. lateral cutaneous nerve of thigh.
It occurs commonly due to focal entrapment of this nerve as it passes
through the inguinal ligament. Rarely, it has other etiologies such as
direct trauma, stretch injury, or ischemia. It typically occurs in isolation.
The LFCN is responsible for the sensation of the anterolateral thigh. It is
a purely sensory nerve and has no motor component.
The LFCN originates directly from the lumbar plexus and has root
innervation from L2-Exp.
The nerve runs through the pelvis along the lateral border of the psoas
muscle to the lateral part of the inguinal ligament. Here, it passes to the
thigh through a tunnel formed by the lateral attachment of the inguinal
ligament and the anterior superior iliac spine. This is the most common
site of entrapment.

(Q.117)

Morgagni hernia common through:


(a)

Left anterior aspect of the diaphragm

(b)

Right anterior aspect of the diaphragm

(c)

Right posterior aspect of the diaphragm

(d)

Left posterior aspect of the diaphragm

Your Response :
Correct Answer :
Exp:

b
Right anterior aspect of the diaphragm

(Ref. Textbook of surgery

by S.Das 768)
Foramen of Morgagni hernia is caused by an anterior defect in the
diaphragm between its attachment to the sternum and ribs. The
abnormality is most commonly right-sided and nearly always is
contained by a hernia sac. It is caused by failure of the fibro tendinous
elements of the sternal part of the diaphragm to fuse with the costal part.
Herniation occurs through the foramen of Morgagni, which normally
contains fat, the superior epigastric arteries, and some lymphatics. The
hernia is surrounded by a sac of pleura and peritoneum and usually
contains omentum and transverse colon and sometimes stomach, small
bowel, and portions of the liver. Morgagni hernia usually appears in
adults and often is associated with obesity, trauma, and other causes of
increased intra-abdominal pressure. Most are right sided as the heart and
pericardium cover left-sided defects. These defects tend to appear early
in childhood and contain liver. Chest radiographs demonstrate a softtissue mass in the right cardiophrenic angle.
(Q.118)

Which of the following is not true about DIPLOIC VEINS?


(a)

Are present in cranial bones

(b)

Develop during the eighth week of intrauterine life

(c)

They are devoid of valves

(d)

They are thin walled with endothelial lining and elastic tissue only

Your Response :
Correct Answer :
Exp:

b
Develop during the eighth week of intrauterine life
(Ref. Grays anatomy of the Human body 564)
The diploic veins occupy the channels in the diploes of cranial bones.
They are large and exhibit pouch-like dilatation: their walls are thin, and
formed of endothelium resting up on a layer of elastic tissue.
Primary ossification centers are present in the long bones and skull by
the 12th week. Also by the 12th week, external genitalia develop to such
a degree that the sex of the fetus can be determined by external
examination (ultrasound). During the 6th week intestinal loops cause a
large swelling (herniation) in the umbilical cord, but by the 12th week
the loops withdraw into the abdominal cavity.

(Q.119)

All the following bones contribute to the formation of Nasal septum EXCEPT:
(a)

Sphenoid

(b)

Lacrimal

(c)

Palatine

(d)

Ethmoid

Your Response :
Correct Answer :
Exp:

b
Lacrimal
Structures forming nasal septum:
Bones
o Vomer

o Perpendicular bone of ethmoid

o Nasal process of frontal bone (small contribution)


sphenoid (small contribution)

o Rostrum of

Cartilage
o Quadrilateral cartilage
(Q.120)

o Lower lateral nasal cartilage

Which of the following the type of joint between ear ossicles?


(a)

Primary cartilaginous

(b)

Secondary cartilaginous

(c)

Synovial

(d)

Fibrous

Your Response :
Correct Answer :
Exp:

c
Synovial
Joints
A. Synovial
Ball and socket o Shoulder,
o Incudomalleolar

o Hip,

o Calcaneo-cuboid,

Condylar

o Knee,

Ellipsoid
Wrist,
Hinge
Pivot

o TM joint

o Atlanto-occipital,

o Interphalangeal,
o Atlanto-axial,

Saddle

o Thumb,

o Metacarpophalangeal,

o Elbow,

o Ankle

o Superior and inferior radioulnar


o sternoclavicular,

B. Cartilaginous
Primary (Synchondrosis/hyaline cartilageous)
a. Joint between epiphysis and diphysis of long bones
b. Spheno-occipital

c. First chondrosternal

d. ostochondral

Secondary(symphysis/fibro cartilaginous)
a. Symphysis pubis

b. Manubriosternal

c. Intervertebral

C. Fibrous
Skull sutures
Syndesmosis
i. Foot plate of stapes with oval window,
ii. Inferior tibiofibular joint
Gomphosis/ peg and socket

(Q.121)

i. Tooth

Which of the following is the FINA.L COMMON PA.THWA.Y for horizontal gaze?
(a)

Oculomotor

(b)

Abducens

(c)

Trochlear

(d)

Vestibular

Your Response :
Correct Answer :
Exp:

b
Abducens (Ref. Harrisons principles of internal medicine 16th ed.

Chapter 25)
Horizontal Gaze
Descending cortical inputs mediating horizontal gaze ultimately
converge at the level of the pons.
Neurons in the paramedian pontine reticular formation are responsible
for controlling conjugate gaze toward the same side.
They project directly to the ipsilateral abducens nucleus.
A lesion of either the paramedian pontine reticular formation or the
abducens nucleus causes an ipsilateral conjugate gaze palsy.
Lesions at either locus produce nearly identical clinical syndromes, with
the following exception: vestibular stimulation (oculocephalic maneuver
or caloric irrigation) will succeed in driving the eyes conjugately to the
side in a patient with a lesion of the paramedian pontine reticular
formation, but not in a patient with a lesion of the abducens nucleus.
(Q.122)

Which of the following cranial nerve nucleus is NOT general somatic efferent?
(a)

Oculomotor

(b)

Trochlear

(c)

Facial

(d)

Abducens

Your Response :
Correct Answer :
Exp:

c
facial
COLUMNS OF CRANIAL NERVE MOTOR NUCLEI
General Somatic efferent
Abducent nucleus
Hypoglossal nucleus
Oculomotor nucleus
Trochlear nucleus
Special visceral efferent

Motor nucleus of trigeminal


Nucleus ambigus
Facial nerve nucleus
General visceral efferent
Superior salivatory nucleus
Lacrimatory nucleus
Inferior salivatory nucleus
Dorsal nucleus of Vagus
Edinger Westphal nucleus
Visceral afferent
Nucleus tractus solitorius
General somatic afferent
Main sensory, spinal nucleus and mesencephalic nucleus of trigeminal
Special somatic afferent
a. Chochlear and vestibular nuclei
(Q.123)

Which of the following is the nucleus of trigeminal nerve where the afferentarch
(impulses) for masseter reflex are carried?
(a)

Spinal nucleus

(b)

Mesencephalic nucleus

(c)

Sensory nucleus

(d)

Motor nucleus

Your Response :
Correct Answer :
Exp:

b
Mesencephalic nucleus
TRIGEMINAL NERVE NUCLEI
Motor Nucleus
The motor nucleus of CN V is located in the pons just medial to the main
sensory nucleus of the trigeminal and adjacent to the point of exit or

entry of the trigeminal nerve fibers. These motor fibers supply the
muscles of mastication (masseter, temporalis, and medial and lateral
pterygoid;
Sensory Nucleus
The main sensory nucleus is located just lateral to the motor nucleus.
The main sensory nucleus receives tactile and pressure sensations from
the face, scalp, oral cavity, nasal cavity, and dura.
Spinal Trigeminal Nucleus
The spinal trigeminal nucleus is a caudal continuation of the main
sensory nucleus, extending from the mid pons through the medulla to the
cervical cord. Central processes from cells in the trigeminal ganglion
conveying pain and temperature sensations from the face descend in the
spinal tract of V and synapse on cells in the spinal nucleus.
Mesencephalic Nucleus
The mesencephalic nucleus of CN V is located at the point of entry of the
fifth nerve and extends into the midbrain. It receives proprioceptive input
from joints, muscles of mastication, extraocular muscles, teeth, and the
periodontium. Some of these fibers synapse mono synaptically on the
motoneurons, forming the sensory limb of the jaw jerk reflex (Masseter
muscle).
There is only one area of the nervous system where the nuclei of primary
sensory neurons are located within the CNS rather than in outside
ganglia. This is the mesencephalic nucleus of CN5 which contains the
nuclei of CN5 proprioceptive fibers.
Locus ceruleus is a norepinephrine-containing brain stem nucleus that
lies near the mesencephalic nucleus of CNExp. It projects to widespread
areas of the brain and may have a general effect on modulating brain
function.
(Q.124)

Which of the following is concerned with pain and temperature?


(a)

Pyramidal tract

(b)

Anterior spinothalamic tract

(c)

Lateral spinothalamic tract

(d)

Dorsal spinocerebellar tract

Your Response :

Correct Answer :
Exp:

c
Lateral spinothalamic tract
SPINOTHALAMIC TRACT
Sensations of heat, cold, and pain are carried into the spinal cord mostly
by thin, unmyelinated sensory neurons.
Within the spinal cord, these neurons synapse with second-order
association neurons that cross over to the contralateral side and ascend to
the brain in the lateral spinothalamic tract.
Fibers that mediate touch and pressure ascend in the anterior
spinothalamic tract.
Fibers of both spinothalamic tracts synapse with third-order neurons in
the thalamus, which in turn project to the postcentral gyrus.
Notice that somatesthetic information is always carried to the postcentral
gyrus in third-order neurons. Also, because of crossing-over,
somatesthetic information from each side of the body is projected to the
postcentral gyrus of the contralateral cerebral hemisphere.

(Q.125)

Which of the following is not carried by posterior column tract?


(a)

Touch

(b)

Pain

(c)

Pressure

(d)

Vibration

Your Response :
Correct Answer :
Exp:

b
Pain
MEDIAL LEMINISCAL SYSTEM
The conduction pathways for the somatesthetic sensesa term that
includes sensations from cutaneous receptors and proprioceptors.
These pathways involve three orders of neurons in series. Sensory
information from proprioceptors and pressure receptors is first carried by
large, myelinated nerve fibers that ascend in the dorsal columns of the
spinal cord on the same (ipsilateral) side.
These fibers do not synapse until they reach the medulla oblongata of the

brain stem; hence, fibers that carry these sensations from the feet are
remarkably long. After the fibers synapse in the medulla with other
second-order sensory neurons, information in the latter neurons crosses
over to the contralateral side as it ascends via a fiber tract, called the
medial lemniscus, to the thalamus. Third-order sensory neurons in the
thalamus that receive this input in turn project to the postcentral gyrus.
(Q.126)

Facial colliculus located at in?


(a)

Pons

(b)

Medulla

(c)

Mid brain

(d)

Interpeduncular fossa

Your Response :
Correct Answer :
Exp:

Pons
Reference: Grays Anatomy, 39th edition
Facial colliculus is located in the floor of fourth ventricle, one on either side of
median sulcus, on the dorsal aspect of PONS. Deep to it lies the sixth nerve
nucleus, which is surrounded by internal genu of motor component of facial
nerve.
Nerve

Components

Function

Opening in
Skull

I.

Olfactory

Sensory

Smell

Openings in
cribriform
plate of
ethmoid

II.

Optic

Sensory

Vision

Optic canal

III.

Oculomotor

Motor

Lifts upper eyelid, Superior


turns eyeball
orbital
upward, downward, fissure
and medially;
constricts pupil;
accommodates eye

IV.

Trochlear

Motor

Assists in turning
eyeball downward

Superior
orbital

V.

and laterally

fissure

Superior
orbital
fissure

Trigeminal
Ophthalmic
division

Sensory

Cornea, skin of
forehead, scalp,
eyelids, and nose;
also mucous
membrane of
paranasal sinuses
and nasal cavity

Maxillary
division

Sensory

Skin of face over


Foramen
maxilla and the
rotundum
upper lip; teeth of
upper jaw; mucous
membrane of nose,
the maxillary air
sinus, and palate

Mandibular
division

Motor

Muscles of
Foramen
mastication,
ovale
mylohyoid, anterior
belly of digastric,
tensor veli palatini,
and tensor tympani

Sensory

Skin of cheek, skin


over mandible,
lower lip, and side
of head; teeth of
lower jaw and
temporomandibular
joint; mucous
membrane of mouth
and anterior two
thirds of tongue

VI.

Abducent

Motor

Lateral rectus
muscle: turns
eyeball laterally

Superior
orbital
fissure

VII.

Facial

Motor

Muscles of face,
cheek, and scalp;
stapedius muscle of
middle ear;
stylohyoid; and

Internal
acoustic
meatus,
facial canal,
stylomastoid

posterior belly of
digastric
Sensory

Taste from anterior


two thirds of
tongue, floor of
mouth, and palate

Secretomotor

Submandibular and
sublingual salivary
parasympathetic
glands, lacrimal
gland, and glands of
nose and palate

Vestibular

Sensory

Position and
movement of head

Cochlear

Sensory

Hearing

foramen

VIII. Vestibulocochlear

IX.

Glossopharyngeal Motor

Stylopharyngeus
muscle: assists
swallowing

Secretomotor Parotid salivary


parasympathetic gland

X.

Vagus

Internal
acoustic
meatus

Jugular
foramen

Sensory

General sensation
and taste from
posterior third of
tongue and pharynx;
carotid sinus and
carotid body

Motor

Constrictor muscles Jugular


of pharynx and
foramen
intrinsic muscles of
larynx; involuntary
muscle of trachea
and bronchi, heart,
alimentary tract
from pharynx to
splenic flexure of
colon; liver and

pancreas

XI.

XII.

(Q.127)

Sensory

Taste from
epiglottis and
vallecula and
afferent fibers from
structures named
above

Cranial root

Motor

Muscles of soft
Jugular
palate, pharynx, and foramen
larynx

Spinal root

Motor

Sternocleidomastoid
and trapezius
muscles

Hypoglossal

Motor

Muscles of tongue Hypoglossal


controlling its shape canal
and movement
(except
palatoglossus)

Accessory

Which of the following is not a feature of lower radial nerve injury?


(a)

Weakness of Brachioradialis

(b)

Inability to extend fingers

(c)

Paralysis of extensor carpi radialis brevis

(d)

Loss of sensations over dorsum of hand

Your Response :
Correct Answer :
Exp:

a
Weakness of Brachioradialis
Radial nerve is known as the great extensor nerve.
Provides innervation of the Brachioradialis, Extensors of the wrist and
fingers, Supinator, and Triceps.
Causes of injury
The radial nerve may be damaged by trauma or entrapped, especially

between the heads of muscles.


In the axilla:
With features of weak triceps,wrist dropand possibly also median and
ulnar nerve involvement. The most common cause is compression.
The radial nerve may be damaged in the axilla by fracture or dislocation
of the head of the humerus.
Saturday night syndrome (so named because it can be acquired by
sleeping with the arm over the back of a chair whilst in a drunken stupor,
so compressing the plexus):
Is due to compression of the lower part of the brachial plexus. As this is
really abrachial plexus injury, the median and ulnar nerves may also be
involved.
It may also be compressed by the use of shoulder crutches.
Nerve function usually fully recovers within a few weeks.
In the upper arm (triceps and brachioradialis are often spared):
May be due to a compression lesion but fracture is the usual cause.
Injections given in the arm of small babies can damage the radial nerve.
As the nerve often passes down in the spiral groove of the humerus, it
may be injured with a fracture of the shaft of the humerus.
At the elbow:
The radial nerve may be entrapped at the elbow at a number of sites but
the most common is the proximal border of the tendon of supinator
called the arcade of Frohse.
Check for tenderness over the radial tunnel. There may be pain when the
fingers are extended against resistance.
Supination from a pronated position along with flexion of the wrist may
reproduce the symptoms.
Lesions at the wrist:
Cause finger drop with a normal wrist and intact sensation.
Causes include fracture of the radius,elbow deformity, soft-tissue masses
and compression by the extensor carpi radialis brevis.
Lesions of the superficial nerves (cause pain and sensory loss but no
motor loss):
At the elbow, ruptured synovial effusion is the most common cause. In

the forearm there may be an aberrant course through the muscles.


At the wrist, causes include compression from plaster casts, wristbands
or handcuffs, especially the type than get tighter with struggling. Other
causes are surgery, injections and nerve tumours.
-------------------------------------------------------------------------------------------------------------------------------------------------------SR. NERVE
DEFICIT OR SIGN

MUSCLE GROUP

FUNCTIONAL

------------------------------------------------------------------------------------------------------------------------------------------------------1

Long thoracic

Serratus anterior

2 Suprascapular
arm

Supraspinatus and

Infraspinatus
3 Axillary
abduct arm
4 Radial
loss of

Winging scapula
Difficulty in initiating

abduction

Deltoid and Teres minor

Extensors of forearms,

wrist proximal phalanges and thumb

Inability to fully

Loss of arm extension,

foreams extension, supination,


abduction, loss of wrist extension

(wrist drop), loss of proximal


phalangeal extension and thumb extension.

5 Musculoflexion,

Flexors of arm and

cutaneous
6 Median
pronator, and

forearm

Weak arm and forearm

weak forearm supination.

Wrist and hand flexors

Paralysis of flexor,

thenar muscles, inability to fully


flex the index and middle fingers
(sign of benediction), Pointing
index finger, Labourers nerve.

7 Ulnar
distal and

Wrist and hand flexors

Inability to extend the

middle phalanges (clawhand);


loss of thumb abduction,
Froment sign positive.
-------------------------------------------------------------------------------------------------------------------------------------------------------(Q.128)

The Physiological locking of the KNEE joint occurs as a result of?


(a)

Medial rotation of tibia over the fixed femur

(b)

Lateral rotation of femur over the tibia

(c)

Lateral rotation of tibia over the femur

(d)

Medial rotation of femur over the fixed tibia

Your Response :
Correct Answer :
Exp:

d
Medial rotation of femur over the fixed tibia (Ref. BD Chaurasia
Vol. II, 3rd ed. 130)
LOCKING AND UNLOCKING OF KNEE JOINT
Locking of the knee joint occurs as a result of medial rotation of the
femur during the last stage of extension.
The AP diameter of the lateral femoral condyle is less than that of the
medial condyle. Asa result, when the lateral condylar articular surface is
fully used up by extension, part of medial condylar surface remains
unused.
At this stage the lateral condyle acts as an axis around which medial
condyle rotates backwards (i.e. medial rotation of the femur occurs ), so
that the remaining part of the medial condylar surface is also taken
up. This movement locks the knee joint.
*Locking is produced by action of Quadriceps femoris (it also produces
extension).
*Unclocking is brought about by the action of the popliteus muscle
(popliteus also protects lateral meniscus by pulling it backwards).

(Q.129)

Coronary ligament is between?

(a)

Menisci and synovium

(b)

Posterior horns of two menisci

(c)

Menisci and Tibia

(d)

Menisci and Femur

Your Response :
Correct Answer :
Exp:

c
Menisci and Tibia (Ref. BD Chaurasia Vol. II, 3rd ed. 125)
Layer III of the knee joint capsule, the deepest layer, is the lateral part of
the joint capsule. It is attached to the edges of the tibia and femur
circumferentially in horizontal planes at the proximal and distal ends of
the knee joint. The capsular attachment to the outer edge of the lateral
meniscus is called the "coronary ligament." The popliteus tendon passes
through a hiatus in the coronary ligament to attach to the femur.
MENISCI
The medial and lateral menisci are wedge-shaped fibrous and
fibrocartilaginous structures between the femoral condyles and the tibial
plateaus.
The medial meniscus is C-shaped, more firmly anchored to the tibia, and
attached to the medial collateral ligament.
The lateral meniscus is O-shaped, less firmly anchored to the tibia, and
not attached to the lateral collateral ligament.
Therefore, the medial meniscus is more commonly injured than the
lateral meniscus.
The "triad" knee injury is composed of tears of the medial collateral
ligament, medial meniscus, and anterior cruciate ligament.
Coronary ligament: The fibrous capsule of knee joint is attached to the
periphery of the menisci. The part of the capusle between the menisci
and the tibia is some times called the coronary ligament.

(Q.130)

Which of the following is the arterial supply toA.CL?


(a)

Fibular head artery

(b)

Descending genicular artery

(c)

Superior genicular artery

(d)

Middle genicular artery

Your Response :
Correct Answer :
Exp:

d
Middle genicular artery (Ref. BD Chaurasia Vol. II, 3rd ed. 52, 71,
72)
Descending genicular artery
Last branch of femoral artery
Arises just above the hiatus magnus
Divides into a superficial saphenous branch that accompanies the
sapheneous nerve and a deep branch which reaches knee by piercing
vastus medialis.
There are 5 genicular branches of Popliteal artery:
Two superior

Two inferior

One middle

The MIDDLE GENICULAR ARTERY pierces the oblique popliteal


ligament of the knee, and supplies the cruciate ligaments and the
synovial membrane of the knee joint.
(Q.131)

A.n altered sensation over the area of great saphenous vein in the leg may occur due
to injury to which of the following nerve?
(a)

Femoral nerve

(b)

Tibial nerve

(c)

Sural nerve

(d)

Fibular nerve

Your Response :
Correct Answer :
Exp:

a
Femoral nerve (Ref: Grays Anatomy, 39th edition)
Cutaneous sensation of the medial leg is provided by the saphenous
nerve, which is a branch of the femoral nerve.
LOWER LIMB NERVE INJURIES
Superior GlutealNerve
Causes loss of abduction of the limb; impairment of gait; patient cannot

keep pelvis level when standing on one leg.


Sign is "Trendelenburg gait:'
Inferior GlutealNerve
Produces a weakned hip extension;
patient has difficulty rising from a sitting position or climbing stairs. .
Femoral Nerve
Induces weakened hip flexion; loss of extension of the knee.
Sensory loss occurs on the anterior thigh, medial leg, and foot. .
Obturator Nerve
Causes a loss of adduction of the thigh as well as sensory loss on medial
thigh.
Sciatic Nerve
Brings about a weakened extension of the thigh; loss of flexion of the
knee; and loss of function below the knee.
Sensory loss on the posterior thigh, leg (except medial side), and foot is
also observed.
Tibial nerve only
Causes a loss of flexion of the knee and digits;
loss of plantar flexion; weakened inversion and
Sensory loss on the leg (except medial)and plantar foot.
Common peroneal nerve
Produces a combination of deficits of lesion of the deep and superficial
peroneal nerves.
Sign is "foot drop."
Deep peroneal nerve
weakened inversion;
loss of extension of the digits;
loss of dorsiflexion "foot
'Sensory loss on anterolateral leg and dorsum of the foot.

Superficial peroneal nerve


loss of eversion of the foot.
Sensory loss on dorsum of foot except the first web space.
(Q.132)

The following are Hybrid muscles except ?


(a)

Pectoralis major

(b)

Pectineus

(c)

Digastric

(d)

Biceps brachii

Your Response :
Correct Answer :
Exp:

d
Biceps brachii
Composite muscles/Hybrid muscles(muscles with Dual nerve
supply):
1 Pectoralis major - medial pectoral nerve and lateral pectoral nerve
2 Brachialis - musculocutaneous nerve and radial nerve(for
proprioception)
3 Adductor magnus - obturator nerve and tibial division of sciatic nerve
4 Pectineus- anterier fibres by femoral nerve and posterior fibres by
obturator nerve
5 Digastric- mandibular division of the trigeminal nerve via the
mylohyoid nerve and posterior belly by facial nerve
6 Flexor pollicis brevis- superficial head by median nerve and deep head
by ulnar nerve
7 Flexor digitorum profundus- lateral half by anterior interosseous
branch of median nerve and medial half by ulnar nerve
8 Biceps femoris- long head by tibial branch of sciatic nerve and short
head by peroneal branch of sciatic nerve.

(Q.133)

Which of the following is not a tributary of cavernous sinus?


(a)

Superficial middle cerebral vein

(b)

Superior petrosal sinus

(c)

Inferior petrosal sinus

(d)

Deep cerebral vein

Your Response :
Correct Answer :
Exp:

d
Deep cerebral vein (Reference: Keith L. Moore, Clinically oriented
Anatomy, 6th edition)
TRIBUTARIES :
Superiorandinferior ophthalmic veins
Sphenoparietal sinus
Superficial middle cerebral veins
DRAINAGE :
Superior Petrosal sinus
Iinferior petrosal sinuses
Emissary veinsthrough theforaminaof the skull (mostly throughforamen
ovale).
There are also connections with thepterygoid plexusof veins viainferior
ophthalmic vein,deep facial veinand emissary veins.

(Q.134)

An otherwise healthy student taking no medications is concerned because he has


noticed several painless uniform "large bumps" at the back of his tongue. These are
most likely?
(a)

Aphthous ulcers

(b)

Candidal colonies

(c)

Circumvallate papillae

(d)

Filiform papillae

Your Response :
Correct Answer :
Exp:

c
The large bumps at the back of his tongue are circumvallate papillae.
These are large circular structures surrounded by moat-like depressions.
The lateral surfaces of these papillae contain taste buds. There are also

small serous-only salivary glands in these papillae.


Aphthous ulcers are small, white, or red mouth lesions.
Candidal colonies appear in thrush, which occurs more commonly in the
immunocompromised host or in those taking antibacterial drugs. You are
told that the patient is healthy and not taking medications, making this
condition unlikely.
Filiform papillae are the most numerous papillae of the tongue. They are
small, elongated cones that create the tongue's rough texture. They do not
contain taste buds.
Fungiform papillae are mushroom-shaped structures scattered among the
filiform papillae. They frequently contain taste buds. They are
intermediate in size between filiform and circumvallate papillae.

TYPES OF TONGUE PAPILLAE :


Conical

Conical papillae are not found in


horses. They are present in the
caudal 1/3 of the tongue. They point
caudally and have no taste buds.
There is a thick epithelium.

Foliate

Eight to twelve papillae in parallel


folds, one either side of the tongue
midline. Consists of a stratified
squamous epithelium, present in the
caudal third of the tongue. There are
taste buds, glands and lymphatics
present.

Vallate

There are three to six, often


secondary papillae in taste buds.
There are broad glands in the caudal
1/3 of tongue. Taste buds and
lymphatics are present.

Fungiform

They form the red dots on tongue


surface and consist of keratinised,
stratified squamous epithelium and
blood vessels. They are involved in
loss of heat via panting in dogs.
They are present in the rostral 2/3 of
the tongue and contain taste buds.

Filiform

(Q.135)

Filiform papillae are the most


numerous and point caudally. There
are no taste buds, glands or
lymphatics. They are the smallest
and consist of a thick keratin on
stratified squamous epithelium.
They are very prominent in cat and
are present in the rostral 2/3 of the
tongue.

Attempts to straighten out a flexed thigh cause great pain in a patient with
appendicitis. This is due to the position of the appendix near which muscle?
(a)

Adductor magnus

(b)

Biceps femoris

(c)

Gluteus maximus

(d)

Psoas major

Your Response :
Correct Answer :
Exp:

d
Psoas major
The path of the psoas major lies in the retroperitoneum and comes close
to the appendix. Acute appendicitis can cause either infection or a
sympathetic inflammation of the psoas. This produces clinically a
"positive psoas sign," in which attempts to straighten the patient's flexed
(to relieve pain) hip produce sometimes marked exacerbation of the pain.
None of the other muscles listed pass near the appendix.

(Q.136)

Which of the following structures does the fetal allantoic duct become in the adult?
(a)

Cloaca

(b)

Medial umbilical ligament

(c)

Urachus

(d)

Ureter

Your Response :
Correct Answer :

Exp:

Urachus Reference: Langmans Medical Embryology (10th edition)


The urachus is a fibrous remnant that extends from the umbilicus to the
urinary bladder. It is also known as the median umbilical ligament of the
anterior abdominal wall.
The cloaca is the primitive, endoderm-lined region that receives the
terminal portion of the hindgut. It is later subdivided into urogenital and
anal areas.
The medial umbilical ligament is a paired structure located deep to the
peritoneum of the anterior abdominal wall.
It is formed by the obliterated umbilical artery.
The ureter is the muscular tube that conveys urine from the kidney to the
urinary bladder.
The urethra is the passageway that carries urine from the bladder to the
perineum.

(Q.137)

Lining cells of the air passages and alveoli include the following EXCEPT
(a)

Kultisky cells

(b)

Clara cells

(c)

Brush cells

(d)

Langerhans cells

Your Response :
Correct Answer :
Exp:

d
Langerhan cells (Ref. Textbook of human histology by I B Singh Fig.
199; Fig. 199)
LUNG HISTOLOGY
The structure of large intrapulmonary bronchi is similia to trachea.
Cartilage is absent in the walls of bronchioles: the criterion that
distinguishes a bronchiole from a bronchus.
The amount of muscle in the bronchial wall increases as the bronchi
become smaller.
Subepithelial lymphoid tissue increases as the bronchi become smaller.
Glands become fewer, and are absent in the walls of bronchioles.

Trachea and larger bronchi are lined by pseudostratified ciliated


columnar epithelium.
As the bronchi become smaller the epithelium first becomes simple
ciliated columnar, then non-ciliated columnar, and finally cuboidal.
Apart from typical ciliated columnar cells, various other types of cells
seen in the epithelium lining the air passages include
Ciliated columnar cells
Serous cells (Non-ciliated)
Lymphocytes and other leucocytes
Goblet cells
Kulchitsky /Feyrter cells (cells similar to diffuse endocrine cells of gut
containing argyrophil granules)
Basal cells
Brush cells
Clara cells (non-cilliated cells present predominantly in terminal
bronchiole which produce surfactant)
The alveoli are mainly lined by two types of epithelial cellsType I cells are flat cells with large cytoplasmic extensions and are the
primary lining cells.
Type II cells (granular pneumocytes) are thicker, contain lamellar
inclusion bodies, and secrete surfactant.
Other special types of epithelial cells may be present:
Pulmonary alveolar macrophages (Dust cells),
Apud cells, and
Lymphocytes,
Plasma cells,
Mast cells (contain heparin, various lipids, histamine, and various
proteases that participate in allergic reactions).
(Q.138)

Bronchial artery supplies lungs up to?


(a)

Tertiary bronchioles

(b)

Terminal bronchioles

(c)

Respiratory bronchioles

(d)

Alveolar ducts

Your Response :
Correct Answer :
Exp:

c
Respiratory bronchioles (Ref. Textbook of human histology by I B
Singh 201)
The lungs recdeive xeoxygenated blood from the right ventricle through
pulmonary arteries.
Within the lung the arteries end in an extensive capillary network in the
walls of alveoli.
Blood oxygenated here is returned to the left atrium through pulmonary
veins.
Oxygeneated blood required for nutrition of the lung itself reaches the
lungs through bronchial arteries, which are distributed to the walls of
bronchi as far as the respiratory bronchiole.
Blood reaching the lung through these arteries is returned to the heart
partly through bronchil veins, and partly through the pulmonary veins.

(Q.139)

All are derivatives of Septum Transversum except:


(a)

Falciform ligament

(b)

Ligamentum teres

(c)

Coronary ligament

(d)

Lesser omentum

Your Response :
Correct Answer :
Exp:

b
Ligamentum teres
(Ref. High yield embryology pg. 39; Langmans embryo 9th/g. 295)
The free margin of the falciform ligament contains the umbilical vein,
which is obliterated after birth to form the round ligament of the liver
(ligamentum teres hepatis).

Thecranialpart of the septum transversum gives rise to thecentral


tendonof the diaphragm and is the origin of themyoblaststhat invade
thepleuroperitonealfolds resulting in the formation of the muscular
diaphragm. Thecaudalpart of the septum transversum is invaded by
thehepatic diverticulumwhich divides within it to form the liver and thus
gives rise to theventral mesenteryof the foregut, which in turn is the
precursor of thelesser omentum, thevisceral peritoneumof theliverand
thefalciform ligament. Though not derived from the septum transversum,
development of the liver is highly dependent upon signals originating
here. Bone Morphogenic Protein 2 (BMP-2), BMP-4, and BMP-7
produced from the septum transversum join Fibroblast Growth Factor
(FGF) signals from the cardiac mesoderm induce part of the foregut to
differentiate towards a hepatic fate.
(Q.140)

All of the following develop in the mesentery of stomach except:


(a)

Liver

(b)

Spleen

(c)

Pancreas

(d)

Kidneys

Your Response :
Correct Answer :
Exp:

d
Kidneys (Ref. Langmans embryo 9th/g. 294)
Lengthening and fusion of the dorsal mesogastrium to the posterior body
wall also determine the final position of the pancreas. Initially the organ
grows into the dorsal mesoduodenum, but eventually its tail extends into
the dorsal mesogastrium.

(Q.141)

Superior Gluteal nerve supplies the following muscles except:


(a)

Gluteus Maximus

(b)

Gluteus Minimus

(c)

Tensor Fascia Lata

(d)

Gluteus Medius

Your Response :
Correct Answer :

Exp:

Gluteus Maximus (Ref: BDC Volume 2, 3rd edition pg. 65)


SUPERIOR GLUTEAL NERVE
Origin
The superior gluteal nerve originates in the sacral plexus.
It arises from the dorsal divisions of L4, L5, and SExp. Q
It leaves the pelvis through the greater sciatic foramen above the
piriformis, accompanied by the superior gluteal artery and vein.
Divisions:
It then divides into a superior and an inferior branch.
The superior branch accompanies the upper branch of the deep division
of the superior gluteal artery and ends in the gluteus minimus.
The inferior branch runs with the lower branch of the deep division of
the superior gluteal artery across the gluteus minimus; it gives filaments
to the gluteus medius and minimus, and ends in the tensor fasciae latae.
The superior gluteal nerve and vessels travel above the piriformis muscle
through the greater sciatic foramen; the inferior gluteal nerve and vessels
travel below the muscle.
Clinical disease
In normal gait, the small gluteal muscles on the stance side can stabilize
the pelvis in the coronal plane.
Weakness or paralysis of these muscles caused by a damaged superior
gluteal nerve can result in a weak abduction in the affected hip joint
(Trendelenburg gait).Q
In a positive Trendelenburg test the pelvis sags toward the normal
unsupported side (the swing leg). The opposite, when the pelvis is
elevated on the swing side, is known as Duchennee limp.
Bilateral loss of the small gluteal muscles results in a waddling gait.
The inferior gluteal nerve originates in the sacral plexus. It arises from
the dorsal divisions of the fifth lumbar and first and second sacral nerves:
it leaves the pelvis through the greater sciatic foramen, below the
piriformis, and divides into branches which enter the deep surface of the
gluteus maximus. It is the principal extensor of the thigh.

(Q.142)

Tensor tympaniis supplied by:

(a)

Facial nerve

(b)

Glossopharyngeal nerve

(c)

Trigeminal nerve

(d)

Vagus nerve

Your Response :
Correct Answer :
Exp:

c
Trigeminal nerve (Ref. Greys anatomy Fig. 782)
Meckel's cartilage (first arch) and Reichert cartilage (second arch). The
otic capsule has a role in formation of the stapes footplate. It is generally
agreed that the head of the malleus and the body and short process of the
incus are formed from Meckel's cartilage and are initially continuous
with the cartilaginous mandible. The mandibular branch of the trigeminal
nerve is the nerve of the first arch; thus it supplies the tensor tympani
muscle, also a derivative of the first branchial arch. The long process of
the incus, handle of the malleus, stapes superstructure, and tympanic
surface of the stapes footplate are derived from the Reichert's cartilage.
The facial nerve is the nerve of the second arch; this supplies the
stapedius muscle. The vestibular surface of the footplate is a derivative
of the mesoderm of the otic capsule, as is the anular ligament.

(Q.143)

True about Scalanenus anterior muscle:


(a)

It is attached to the tubercle of second Rib

(b)

It is anterior to the transverse cervical artery.

(c)

It is pierced by the Phrenic nerve.

(d)

It separates the subclavian vein from the subclavian artery.

Your Response :
Correct Answer :
Exp:

d
It separates the subclavian vein from the subclavian artery.
(Ref. Grays Anatomy of the Human Body/pg. 577)
Around scalenus anterior muscle from lateral to medial passes the
phrenic nerve (C3-C5) which is the sole motor innervation to the
diaphragm. Also in this region is the subclavian artery and vein. The vein
lies anterior to the scalenus anterior while the artery is posterior to it.

Scalenus muscles:
a. Scalenus anterior - anterior tubercles of the transverse cervical
processes to the scalene tubercle of the 1st rib.
b. Scalenus medius - posterior tubercles of all of the transverse cervical
processes to the first rib.
c. Scalenus posterior - posterior tubercles of the transverse cervical
processes to the 2nd rib.
(Q.144)

Lacrimation is affected in injury to:


(a)

Greater petrosal nerve

(b)

Nasociliary nerve

(c)

Nerve to stapedius

(d)

Chorda tympani

Your Response :
Correct Answer :
Exp:

a
Greater petrosal nerve (Ref. BD Chaurasia, Anatomy, Vol 3, 2nd/
pg. 48)
Lacrimal gland:
It is situated at the upper and outer aspect of eyeball in the lacrimal fossa.
Lacrimal nerve (has sensory and secretomotor fibres) supply it.
The secretomotor fibres starts in lower pons, pass through nervus
intermedius, geniculate ganglion,
greater petrosal nerve, nerve of pterygoid canal, Ptyregopalatine
ganglion, zygomatic nerve,
zygomatico-temporal nerve, communicating branch of lacrimal
nerve Lacrimal gland.
The end of the labyrinthine segment of facial nerve is marked by the
formation of the geniculate ganglion. In this region, the greater
superficial petrosal nerve carrying fibers to the lacrimal gland leaves
anteriorly from the ganglion by means of the hiatus of the facial canal.
The greater superficial petrosal nerve travels anteriorly carrying
parasympathetic fibers, for instance, to the lacrimal gland.

(Q.145)

Cranial nerve carrying parasympathetic fibers?


(a)

II

(b)

VII

(c)

XI

(d)

XII

Your Response :
Correct Answer :
Exp:

b
VII

(Ref. BDC 4th/pg. 334; Moores anatomy 5th/pg. 1124)

Cranial nerves carrying visceral motor axons (presynaptic


parasympathetic) are:
- Cranial nerve III (ciliary ganglion)
- Cranial nerve VII (Pterygopalatine & Submandibular ganglion)
- Cranial nerve IX (Otic ganglion)
- Cranial nerve X (Visceral ganglions)
(Q.146)

The following are branches of posterior cord of brachial plexus except ?


(a)

Musculocutaneous nerve

(b)

Thoracodorsal nerve

(c)

Axillary nerve

(d)

Radial nerve

Your Response :
Correct Answer :
Exp:

a
Musculocutaneous nerve
REF : CLINICAL ANATOMY BY REGIONS SNELL 8th Ed.
Summary of the Branches of the Brachial Plexus and Their
Distribution
Branches
Roots

Distribution

Dorsal scapular nerve


(C5)

Rhomboid minor, rhomboid major, levator


scapulae muscles

Long thoracic nerve


(C5, 6, 7)

Serratus anterior muscle

Upper Trunk
Suprascapular nerve
(C5, 6)

Supraspinatus and infraspinatus muscles

Nerve to subclavius (C5, Subclavius


6)
Lateral Cord
Lateral pectoral nerve
(C5, 6, 7)

Pectoralis major muscle

Musculocutaneous
nerve (C5, 6, 7)

Coracobrachialis, biceps brachii, brachialis


muscles; supplies skin along lateral border of
forearm when it becomes the lateral
cutaneous nerve of forearm

Lateral root of median See medial root of median nerve


nerve (C5, 6, 7)
Posterior Cord
Upper subscapular
nerve (C5, 6)

Subscapularis muscle

Thoracodorsal nerve
(C6, 7, 8)

Latissimus dorsi muscle

Lower subscapular
nerve (C5, 6)

Subscapularis and teres major muscles

Axillary nerve (C5, 6)

Deltoid and teres minor muscles; upper


lateral cutaneous nerve of arm supplies skin
over lower half of deltoid muscle

Radial nerve (C5, 6, 7,


8; T1)

Triceps, anconeus, part of brachialis,


extensor carpi radialis longus; via deepradial
nerve branch supplies extensor muscles of
forearm: supinator, extensorcarpi radialis
brevis, extensor carpi ulnaris, extensor
digitorum, extensor digitiminimi, extensor
indicis, abductor pollicis longus, extensor

pollicis longus, extensor pollicis brevis; skin,


lower lateral cutaneous nerve of arm,
posteriorcutaneous nerve of arm, and
posterior cutaneous nerve of forearm; skin
onlateral side of dorsum of hand and dorsal
surface of lateral three and a halffingers;
articular branches to elbow, wrist, and hand
Medial Cord
Medial pectoral nerve
(C8; T1)

Pectoralis major and minor muscles

Medial cutaneous nerve Skin of medial side of arm


of arm joined by
intercostal brachial
nerve from second
intercostal nerve (C8;
T1, 2)
Medial cutaneous nerve Skin of medial side of forearm
of forearm (C8; T1)

(Q.147)

Ulnar nerve (C8; T1)

Flexor carpi ulnaris and medial half of flexor


digitorum profundus, flexor digitiminimi,
opponens digiti minimi, abductor digiti
minimi, adductor pollicis, third and fourth
lumbricals, interossei, palmaris brevis, skin
of medial half ofdorsum of hand and palm,
skin of palmar and dorsal surfaces of medial
oneand a half fingers

Medial root of median


nerve (with lateral root)
forms median nerve
(C5, 6, 7, 8; T1)

Pronator teres, flexor carpi radialis, palmaris


longus, flexor digitorum superficialis,
abductor pollicis brevis, flexor pollicis
brevis, opponens pollicis, first twolumbricals
(by way of anterior interosseous branch),
flexor pollicis longus, flexor digitorum
profundus (lateral half), pronator quadratus;
palmar cutaneousbranch to lateral half of
palm and digital branches to palmar surface
of lateralthree and a half fingers; articular
branches to elbow, wrist, and carpal joints

In case of IVC obstruction, the collaterals open through all the following EXCEPT:
(a)

Superior epigastric & inferior epigastric vein.

(b)

Posterior Intercostal & iliolumbar vein.

(c)

Azygous vein & internal thoracic.

(d)

Lateral thoracic & superior circumflex iliac veins.

Your Response :
Correct Answer :
Exp:

b
Posterior intercostal & iliolumbar vein
ed. 102, 125, 265)

(Ref. Snells anatomy 5th

Internal thoracic, lateral thoracic, azygos veins (tributaries of SVC) all


form collaterals with the tributaries of IVC.
Internal thoracic vein inferior epigastric vein external iliac vein
IVC.
Lateral thoracic superior epigastric vein great saphenous vein
femoral vein IVC.
Azygos veins (main and hemiazygous veins) firm direct connection
between SVC and IVC.
(Q.148)

Regarding Clavicle fracture


Assertion there is upward displacement of the proximal fragment
Reason: it is due to sternocleidomastoid muscle
(a)

(b)

Both Assertion and Reason are true, and Reason is the correct
explanation for Assertion
Both Assertion and Reason are true but Reason is not the correct
explanation for
Assertion

(c)

Assertion is true but Reason is false

(d)

Reason is true but Assertion is false

Your Response :
Correct Answer :
Exp:

a
Both Assertion and Reason are true, and Reason is the correct
explanation for Assertion
Fracture of the clavicle: results from a fall on the shoulder or

outstretched hand or may be caused by the obstetrician in breech


(buttocks) presentation or may occur when the infant presses against the
maternal pubic symphysis during its passage through the birth canal. It
occurs in the junction of the middle and lateral thirds, which is the
weakest point and results in upward displacement of the proximal
fragment because of the pull of the
sternocleidomastoid muscle and downward displacement of the distal
fragment because of the pull of the deltoid muscle and gravity.
(Q.149)

Common peroneal nerve is related to:


(a)

Neck of fibula

(b)

Shaft of fibula

(c)

Medial tibial condyle

(d)

Posteromedial aspect of proximal tibial shaft

Your Response :
Correct Answer :
Exp:

a
Neck of fibula
The common peroneal nerve lies posterior to the laterally situated biceps
femoris tendon in relationship to neck of fibula.

(Q.150)

Which of the following cell types is derived from neuroepithelial cells?


(a)

Astrocytes

(b)
(c)

Enterochromaffin cells
Melanocytes

(d)

Odontoblasts

Your Response :
Correct Answer :
Exp:

a
Reference: Langmans Medical Embryology (10th edition)
Astrocytes and oligodendrocytes are both derived from glioblasts, which,
in turn, are derived from neuroepithelial cells. Other neuroepithelial cell
derivatives include neuroblasts and ependymal cells. The astrocytes are

the largest and most numerous glial cells. These cells are responsible for
maintaining the blood-brain barrier, creating a three-dimensional
framework for the central nervous system, performing repairs in
damaged neural tissues, and controlling the interstitial environment.
All the other choices are derived from neural crest cells. Other neural
crest derivatives include the neurons of the parasympathetic and
sympathetic ganglia (including the adrenal medulla), the dorsal root
ganglia of the peripheral nervous system, the sensory ganglia of cranial
nerves V, VII, IX, and X, and the leptomeninges (pia and arachnoid).
(Q.151)

Which is nottrue about oculomotor nerve?


(a)

It is parasympathetic

(b)

Passes through inferior orbital fissure

(c)

In palsy, there is lateral deviation of eye.

(d)

The paired oculomotor nerves exit from the caudal aspect of the
interpeduncular fossa.

Your Response :
Correct Answer :
Exp:

b
Passes through inferior orbital fissure (Ref. BDC-III-3rd ed. 72)
Foramina of Skull Base

Transmits

Superior orbital fissure


1 Oculomotor nerve
2 Trochlear nerve

III
IV

3 Abducens nerve
4 Ophthalmic branch of trigeminal nerve V
5 Ophthalmic veins (superior and inferior)
6 Lacrimal and frontal nerves
7 Meningeal branch of lacrimal artery
8 Orbital branch of middle meningeal artery
Interior orbital fissure
1 Infraorbital vessels

2 Minor branches from pterygopalatinc ganglion


(Q.152)

A patient experiences visual difficulties. When a light is shined in her right eye,
there is no pupillary response in either eye. However, upon shining a light in her left
eye, both ipsilateral and contralateral pupillary responses are apparent. Her
extraocular movements are intact. What is the most likely location of her lesion?
(a)

Oculomotor nerve, left side

(b)

Oculomotor nerve, right side

(c)

Optic nerve, left side

(d)

Optic nerve, right side

Your Response :
Correct Answer :
Exp:

d
Optic nerve, right side
This woman has a "Marcus-Gunn pupil" with a defect in the afferent
pathway of the optic nerve (in this case, on the right side). Recall that the
afferent limb of the papillary light reflex is the optic nerve (CN II); the
efferent limb is the oculomotor nerve (CN III; parasympathetic fibers).
When light is shined into her right eye, because her right optic nerve is
not functioning properly, the light signal is not transmitted to the central
nervous system (CNS), resulting in no papillary response. As light is
shined into her left eye, the left optic nerve transmits the signal to the
CNS, which then sends an outbound signal through both the right and
left oculomotor nerves to cause pupillary constriction in both eyes.
Anatomically, the optic nerve is a special sensory nerve that originates in
the retina of the eye, passes through the optic foramen of the sphenoid
bone, and has a destination in the diencephalon via the optic chiasm.
The oculomotor nerve) innervates all extraocular muscles except the
lateral rectus (innervated by the abducens nerve) and the superior oblique
(innervated by the trochlear nerve). The oculomotor nerve also mediates
papillary constriction (parasympathetic fibers), eyelid opening (levator
palpebrae), and innervates the ciliary muscle (allowing accommodation).
A patient with a lesion of the left optic nerve would have no papillary
responses in either eye when shining a light in the left eye; pupillary
responses would be present in both eyes when shining a light in the right
eye.

(Q.153)

Y chromosome is:

(a)

Acrocentric

(b)

Metacentric

(c)

Submetacentric

(d)

Telocentric

Your Response :
Correct Answer :
Exp:

a
Acrocentric
(Ref. Nelson Textbook of Pediatrics 17th ed. Pg. 383; Chapter 70)
Cytogeneticists arrange chromosomes by size in pairsthe largest being
chromosome 1 and the smallest, chromosome 22 (although chromosome
21 has been found to actually be the smallest)and then the sex
chromosomes X and Y.
The X chromosome is a large submetacentric chromosome, and the Y
chromosome is a small acrocentric chromosome.
The position of the centromere in regard to the chromosome arms is
another distinguishing feature of each chromosome. The short arm of a
chromosome is referred to as p (for petite) and the long arm as q (for the
next letter in the alphabet).
Robertsonian rearrangements are a special class of translocation, in
which the long arms of two acrocentric chromosomes (chromosomes 13,
14, 15, 21, and 22) join together, generating a fusion chromosome that
contains virtually all of the genetic material of the original two
chromosomes.

(Q.154)

Cells in intestine that migrate to surface include the following EXCEPT


(a)

Paneth cells

(b)

Enterocytes

(c)

Goblet cells

(d)

Enteroendocrine cells

Your Response :
Correct Answer :
Exp:

a
Paneth cells

(Ref. Ganong Physiology 22nd ed. 506 and Shakelfords text book of
alimentary tract surgery 7th ed. 998)
Paneth (zymogen) cells reside at the base of crypts and are the only cell
type to undergo downward migration from the proliferation zone.
Paneth cells provide host defense against microbes in the small intestine
They are known to produce lysozyme, which destroys bacteria. They are
also known to secrete defensins.
In addition to defensins, Paneth cells secrete lysozyme and
phospholipase A2, both of which have clear antimicrobial activity. This
battery of secretory molecules gives Paneth cells a potent arsenal against
a broad spectrum of agents, including bacteria, fungi and even some
enveloped viruses.
(Q.155)

Common carotid artery bifurcation occurs at the level of(a)

Hyoid bone

(b)

Cricoid cartilage

(c)

Superior border of thyroid cartilage

(d)

Inferior border of thyroid cartilage

Your Response :
Correct Answer :
Exp:

c
Superior border of thyroid cartilage (Ref. Greys anatomy fig. 507)
The Common Carotid Artery
The common carotid arteries differ in length and in their mode of origin.
The right begins at the bifurcation of the innominate artery behind the
sternoclavicular joint and is confined to the neck.
The left springs from the highest part of the arch of the aorta to the left
of, and on a plane posterior to the innominate artery, and therefore
consists of a thoracic and a cervical portion.
The cervical portions of the common carotids passes obliquely upward,
from behind the sternoclavicular articulation, to the level of the upper
border of the thyroid cartilage, where it divides into the external and
internal carotid arteries.
The principal arteries of supply to the head and neck are the two
common carotids; they ascend in the neck and each divides into two

branches, viz., (1) the external carotid, supplying the exterior of the head,
the face, and the greater part of the neck; (2) the internal carotid,
supplying to a great extent the parts within the cranial and orbital
cavities.
(Q.156)

While exposing left subclavian artery which of the following muscle is not dissected?
(a)

Omohyoid

(b)

Scalenus anticus

(c)

Scalenus medius

(d)

Sternocleidomastoid

Your Response :
Correct Answer :
Exp:

c
Scalenus medius
The subclavian artery
The first part of the left subclavian artery arises from the arch of the
aorta, behind the left common carotid, and at the level of the fourth
thoracic vertebra; it ascends in the superior mediastinal cavity to the root
of the neck and then arches lateral ward to the medial border of the
Scalenus anterior. Relations.It is in relation, in front, with the vagus,
cardiac, and phrenic nerves, which lie parallel with it, the left common
carotid artery, left internal jugular and vertebral veins, and the
commencement of the left innominate vein, and is covered by the
Sternothyreoideus, Sternohyoideus, and Sternocleidomastoideus; behind,
it is in relation with the esophagus, thoracic duct, left recurrent nerve,
inferior cervical ganglion of the sympathetic trunk, and Longus colli;
higher up, however, the esophagus and thoracic duct lie to its right side;
the latter ultimately arching over the vessel to join the angle of union
between the subclavian and internal jugular veins. Medial to it are the
esophagus, trachea, thoracic duct, and left recurrent nerve; lateral to it,
the left pleura, and lung.
The first part of the right subclavian artery arises from the innominate
artery, behind the upper part of the right sternoclavicular articulation, and
passes upward and lateralward to the medial margin of the Scalenus
anterior. It ascends a little above the clavicle. Relations.It is covered,
in front, by the integument, superficial fascia, Platysma, deep fascia, the
clavicular origin of the Sternocleidomastoid, the Sternohyoid, and
Sternothyroid, and another layer of the deep fascia. It is crossed by the
internal jugular and vertebral veins, by the vagus nerve and the cardiac
branches of the vagus and sympathetic, and by the subclavian loop of the

sympathetic trunk which forms a ring around the vessel. The anterior
jugular vein is directed lateral ward in front of the artery, but is separated
from it by the Sternohyoid and Sternothyroid. Below and behind the
artery is the pleura, which separates it from the apex of the lung; behind
is the sympathetic trunk, the Longus collie and the first thoracic vertebra.
The right recurrent nerve winds around the lower and back part of the
vessel
The third portion of the subclavian artery runs downward and lateral
ward from the lateral margin of the Scalenus anterior to the outer border
of the first rib, where it becomes the axillary artery. This is the most
superficial portion of the vessel, and is contained in the subclavian
triangle. The left subclavian artery is more deeply placed than the right in
the first part of its course, and, as a rule, does not reach quite as high a
level in the neck. The posterior border of the Sternocleidomastoid
corresponds pretty closely to the lateral border of the Scalenus anterior,
so that the third portion of the artery, the part most accessible for
operation, lies immediately lateral to the posterior border of the
Sternocleidomastoid. It is covered, in front, by the skin, the superficial
fascia, the Platysma, the supraclavicular nerves, and the deep cervical
fascia. The external jugular vein crosses its medial part and receives the
transverse scapular, transverse cervical, and anterior jugular veins, which
frequently form a plexus in front of the artery. Behind the veins, the
nerve to the Subclavius descends in front of the artery. The terminal part
of the artery lies behind the clavicle and the Subclavius and is crossed by
the transverse scapular vessels. The subclavian vein is in front of and at a
slightly lower level than the artery. Behind, it lies on the lowest trunk of
the brachial plexus, which intervenes between it and the Scalenus
medius. Above and to its lateral side are the upper trunks of the brachial
plexus and the Omohyoid. Below, it rests on the upper surface of the first
rib.
(Q.157)

Facial nerve can be assessed by testing action of which of the following muscle?(a)

Orbicularis oris

(b)

Temporalis

(c)

Masseter

(d)

Pterygoid

Your Response :
Correct Answer :
Exp:

a
Orbicularis oris (Ref. BD Chaurasia Anatomy Vol. III 4th ed. 52, 54,

141)
Facial nerve can be assessed by testing Orbicularis oris muscle.
All muscles around mouth are dilators (of oral fissure) except orbicularis
oris.
The orbicularis oris muscle is the sphincter muscle around the mouth.
The Orbicularis oris is not a simple sphincter muscle like the Orbicularis
oculi; it consists of numerous strata of muscular fibers surrounding the
orifice of the mouth but having different direction.
It is supplied by cranial nerve VII, buccal branch of the facial nerve.
(Q.158)

Pudendal nerve supplying sphincter of bladder has the root value of?
(a)

T12-L1

(b)

L4-L5

(c)

S2-S4

(d)

L2-L3

Your Response :
Correct Answer :
Exp:

c
S2-S3 (Ref. BD Chaurasia Anatomy Vol. II. 4th ed. 76, 335, 348)
A nerve that is formed by fibers from the second, third, and fourth sacral
nerves, passes through the greater sciatic foramen, and accompanies the
internal pudendal artery to terminate as the dorsal nerve of the penis or of
the clitoris.
The pudendal nerve is a nerve in the pelvic region that innervates the
external genitalia of both sexes, as well as sphincters for the bladder and
the rectum.
The pudendal nerve originates in the sacral plexus; it derives its fibers
from the ventral branches of the second, third, and fourth sacral nerves
(S2, S3, S4).
It passes between the piriformis and coccygeus muscles and leaves the
pelvis through the lower part of the greater sciatic foramen.
It crosses the spine of the ischium, and reenters the pelvis through the
lesser sciatic foramen.
It accompanies the internal pudendal vessels upward and forward along

the lateral wall of the ischiorectal fossa, being contained in a sheath of


the obturator fascia termed the pudendal canal.
The pudendal nerve gives off the inferior anal nerves. It soon divides into
two terminal branches: the perineal nerve, and the dorsal nerve of the
penis (males) or the dorsal nerve of the clitoris (in females).
Inferior anal nerves given off shortly after passing through the Greater
sciatic foramen. Perineal nerve more superficial terminal branch Dorsal
nerve of penis/Dorsal nerve of clitoris deeper terminal branch, travelling
into the deep perineal pouch
The pudendal nerve innervates the penis and clitoris, bulbospongiosus
and ischiocavernosus muscles, and areas around the scrotum, perineum,
and anus. At sexual climax, the spasms in the bulbospongiosus and
ischiocavernous results in ejaculation in the male and most of the
feelings of orgasm in both sexes.
(Q.159)

Adult cells cannot proliferate like fetal cells because:


(a)

CDK inhibitor act at S phase to prevent proliferation of adult cell.

(b)

In fetal cells, protease is absent for degrading CDK.

(c)

Cell cycle arrest in adults in response to DNA damage is more.

(d)

Cyclin-dependent kinase inhibitors block the cell cycle without


binding to cyclin-CDK complexes.

Your Response :
Correct Answer :
Exp:

a
CDK inhibitor act at S phase to prevent proliferation of adult cells
The orderly progression of cells through the various phases of cell cycle
is orchestrated by cyclins and cyclin-dependent kinases (CDKs), and by
their inhibitors.
There are two main classes of CDK inhibitors: the Cip/Kip and the
INK4/ARF families.
These inhibitors function as tumor suppressors and are frequently altered
in tumors

(Q.160)

Thoracic duct receives tributaries from all the following EXCEPT?


(a)

Bilateral ascending lumbar ducts

(b)

Bilateral descending thoracic ducts

(c)

Left upper intercostal ducts

(d)

Right bronchomediastinal lymphatic trunk

Your Response :
Correct Answer :
Exp:

d
Right bronchomediastinal lymphatic trunk
Thoracic duct
The thoracic duct is the largest lymphatic channel in the body.
It returns lymph to the venous circulation at the junction of the left
internal jugular vein and the left subclavian vein.
The thoracic duct develops from the distal portion of the right thoracic
duct, the anastomosis, and the cranial portion of the left thoracic duct.
The right lymphatic duct is derived from the cranial portion of the right
thoracic duct. Both ducts maintain their original connections with the
venous system and empty into the junction of the internal jugular and
subclavian veins.

(Q.161)

In median nerve injury at wrist, what is spared?


(a)

Abductor pollisis brevis

(b)

Adductor policis

(c)

Opponens pollicis

(d)

Flexor pollicis brevis

Your Response :
Correct Answer :
Exp:

b
Adductor policis
(Ref. BDC anatomy 4th d. vol. Exp. 52; and Keith L Moore Anatomy
5th Ed. 822)
MEDIAN NERVE
At the Elbow
Loss of flexion of the digits, thenar muscles, and lumbricals 1 and 2;
weakened wrist flexion; ulnar deviation upon flexion of the wrist; loss of

pronation. Sensory loss on lateral palm and digits 1, 2, and 3, and one
half of Exp. Sign is "ape or simian hand" and "flattening of the thenar
At the Wrist
Loss of function of the thenar muscles and lumbricals 1 and 2; "clawing"
of digits 2 and Exp.
Sensory loss on palmar surface of digits 1, 2, and 3, and one-half of Exp.
Sign is "ape or simian hand" and "flattening of thenar eminence." Carpal
tunnel compression or wrist laceration.
ULNAR NERVE
At the Elbow (medial epicondyle)
Weakened wrist flexion; radial deviation upon flexion of the wrist; loss
of abduction and adduction of the digits; loss of hypothenar muscles and
lumbricals 3 and Exp. Weakened flexion of digits 4 and Exp. Sensory
loss on digits 5 and one half of Exp. Sign is "claw hand."
At the Wrist
Loss of abduction and adduction of the digits; loss of the hypothenar
muscles and lumbricals 3 and Exp. Sensory loss on digits 5 and one half
of Exp. Sign is "claw hand:'
Adductor policis is supplied by ulnar nerve and hence it will be spared if
median nerve is injured.
(Q.162)

The left adrenal vein drains directly into which of the following veins?
(a)

Hemiazygos vein

(b)

Inferior vena cava

(c)

Left renal vein

(d)

Splenic vein

Your Response :
Correct Answer :
Exp:

c
Left renal vein
The left adrenal vein and the left gonadal vein (either testicular or
ovarian) drain into the left renal vein. The left renal vein then drains into
the inferior vena cava. In contrast, the right adrenal vein and right
gonadal vein drain directly into the inferior vena cava.

The hemiazygos vein receives the venous drainage from the body wall
on the left side of the thorax and abdomen. No visceral organs drain
directly to the azygos or hemiazygos veins.
The inferior vena cava receives the direct venous drainage from the right
adrenal vein, but not the left adrenal vein. Remember, the inferior vena
cava is on the right side of the abdomen.
The splenic vein receives the venous drainage from the spleen and part of
the pancreas and stomach. The splenic vein is part of the portal venous
system.
The superior mesenteric vein receives venous drainage from much of the
intestinal tract. It is part of the portal venous system and joins with the
splenic vein to form the portal vein.
(Q.163)

True regarding Portal system?


(a)

Is Intrahepatic only

(b)

It drains into SVC

(c)

Flow is centrifugal

(d)

Portal vein is formed by union of SMV and splenic vein

Your Response :
Correct Answer :
Exp:

d
Portal vein is formed by union of SMV and splenic vein
(Ref. BDC Anatomy Vol. Exp. Pg. 234)

(Q.164)

Which of the following is present in males but not in females?


(a)
(b)
(c)
(d)

Bulbospongiosus muscle
Bulbourethral gland
Corpus cavernosum
Membranous urethra

Your Response :
Correct Answer :
Exp:

b
Bulbourethral gland

The bulbourethral glands are paired structures located within the deep
perineal pouch, embedded within the sphincter urethrae. Their ducts pass
to the spongy urethra. The homologous female structures are the greater
vestibular (Bartholin's) glands, which are located in the superficial
perineal pouch.
The bulbospongiosus muscles lie superficial to the bulb of the penis in
males and to the bulbs of the vestibule in females.
The corpora cavernosa are paired structures, consisting of cavernous
erectile tissue that form a large portion of the penile shaft in males and of
the body of the clitoris in females.
The membranous urethra is the portion of the urethra that passes through
the urogenital diaphragm in both males and females.
The perineal body is the centrally located tendinous structure that
provides attachment for perineal musculature in both males and females.
It separates the urogenital area from the anal area and is an important
obstetric landmark.
(Q.165)

In Post-ductal Coarctation of aorta all the following participate in distal collateral


formation EXCEPT:
(a)

Axillary artery

(b)

Vertebral artery

(c)

Suprascapular artery

(d)

Posterior intercostal artery

Your Response :
Correct Answer :
Exp:

b
Vertebral artery
In the anterior system, the internal mammary arteries and the epigastric
arteries join to form collaterals which supply the abdominal wall and the
lower extremities.
In the posterior system, the parascapular arteries connect with the
intercostal arteries to form collaterals which supply the distal aortic
compartment and primarily the abdominal viscera.
The left subclavian may form collaterals through linkage to the IMA's
and intercostals to give blood supply distally. The right subclavian will
join with the vertebral, spinal, cervical, and scapular branches, and will

eventually provide blood supply to the intercostal circuit.


(Q.166)

Punnett Square is used to calculate?


(a)

Genotype of the offspring.

(b)

Diseased and non-diseased individuals

(c)

Statistical data collection

(d)

Anatomical surface area of human body

Your Response :
Correct Answer :
Exp:

a
Genotype of the offspring.
A Punnett square is a chart which shows/predicts all possible gene
combinations in a cross of parents (whose genes are known). Punnett
squares are named for an English geneticist, Reginald Punnett. He
discovered some basic principles of genetics, including sex linkage and
sex determination. He worked with the feather color traits of chickens in
order to quickly separate male and female chickens.

(Q.167)

In which of the following blood is oxygenated?


(a)

Right heart

(b)

Umbilical vein

(c)

Umbilical artery

(d)

Pulmonary artery

Your Response :
Correct Answer :
Exp:

b
Umbilical vein.
The bronchial arterial blood is oxygenated blood, in contrast to the
partially deoxygenated blood in the pulmonary arteries.
In utero, the lungs do not perform gas exchange and accordingly,
pulmonary blood vessels are markedly constricted. Oxygenated blood
flows from the placenta through the umbilical vein to the right heart
where the majority of the blood is shunted to the aorta through the
foramen ovale and patent ductus arteriosus.

Having circulated through the capillaries of the placental villi, the fetal
blood returns through the umbilical vein to the fetus about 80% saturated
with oxygen and containing many important nutrients, antibodies, and
hormones.
The oxygenated blood then passes toward the liver. However, the greater
volume of it bypasses the liver, since this organ is not fully functioning,
and travels to the inferior vena cava by way of the ductus venosus. At the
same time, the ductus venosus receives poorly oxygenated blood from
the gut by way of the left branch of the portal vein. In addition, the
inferior vena cava already contains venous blood from the lower part of
the trunk and the lower limbs of the fetus.
As a consequence of this admixture of blood from these various sources,
the inferior vena cava contains blood about 67% saturated with oxygen.
(Q.168)

Smooth muscles of the dorsal aorta develop from which mesoderm?


(a)

Paraxial mesoderm

(b)

Intermediate mesoderm

(c)

Lateral plate mesoderm

(d)

Septum transversum

Your Response :
Correct Answer :
Exp:

c
Lateral plate mesoderm
(Ref. Human Embryology by IB Singh 6th/pg. 46; Langmans
embryology 9th/pg. 199)
With the exception of some smooth muscle tissue, the muscular system
develops from the mesodermal germ layer and consists of skeletal,
smooth, and cardiac muscle.
1 Skeletal muscle : Derived from paraxial mesoderm which forms
somites from the occipital to the sacral regions and somatometric in the
head.
2 Smooth muscle
Differentiates from splanchnic mesoderm (which is a part of lateral plate
mesoderm) surrounding the gut and its derivatives.
3 Pupillary, mammary gland and sweat gland muscles From ectoderm
4 Cardiac muscle : Derived from splanchnic mesoderm surrounding the

heart tube.
The truncus arteriosus, which is the distal part of the bulbus cordis is
continuous beyond the peri- cardium with a large vessel called the aortic
sac. This sac gives off two branches, each of which runs dorsally in the
first pharyngeal arch on each side in the developing embryo. The
branches then pass caudally in the posterior wall of the embryo as the
two dorsal aortae. Five additional arteries now join the aortic sac to the
dorsal aortae. Meanwhile, the two dorsal aortae fuse throughout much of
their lengths to form the descend- ing thoracic aorta and the abdominal
aorta.
(Q.169)

Diaphragm develops from following except:


(a)

Septum transversum

(b)

Dorsal mesogastrium

(c)

Pleuro-peritoneal membrane

(d)

Cervical myotome

Your Response :
Correct Answer :
Exp:

d
Cervical myotome. Reference: Langmans Medical Embryology (10th
edition)
The diaphragm is derived from:
Septum transversum (which forms the central tendon of the diaphragm);
Two pleuroperitoneal membranes;
Muscular components from the lateral and dorsal body walls; and
The mesentery of the esophagus, in which the crura of the diaphragm
develop.

(Q.170)

Which of the following is a branch of trunk of the brachial plexus?


(a)

Nerve to subclavius

(b)

Superior intercostal nerve

(c)

Lateral thoracic nerve

(d)

Anterior thoracic nerve

Your Response :
Correct Answer :
Exp:

a
Nerve to subclavius. (Ref. BDC vol.1/ 4th/ 53)
Branches of posterior cord
Branches of Lateral cord

Branches of Medial cord

1 Upper subscapular nerve (C5,C6)


(C8,T1) Lateral pectoral nerve

Medial cutaneous nerve of arm

2 Lower subscapular nerve (C5,C6)


(C8,T1)
Lateral root of median.

Medial pectoral nerve

3 Nerve to latissimus dorsi/thoracodorsal


Musculocutaneous nerve
nerve (C6,C7,,C8)

(Q.171)

Medial cutaneous nerve of

forearm (C8,T1)

4 Axillary/circumflex nerve (C5,C6)


(C8,T1)
-

Medial root of median nerve

5 Radial nerve (C5,C6, C7,,C8 and T1)


-

Ulnar (C7,C8,T1)

True about Common peroneal nerve palsy is following except:


(a)

Loss of extension of great toe

(b)

Anaesthesia on the sole of the foot

(c)

Associated with fracture neck of fibula

(d)

Foot drop

Your Response :
Correct Answer :
Exp:

b
Anaesthesia on the sole of the foot
(Ref. BDC Anatomy 4th/ vol. 2 pg.86 & 174 & Fig. Exp.3)
NERVE
ABNORMALITY
1 Superior Gluteal Nerve
Impairment of gait;

- Causes loss of abduction of the limb;

Patient cannot keep pelvis level when standing on one leg. This is called
"Trendelenburg sign or gait
2 Inferior Gluteal Nerve

- Produces a weakned hip extension;

Patient has difficulty rising from a sitting position or climbing stairs.


3 Femoral Nerve

- Induces weakened hip flexion;

Loss of extension of the knee.


Sensory loss occurs on the anterior thigh, medial leg, and foot.
4 Obturator Nerve - Causes a loss of adduction of the thigh as well as
sensory loss on medial thigh.
(Q.172)

Femoral head is mainly supplied by?


(a)

Medial circumflex artery

(b)

Lateral circumflex artery

(c)

Profunda femoris artery

(d)

External iliac artery

Your Response :
Correct Answer :
Exp:

b
Lateral circumflex artery.
The vascular supply of the femoral head is also of paramount
importance. There are three main sources of vascular supply:
The retinacular vessels arising from the lateral femoral circumflex artery
and the inferior metaphyseal artery and then running beneath the
synovium along the neck, which they penetrate proximally both
anteriorly and posteriorly;
The interosseous circulation crossing the marrow spaces from distal to
proximal; and
Unreliably, the ligamentum teres artery.

(Q.173)

Derivatives of hypogastric sheath are following except:


(a)

Lateral ligament of urinary bladder

(b)

Presacral fascia

(c)

Broad ligament of uterus

(d)

Transverse cervical ligament

Your Response :
Correct Answer :
Exp:

c
Broad ligament of uterus.
(Ref. Greys Anatomy 12th/pg. 251)
HYPOGASTRIC SHEATH
The hypogastric sheath is a thick band of condensed pelvic fascia with
following features:
A fascial condensation which does not merely separate two potential
spaces
Gives passage to the vessels and nerves passing from the lateral wall of
the pelvis to the pelvic viscera, along with the ureters and, in the mate,
the ductus deferens.
As it extends medially from the lateral pelvic wall, the hypogastric
sheath divides into three laminae (leaflets or wings) which pass to or
between the pelvic organs, conveying neurovascular structures and
providing support. (Because of the latter function, they are also referred
to as ligaments):
a. anteriormost lamina lateral ligament of the bladder.
Passes to the bladder, conveying superior vesical arteries and veins/
b. Posteriormost lamina = presacral fascia.
Passes to the rectum, conveying the middle rectal artery and vein.
c. Middle lamina
i. In the male, forms a relatively thin fascial partition, the rectovesical
septum, between the back of the bladder and prostate anteriorly and the
rectum posteriorly.
ii. In the female, the middle lamina is most substantial of the three,
passing medially to the uterine cervix and vagina as the transverse
(cardinal) cervical ligament (lateral cervical or Mackenrodts ligament).

(Q.174)

Following are in posterior relation to the head of pancreas except:


(a)

Common bile duct

(b)

First part of duodenum

(c)

Inferior vena cava

(d)

Aorta

Your Response :
Correct Answer :
Exp:

b
First part of duodenum. (Ref. BDC 4th/Vol2 pg. 284)
Relations
Anteriorly: From right to left: the transverse colon and the attachment of
the transverse mesocolon, the lesser sac, and the stomach.
Posteriorly: From right to left: the bile duct, the portal and splenic veins,
the inferior vena cava, the aorta, the origin of the superior mesenteric
artery, the left psoas muscle, the left suprarenal gland, the left kidney,
and the hilum of the spleen
Pancreatic Ducts
The main duct of the pancreas begins in the tail and runs the length of the
gland, receiving numerous tributaries on the way. It opens into the
second part of the duodenum at about its middle with the bile duct on the
major duodenal papilla. Sometimes the main duct drains separately into
the duodenum. The accessory duct of the pancreas, when present, drains
the upper part of the head and then opens into the duodenum a short
distance above the main duct on the minor duodenal papilla. The
accessory duct frequently communicates with the main duct.

(Q.175)

The following nerves arise from anterior branch of mandibular division of


trigeminal nerve except:
(a)

Nerve to Temporalis

(b)

Nerve to Medial pterygoid

(c)

Nerve to Lateral pterygoid

(d)

Buccal nerve

Your Response :
Correct Answer :
Exp:

b
Medial pterygoid. (Ref. BDC Vol. Exp. 4th/ pg. 145)

Mandibular nerve:
Largest branch of trigeminal nerve.
Sensory as well as motor distribution.
Nerve of first pharyngeal arch.

Branches:
o From main trunk:
Meningeal branch

Nerve to medial pterygoid

o From anterior division:


Buccal nerve (a sensory branch)
Nerve to temporalis

Nerve to lateral pterygoid

o From posterior division:


Auriculotemporal nerve
alveolar nerve
(Q.176)

Lingual nerve

Inferior

Pain in ethmoid sinusitis is carried by:


(a)

Lacrimal nerve

(b)

Frontal nerve

(c)

Nasociliary nerve

(d)

Infratrochlear nerve

Your Response :
Correct Answer :
Exp:

c
Nasociliary nerve.
(Ref. BDC Vol. Exp. 4th/ pg. 118)
SENSORY INNERVATION
Nasociliary Nerve
The nasociliary nerve passes above and medial to the eye before giving
off following branches to the nose and the eye:
a. Branches to ciliary ganglion

b. Anterior ethmoidal

c. Posterior ethmoidal
ciliary nerves

d. Infratrochlear

e. 2-3 long

The nasal component is made of the ethmoidal and nasal nerves that
carry sensation from the roof of the nasal cavity, the skin of the nose, and
the sphenoid and ethmoid sinuses.
The ciliary component is made of the long and short ciliary nerves that
carry sensation from the eye and cornea.
Lacrimal Nerve
eye.

The lacrimal nerve passes above and lateral to the

It carries sensation from the lateral part of the upper eyelid.


Frontal Nerve
The frontal nerve passes over the eye and divides into
(1) Supratrochlear
(2) Supraorbital nerves.
(Q.177)

The three germ cell layers in embryo are formed during?


(a)

Ovulation

(b)

Neurilation

(c)

Gastrulation

(d)

Insulation

Your Response :
Correct Answer :
Exp:

c
Gastrulation.
Ref: Langmans Medical Embryology (10th edition)
The most characteristic event occurring during the third week of
gestation is gastrulation, the process that establishes all three germ layers
(ectoderm, mesoderm, and endoderm) in the embryo. Gastrulation begins
with formation of the primitive streak on the surface of the epiblast.
Thus, the epiblast, through the process of gastrulation, is the source of all
of the germ layers, and cells in these layers will give rise to all of the
tissues and organs in the embryo.

(Q.178)

The following are supplied by Posterior cerebral artery except:

(a)

Mid brain

(b)

Pons

(c)

Thalami

(d)

Striate cortex

Your Response :
Correct Answer :
Exp:

b
Pons
(Ref. Harrison 17th/pg. 2525)
The posterior circulation is composed of the paired vertebral arteries, the
basilar artery, and the paired posterior cerebral arteries.
PCA supplies ipsilateral subthalamus and medial thalamus and in the
ipsilateral cerebral peduncle and midbrain; and it also suppliesthe medial
temporal and occipital lobes, in which is striate cortex.

(Q.179)

The following are present in the floor of 3rd ventricle except:


(a)

Infundibulum

(b)

Mammillary bodies

(c)

3rd cranial nerve

(d)

Optic stalk

Your Response :
Correct Answer :
Exp:

c
3rd cranial nerve.
(Ref. BDC Vol. Exp. 4th/ pg. 367)
The roof is formed by a layer of epithelium, which stretches between the
upper edges of the lateral walls of the cavity and is continuous with the
epithelial lining of the ventricle. It is covered by and adherent to a fold of
pia mater, named the tela chorioidea of the third ventricle, from the under
surface of which a pair of vascular fringed processes, the choroid
plexuses of the third ventricle, project downward, one on either side of
the middle line, and invaginate the epithelial roof into the ventricular
cavity.
The floor slopes downward and forward and is formed mainly by the

structures which constitute the hypothalamus: from before backward


these are: the optic chiasma, the tuber cinereum and infundibulum, and
the corpora mammillaria. Behind the last, the floor is formed by the
interpeduncular fossa and the tegmenta of the cerebral peduncles. The
ventricle is prolonged downward as a funnel-shaped recess, the recessus
infundibuli, into the infundibulum, and to the apex of the latter the
hypophysis is attached.
The anterior boundary is constituted below by the lamina terminalis, a
thin layer of gray substance stretching from the upper surface of the optic
chiasma to the rostrum of the corpus callosum; above by the columns of
the fornix and the anterior commissure. At the junction of the floor and
anterior wall, immediately above the optic chiasma, the ventricle
presents a small angular recess or diverticulum, the optic recess. Between
the columns of the fornix, and above the anterior commissure, is a
second recess termed the vulva. At the junction of the roof and anterior
wall of the ventricle, and situated between the thalami behind and the
columns of the fornix in front, is theinterventricular foramen (foramen of
Monro) through which the third communicates with the lateral ventricles.
(Q.180)

Urethral crest is present in?


(a)

Prostatic urethra

(b)

Membranous urethra

(c)

Bulbar urethra

(d)

Penile urethra

Your Response :
Correct Answer :
Exp:

a
Prostatic urethra.
(Ref. BDC anatomy 4th ed. vol.2- pg.349; Oxford Textbook of
Surgery /2nd pg. 1430)
Prostatic urethra
Prostatic urethra is widest and most dilatable part of male urethra.
Measures 3cm in length.
Semilunar on transverse section on with its convexity directed
forwards/anteriorly and concavity posteriorly.
Posterior wall of prostatic urethra is characterized by:

Urethras crest/verumontanum,
Colliculus seminalis and
Prostatic sinuses of openings of 20-30 prostatic glands.
Prostatic utricle is homologus to uterus on vagina.
Membranous urethra is narrowest and least dilatable part of male urethra
containing bulb.
PeriUrethral glands of cowper are present.
(Q.181)

Pneumatic bones are following except?


(a)

Mastoid

(b)

Maxilla

(c)

Mandible

(d)

Ethmoid

Your Response :
Correct Answer :
Exp:

c
Mandible. (Ref: BDC Handbook of general anatomy/Exp.)
Sesamoid bones
1 Patella
2 Pisiform
3 Fabella
4 Sphenoid
Pneumatic bones
1 Ethmoid
2 Maxilla
3 Mastoid (temporal)
4 Frontal
Membranous (Dermal) bones
1 Skull vault bones

2 Facial bones
(Q.182)

Joint between the Growth plate and diaphysis is:


(a)

Primary cartilaginous joint

(b)

Secondary cartilaginous joint

(c)

Fibro-cartilaginous joint

(d)

Gomphosis joint

Your Response :
Correct Answer :
Exp:

a
Primary cartilagenous joint. (Ref. BDC vol.1/ 4th/ 154; Snells
antomy 7th/pg. 15-17)
Primary cartilagenous joint
A primary cartilagenous joint is joint where two bones are separated
from each other by hyaline cartilage at the joint.
Movements are not possible at this joint.
- E.g.
The joint between epiphysis (growth plate) and diaphysis of long bones.
The joint between the first rib and sternum.
B. Cartilagenous joints
Primary (Synchondrosis/hyaline cartilageous) (AIIMS may 2004)
Joint between epiphysis and diphysis of long bones

o Spheno-occipital

First chondrosternal
Costochondral
Secondary(symhy sis/fibro cartilaginous)
Symphysis pubis
Manubriosternal
Intervertebral
(Q.183)

A. surgeon resects the part of left lobe of liver adjacent to falciform ligament. Which
of the following segments surgeon resected?

(a)

Segment 1 and 4a

(b)

Segment 2 and 3

(c)

Segment 1 and 3

(d)

Segment 1 and 4b

Your Response :
Correct Answer :
Exp:

b
Segments 2 and Exp.
-------------------------------------------------------------------------------------------------------------------------------------------------------Conventional Terminology

Couinaud's Segments

------------------------------------------------------------------------------------------------------------------------------------------------------Extended right hepatic lobectomy

4, 5, 6, 7, 8 (1)

Right trisegmentectomy
Right hepatic lobectomy

5, 6, 7, 8

Extended left hepatic lobectomy

2, 3, 4, 7, 8 (1)

Left trisegmentectomy
Left hepatic lobectomy

2, 3, 4, (1)

Left lateral segmentectomy

2, 3

Left lateral lobectomy


---------------------------------------------------------------------------------------------------------------------------------------------------(Q.184)

The following are derivatives of Paramesonephric duct except:


(a)

Uterus

(b)

Ovary

(c)

Fallopian tube

(d)

Upper vagina

Your Response :

Correct Answer :
Exp:

b
Ovary
(Ref. Langmans medical embryology pg. 360; Fig. Exp.24)
At first both the male and the female have 2 pairs of genital or sex
ducts:
Mesonephric (Wolffian medial) and
Paramesonephric (Mllerian lateral) ducts.
The paramesonephric ducts in the male degenerate except for a small
portion at their cranial ends, the appendix testis.
The paramesonephric ducts develop into the main genital ducts of the
female.

(Q.185)

Myelin sheath is synthesized by?


(a)

Oligodendrocytes

(b)

Microglia cells

(c)

Astrocytes

(d)

Tanycytes

Your Response :
Correct Answer :
Exp:

a
Oligodendrocytes.
(Ref. Human Histology by IB Singh, 3rd / pg. 12-37, 165)
Schwann cells myelinate the peripheral nerves. These cells originate
from neural crest, migrate peripherally, and wrap themselves around
axons, forming the neurilemma sheath. The myelin sheath surrounding
nerve fibers in the spinal cord has a completely different origin, the
oligodendroglial cells.

(Q.186)

The nerve fibers of all the following cranial nerves are related to nucleus ambiguous
except:
(a)

Hypoglossal

(b)

Accessory

(c)

Vagus

(d)

Glossopharyngeal

Your Response :
Correct Answer :
Exp:

a
Hypoglossal
NUCLEUS AMBIGUOUS
Most axons are in CN X
Innervates striated muscles of palate, pharynx, larynx
Efferent limb of gag reflex
Some axons are in CN IX (glossopharyngeal) from rostral portion of
nucleus ambiguous
Innervates stylopharyngeus
Dysfunction is difficult to identify
Some axons are in CN XI (spinal accessory) from caudal portion of
nucleus ambiguous
Joins CN 10 and innervates same structures
Lesion causes: difficulty swallowing (because innervates pharynx)

(Q.187)

Capacitation takes place in?


(a)

Vas deferens

(b)

Epididymis

(c)

Seminiferous tubules

(d)

Female genital tract

Your Response :
Correct Answer :
Exp:

d
Female genital tract.
(Ref. Langmans Embryology 9th/pg. 38)
Capacitation is a period of conditioning in the female reproductive tract
that in the human lasts approximately 7 hours. Much of this conditioning,

which occurs in the uterine tube, entails epithelial interactions between


the sperm and mucosal surface of the tube. During this time a
glycoprotein coat and seminal plasma proteins are removed from the
plasma membrane that overlies the acrosomal region of the spermatozoa.
Only capacitated sperm can pass through the corona cells and undergo
the acrosome reaction.
(Q.188)

Killians dehiscence involves?


(a)

Superior constrictor and cricopharyngeus muscle

(b)

Superior constrictor and inferior constrictor

(c)

Middle constrictor and inferior constrictor

(d)

Inferior constrictor and cricopharyngeus muscle

Your Response :
Correct Answer :
Exp:

d
Inferior constrictor and cricopharyngeus muscle.
A Zenker diverticulum is a herniation through a weak area between the
inferior pharyngeal constrictor fibers and the cricopharyngeus muscle a
area known as a Killian dehiscence.

(Q.189)

Crypts of Lieberkuhn are found in?


(a)

Distal Colon & small bowel

(b)

Fallopian tube

(c)

Large bowel

(d)

Small bowel

Your Response :
Correct Answer :
Exp:

d
Small bowel.
(Ref. Human anatomy by BDC 4th/pg. 245; Ganong 23rd/Chapter
26, Figure 26 2)
Small intestine is characterized by intestinal glands or crypts of
Lieberkuhn, which a are simple tubular glands distributed over the entire
mucous membrane of the jejunum and ileum. They open by small
circular apertures on the surface of mucous membrane between the villi.

They secrete digestive enzymes and mucus. He epithelial cells deep in


the crypts show a high level of mitotic activity. The complete epithelial
lining of the intestine is replaced every 2 4 days.
(Q.190)

The function of Paneth cells:


(a)

Secretion of Mucus

(b)

Secretion of Peptides

(c)

Secretion of lysozyme

(d)

Secretion of Alkaline fluid

Your Response :
Correct Answer :
Exp:

c
Secretion of lysozyme.
(Ref. IB Singh histology 2nd ed. 220; Ganong Physiology 22nd ed.
506)
Paneth cells are known to produce lysozyme & phospholipase A, which
destroys bacteria. They are also known to secrete defensins.
This battery of secretory molecules gives Paneth cells a potent arsenal
against bacteria, fungi and even some enveloped viruses.

(Q.191)

The following drain in IVC except:


(a)

Right testicular vein

(b)

Left testicular vein

(c)

Right renal vein

(d)

Left renal vein

Your Response :
Correct Answer :
Exp:

b
Left testicular vein.
(Ref. BDC 4th Ed. Vol-2; Pg. 218)
The veins emerging from the testis form Pampiniform plexus. The
anterior part of the plexus is arranged around the testicular artery, the
middle part around the ductus deferens and its artery, and the posterior

part is isolated. The plexus condenses into 4 veins at superficial inguinal


ring, into two veins at deep inguinal ring. The veins accompany the
testicular artery. Ultimately one vein is formed which drains into the IVC
on the right side, and onto the left renal vein on the left side.
(Q.192)

Carcinoma of prostate is common in?


(a)

Central zone

(b)

Transition zone

(c)

Peripheral zone

(d)

Equal in all zones

Your Response :
Correct Answer :
Exp:

c
Peripheral zone. (Ref. BDC 4th/VolExp./pg. 373)
The outer larger zone is composed of large branched glands, the ducts of
which curve backwards and open mainly into the prostatic sinuses. This
zone is frequently the site of carcinoma.
The inner smaller zone is composed of the submucosal glands opening in
the prostatic sinuses, and a group of short, simple mucosal glands
surrounding the upper part of the urethra. This zone is typically prone to
benign hypertrophy.

(Q.193)

Anorectal ring is formed by the following except:


(a)

Puborectalis

(b)

Superficial part of external sphincter

(c)

Deep part of external sphincter

(d)

Internal sphincter

Your Response :
Correct Answer :
Exp:

b
Superficial part of external sphincter.
(Ref. BDC 4th/VolExp./pg. 383; )
Anorectal ring is a muscular ring present at the anorectal junction. It is
formed by the fusion of the puborectalis, deep external sphincter and the

internal sphincter. It is easily felt by finger in the anal canal. Surgical


division of this ring results in rectal incontinence. The ring is less marked
anteriorly where the fibers of the puborectalis are absent.
(Q.194)

Bursa that communicates with the knee joint?


(a)

Suprapatellar bursa

(b)

Prepatellar bursa

(c)

Superficial infrapatellar bursa

(d)

Deep infrapatellar bursa

Your Response :
Correct Answer :
Exp:

a
Suprapatellar bursa.
(Ref. BDC 4th/VolExp./pg. 147 )
The Suprapatellar (Quadriceps) Bursa
This large saccular extension of the synovial capsule passes superiorly
between the femur and the tendon of the quadriceps femoris muscle.
The clinically important suprapatellar bursa extends about 8 cm superior
to the base of the patella.
The suprapatellar bursa permits free movement of the quadriceps tendon
over the distal end of the femur and facilitates full extension and flexion
of the knee joint.
The bursa is held in position by the part of the vastus intermedius muscle
called the articular genus muscle.

(Q.195)

Luminal narrowing of which of the following vessels would compromise blood flow
through the renal arteries?
(a)
(b)
(c)
(d)
Your Response :

Abdominal aorta
Celiac trunk
Common iliac artery
Inferior mesenteric artery

Correct Answer :
Exp:

a
Abdominal aorta
The renal arteries emerge from the abdominal aorta at about the level of
the L1/L2 intervertebral disk and travel at nearly right angles to it (on the
right, passing posterior to the inferior vena cava) to enter the hilum of the
kidney.
The celiac trunk gives off the common hepatic, splenic, and left gastric
arteries.
The common iliac artery gives off the internal and external iliac arteries.
In addition, an unascended pelvic kidney may be supplied by the
common iliac artery.
The inferior mesenteric artery gives off the superior rectal, sigmoid, and
left colic arteries.
The superior mesenteric artery gives off the inferior
pancreaticoduodenal, intestinal (ileal and jejunal), right colic, middle
colic, and ileocolic arteries.

(Q.196)

The anterior margin of epiploic foramen of Winslow contains:


(a)

Portal vein, Hepatic artery and bile duct

(b)

Caudate lobe

(c)

IVC

(d)

Pancreatic duct

Your Response :
Correct Answer :
Exp:

a
Portal vein, Hepatic artery and bile duct.
(Ref. BD Chaurasia, Human Anatomy Vol II, 4th/pg. 231-232)
Foramen of Winslow/ Epiploic foramen
Boundaries

Of Epiploic foramen

1 Anteriorly Right free margin of lesser sac omentum (containing


portal vein, hepatic artery & bile duct)
2 Posteriorly

IVCQ, suprarenal gland & T12 vertebra

3 Superiorly

Caudate process of liver

4 Inferiorly
(Q.197)

1st part of duodenum & horizontal part of hepatic artery.

Which of the following is not associated with Eustachian tube?


(a)

Tensor tympani

(b)

Palatoglossus

(c)

Levator veli palatini

(d)

Tensor veli palatini

Your Response :
Correct Answer :
Exp:

b
Palatoglossus
There are four muscles associated with the function of the Eustachian
tube:
Levator veli palatini
Tensor tympani

(Q.198)

Salpingopharyngeus
Tensor veli palatini

Corpora arenacea are found in?


(a)

Posterior pituitary

(b)

Corpus striatum

(c)

Pineal gland

(d)

A.mygdaloid body

Your Response :
Correct Answer :
Exp:

c
Pineal gland.
Sections of the penial gland stained with haemotoxylin and eosin reveal
little details. The organ appears to be a mass of cells amongst which
there are blood capillaries and nerve finbers. A distinctive feature of the
penial in such sections is the presence of irregular masses made up
mainly of Calcium salts. These masses constitute the corpora arenacea
OR brain sand.

(Q.199)

All the following are derivatives of the neural crest, except:

(a)

Melanocyte

(b)

Adrenal medulla

(c)

Sympathetic ganglia

(d)

Cauda equina

Your Response :
Correct Answer :
Exp:

d
Cauda equina.
(Ref. Human Histology by IB Singh, 5th/ pg. 321)
Neural crest derivatives:
a. ANS,

b. Dorsal root ganglia,

d. Chromaffin cells of adrenal medulla,


f. Pia and arachnoid,
g. Celiac ganglion,

(Q.200)

e. Enterochromaffin cells,

h. Schwann cells,

j. Parafollicular (C) cells of thyroid,


Bones of the skull.

c. Melanocytes,

i. Odontoblasts,

k. Laryngeal cartilage,

Which of the following is true regarding gastrulation?


(a)

Establishes all the three germ layers

(b)

Occurs at the caudal end of the embryo prior to its cephalic end

(c)

Involves the hypoblastic cells of inner cell mass

(d)

Usually occurs at 4 weeks

Your Response :
Correct Answer :
Exp:

a
Establishes all the three germ layers
(Ref. Human Histology by IB Singh, 7th ed., pg41; Langmans
embryology 9th/pg. 65)
Gastrulation
It is the most characteristic event occurring during the third week of
gestation

l.

Gastrulation is process that establishes all three germ layers (ectoderm,


mesoderm, and endoderm) in the embryo.
It begins with formation of the primitive streak on the surface of the
epiblast.
Initially, the streak is vaguely defined, but in a 15- to 16-day embryo, it
is clearly visible as a narrow groove with slightly bulging regions on
either side. The cephalic end of the streak, the primitive node, consists of
a slightly elevated area surrounding the small primitive pit. Cells of the
epiblast migrate toward the primitive streak. Upon arrival in the region of
the streak, they become flask-shaped, detach from the epiblast, and slip
beneath it. This inward movement is known as invagination.
Once the cells have invaginated, some displace the hypoblast, creating
the embryonic endoderm, and others come to lie between the epiblast and
newly created endoderm to form mesoderm. Cells remaining in the
epiblast then form ectoderm.
Thus, the epiblast, through the process of gastrulation, is the source of all
of the germ layers, and cells in these layers will give rise to all of the
tissues and organs in the embryo.

Das könnte Ihnen auch gefallen